You are on page 1of 471

Table of Contents

Supject Paga
a Fertitizotion 1

' Plocenta 5
'? Amniotic fluid 1t
? Umbilicol cord 7?

' Fetol Circulotion 13


' Moternol odoptation t4
' Diognosis of pregnqncy 20
' 'Antenotal core Z3
q Post notol care 34
' Assessment Of fetal well-being 36
o Ante-portum ossessment Of fetal well-being 36
o rntraipartum ossessment of fetsl wetl- being 43
o Amniocentesis 45
o Congenitol fetol molformotions 48

Supject Paga

AAinor disturbonces during Pregnoncy 55


Obstetric hemorrhoge 57
o Abortion 57
o Ectopic preglqncy 79

o 6esfotionol trophbblosfic diseoses 9?

Ahteportum hemorrhoge 101

Cordioc diseosos with Pregnoncy 114


Vomiting with pregnoncy t23
Diobetes mellitus with Pregncncy 127'
Hypertensive disorders with pregnoncy 141

Moterno! mortality rote 160


I III III ITI III III ITTII'

tl
rl
-A -II

I
I

regnancu I
lllrrr rrr rrr r-r rrr rrr rtJ
-rr
* Definition:
9 It is the union of:
* The mature spermatoZoon &
tthe secondary oocyte dlt+
"Ji
':

To form the zygote


* Sitea In the lateral 'L/3 of the Fallopian tube (ompullo l."iJl ots-lt;
* Timing: Within 24 hours after ovulation.

* Tronsoort of soerms
? Matuie sperms (22X or 22y) ascend up in female genital system
t This is helped by the seminol prostoglondins.
? Seminal PG increases(Rt:
G Sperm motility
G Uterine & tubol peristolsi5 + Upward suction of sperms.
G Sperms reach fallopian tube within 4O min.
? Then capacitation of sperms occurs within 2-6 hrs

? Loss of its glycoprotein coat to help fertilization.


? It is initiated once the sperm pass the cervix
T It is helped lastly by zonal proteins & follicular fluid.

? Increase of acrosomal permeability


I This leads to production of hyolouronidose
? Hyalouronidase facilitates penetration of zona pellucida.

g Ovulation dccurs after completion of meiosis f -, 2ry oocyte


+1st polar body.
I Ovum (immotile) pick up by tubal fimbria (fimbria ovaniea #_rht),
, PG from follicular fluid increases tubal & cilial motility
ttin/ yf
Dorsa/ rnese.,ty
(D)

Tish't
**

( Di1/oi/1 2/
llilcts;

Meio sis $)
t"los;s (!)

(
I shorl neck 2 ccalriolct'

loal ta;[
I Sperms become attached to specific receptors on the zona
pellucida (ZP) with loss of the outer acrosomal membrane
I Then hyalouronidase release with dissolution of cumulus
oophorous & corona radiate (granulose cells qr.l 6prrt).
? Only 1 sperm penetrates the ZP(vesicular mole,lslYtj).
t Polyspermia@ is prevented by
zonal block (zonal hardening & becomes impermeable)
G Formation'of the fertilization membrane due to release of
cortical granulesl {4}ll i,. ) in the perivitelline space.

After fertilization: meiosis II is completed -r formation of


femate pronucleus (mature ovum * the 2nd polar body).
I The'head of the sperm swells to form the mate pronucleus &
loses its tail & body
I The nuclear membrane around the 2 pronuclei disappears with
fusion of chromosomes >> 46xx or 46xy zygote.

I Within 24 hqurs The zygote divides to form morulo (round


mass of 16 cells called blostomeres 1c -."1 ^rl* .Js )
I Themorulo reaches the uterus 3 days,post fertilization being
G nourished by tubal milk
G Aided by tubal motility & cilial movement.
t Fluid collects in the morulo to form blostocyst;
r* is nourished by endometriat secretions = the uterine milk.
* It has an inner cell mass 1OO-250 cells (Embryonic mass
+ embryo) & outer cell mass (trophoblast {placenta).
* It is free in the uterus for 3-4 davs, then corona radiate &
zona pellucida disappears (ectopib lJd^1 Dtii.le 0HLiU lr:ll)
I At 7th dav it has 2 layers (endo & ectoderm)
? At lOth dav it has 2 sacs (amniotic & yolk sac)
? At 15th day development of the mesoderm (3rd layer).
I Embryonic period (embryo) extends to the end of 7th weekof
intrauterine life at which major organs are formed then after
the 7th week it bgcomes the fetus
,*..

's r'70'y

7na/ bto cK q/z


)+rl {-nf,'l;Zafro'n oF
2r/ oocyfe

e
ttP' ),o:) 2J oo c/ rt Hat /r'
-7 Qlum t 2,rl 1ola, l,f,

Z yfote
o*1'r
Z
1F.il, ,'
O^pu//*
.ti
. orr?a
a)J LA s
Definition:
I tt is embedding of the blosiocyst into the deciduo lit+-
Timino
C Zth doy after fertitization (maximal tevet of P from corpus tuteum).
Illechonism:
-C
Trophoblast sinks in stratum compactum titt the Nitabuch
layer.
C Nit"brch tayer is a fibrinoid loyer in the decidua (few mm deep)
if
absent will lead to morbid odherence (accrete, .increta &
percreta) with failed placenta delivery
C Imptantation bleeding may occur (Hartman signt5t+ 1

G the btastocyst secretes HCG which maintains corpus luteum


) e+p ) endometrium becomes decidua + amenorrhea.

C Decidua basalis:
.It is deep to embedded ovum.
: It forms the maternal part of placenta.
Q Decidua capsularis:
. It covers the ov & separates it from uterine cavity.
C Decidua vera (parietalis):
. It lines the rest of uterine cavity.
nction of Deciduo
1. Nutrition * Implantation of the blastocyst.
2. Form placenta.
3. Protective against invasive power of trophoblast.
4. Immune protection (against law of transplantation)
* Woves of invosion:
C Chorio-decidual space:
@ It is between the trophoblasts & the decidual vessels
q
Trophoblasts invade the decidual vessels to transform the
@

chorio-deciduat space into the intervillous space.


C 2rv wave of invasion (at 20th week),
@ Trophoblastic invasion of the media of the vessets (sinuses) &
allowing blood to circulate in this space (absent in preeclampsial.
f,a Jor.e.{=iqL Co,vi ly

C.G?.

,11 d'
inylantah6o

Dcc;J"aL
.Sh. 6 otsa.l;r 6Fcce-
jltF'
t 2 wtceKr

TTere are two w ave-.f oFinvdsion.


Deei lual bloel ycttel,

Chor;o -Drri)uaLff"r.

Dec;luaL fioute<
*
r. Cyto-trophoblast (Langhans'cells): Mesoderm
q Large mononuctear cells (inner layer).
z. SJrncitio-trophoblast :
c 4 sheet of protoplasm without cell membranes (outer layer).
Mesoderm appears inner to Cytotrophoblast.
G The 3 laJrers: (chorion)

r. Bqisns,u-xllls y cyt o-t ro p h o b I a st a n d cyto - t ro p h o b I a s t ) .


-( S n i

z. 9"e-ggndgsxJillgg,-( p ri m a ry v i I I u s * m e so d e rm ) .
r-LertkEyJr.L[ls -" Seco n d a ry vi I I us
+ feta I b ood vesse ls" . I

4-Anshp:.L+9,-yltLi Proliferation of trophoblast of the villi & not


invaded by mesenchyme but anchored to the decidual plate.

frondosum (form the fetal part ofthe placenta).

* Vitti related to d. capsularis disappear leaving a smooth membrane chorion


leave) . it is in contact with the amnion to form the avascular amniochorion
lF Also decidua cansularis unites with decidua vera.

# Origin
* It forms the amniotic sac & surrounds the feitus containing the
amniotic fluid. It is single layer from the fetal ectoderm.
* It is O.5 mm -* 5 layers (non ciliated cuboidal cells +
Basement membrane + compact layer + fibroblastic layer +
spongy layer which contain mucus can slide on the chorion).
* ns:
L. The amnion is in contact but not adherent to the chorion leave.
2. Placental amnion overlies the surface of the placenta
3. Amnion covering the umbilical cord
* function:
* It secretes vasoactive peptides ) regulate the flow ln
chorionic vessels.
trophoblor
F;br;a
Fbg
CAori6nl s tl,
v;
* (klL dt z!
,ytaL
c vlttl .,

3r/ villut
C Hehrc ttillus)

Chorion F-oaJostr*1
F,
s -+
JU
0 'basalis
I
lhe plecenta,

>-/---/fN.)e:_S
G Fetol oort:
Q Choroin frondosum mesoderm + fetal vessels
= Trophoblast +
(chorionic artefies & veins) projecting as villi into the intervillous
--!{_

space.
G Moternol port:
+ Decidua basalis (decidual/basal plate).

* Shooe
+
@ Discoid, villous, hemochorial structure
* Site
)( Fundus = Upper uterine segment 99.5%,60% posterior (implantation &tl'.)
)( Sit. of cord attachment: eccentric
* Size:
$ Thickness: 2.5 cm in center & gradually tapers towards periphery.
o Diameter: l5-20 (+18) cm in diameter
9 Weight: 5009 (l/6 of the fetal weight at term).

It hos 2_'.ortoces

UF"tol surfoce
a Smooth glistening g+l+
+ It is covered by amnion .r

+ The umbilical cord is inserted to this surface.


UiAot"rnot surfoce
o Dull red, irregular.
+ It is divided by grooves, to l5-20 unequal lobes called "cotyledons".
9 Each cotyledon is supplied by a branch of the chorionic artery & a vein.
+ It is covered by a gray layer of decidua (the basal plate @).
N,B: Fetol membrlanes
The omnion u+ irr chorlon u d!.
\r'
x
Nr'
The inner membrane /\ The outer membrane
\,
,\ Covers the fetal surface of the ,l In contact with the uterine wall, it
placenta & the cord ends at the margin of the placenta
br' Less transparent
)(' Transparent, glistening grayish et

/z--.\7N
I AdYector st.ff- in
a>* -lhq f,^,.du1

Hq,l-ernef s;rrli^te '

f(qfernaL
g*rf acq,.

- Ou// reJ
1\e srJ is
-,DivilcJ info
eccenferi<
CofyleJons
Ye.ssefr on thc
-Sur fa';ce--
btt uP , ., 2,5 c^
FlaL me,^L.

2o
E,'
u_, b3
\t0 Chtie 4

o{rtriltu)l2Jt2
D".t.<r^^ ;r-;r:U
Functions of the placenta l+ rb fJEj dlJ*)ao

r. Respirotory function : COz and 02 transfer by simple diffusion.

a.Immunologic: (IgG cah pass but not A or E or M).


a. HemoPoietic: It forms HbF.
s, Enzymotic
$ oxytocinase, insulinase, heat stable alkaline phosphatase & histaminase.
e. Endocrine funcfions t+ Lta .bi:ts@6*

* a (92 amino acids common for F.S.H., L.H. & T.S.H.)


* p subunits (145 AAs & specific)
* Carbohydrates part (glucose, fructose, glucosamine, sialic acid which increase
the half life by decreasing the hormone clearance).
Site of production o
+ Syncytiotrophoblast (previously thought cytotrophoblast)
Time of p!"oduction
a App.u.s at 1'B day of implantation
a Th. doubling time is 1.4 - 2 days.
Peok
* It reaches a peak (50 000 mIU/mL'l at 8th -10th 1vft l\r a4.r
o Then falls to 5000 ru/L by about 15 wk (100-130 day)
o Then remains at this level till near term.
o It increases in muttiple preonancv & vesicular mole
Disoppeoronceo@
'." 2 weeks after delivery
* 4 weeks after abortion
* 8-12 weeks after evacuation of a hydatidiform mole.
Assessed by
o Urine analysis (conventional oregancy iest ) ELISA 90o/o accurate) g 1
wk after missed period s/t+Jl-164 ) at 50 mu/ml
o Serum pregnancy (RIA most sensitive) ) 1 wk after fertilization
(before missing a period) c'it--,| ;t& + at 5 mu/ml
-.- lfu,non chor;on; c Aoyl'lflilts
lhe

9LV c-o Prote i (\


1,J,/:l tyeciFic
fta/,'c aCid rtoa

P"o ,r^ _ ,ot w (


f,otcco

f,yac;l;o. e 4!:-
4.,_ ,-;- J/:
t4 ctt'iae ua6,\an3eJ

ut

inTlanfetion ,2
fz
,\
.9

Hcq aJuuri b
*ve 6eJ Lo.ck zuu
oValue of B-subunit stimotion
o Diognostic:
- Diasnosis of pregnancy normal pregnancy, missed & threatened abortion
(doubling very 2 days), ectopic pregnancy (UiS+B-subunit)
- Diagnosis and follow up of trophoblastic tumors, genn cell ovarian tumors
(choriocarcinoma, embryoma), in some testicular tumors
+ Thenopeutic: threatened abortion & undescended testis in males.
. Funclion
1. Maintains the CL of the pregnancy till development of the placenta at 1Oth wk.
') .'
Immun omodulation during pregnancy
3. It directs male development: testosterone production & testicular descent.

. Natune
' It is"a single chain polypeptide l9l amino acids.
j ' It is similar to GH & PR.L hormone.
squrce.
+
Syncitiotrophoblast
. Time of production
o It appears at 3rd week postfertilization then increases progressively
o It reaches a plateau at 36 weeks.
. Level
* Tpglmt at 36weeks
* .4pgtml considered fetal danger zone after 30 weeks
O But in some, cases there may be no HPL production. '
.
+ It is a good indicator of functioning placental mass
+ So it is used in cases of high risk pregnancy as PE, postmaturity, IUGR.
+ It is of little value in erythroblastosis fetalis, diabetes due to large placenta.
,o Its increase reflects placental weight not function.
.
1. Similar to"GH and Pnolactin ) stimulates growth of breasts.
2. On CHO & fat: lipolytic + inhibits glucose uptake by the mother & inhlbitr
gluconeogenesis (anti-insulin effect) spares glucose and amino aoids for fetus.
3. [t is luteotrophic,
somatotrophio, mammotrophic & lactogenic.
H u mo n chorionic thwotropin

Humon plocentol proloctin


+ PAPP-A i! fin PE @ar
Relaxln
'. + It is released by CL (main source), decidua & the placenta (chorion)
1€. It may cause uterine relaxation, softening and effacement of the cervix

* Inhibin inhibits the secretion of GnRH and HCG.


.3 Activin stimulates the release of these hormones.

Sourcet
G It it formed bLCL in the first,7 wk.
E Tt.o {CL & placenta} till l0'h week
I thm completely produced by placenta (Syncytiotr.) after 1Oft week.
t Level
a It is 50ng/ml at Twks
A Then small fall in the next 3 wks
t Then rises till near term (l50ng/ml)
Then J t wtr before labor.
^ It is excreted in urine as pre!flondiol lq l.rg
A
t Eglgllqr
I Tocolytic
' - Hypertrophy & hyperplasia of smooth muscles fibers with estrogen.

' Proliferation of breast acini to prepare for lac.tation.

' Formed by the ieto-olacental unit (E3) 15 r.g.


' They are E$rgslEl), Estradiol (E2), $.1!4!q!(ED & EstetrDl (E4 er- t-i.)
r E3:82:Elin non-pregnant is 3:2:l & in pregnanc El &82 are T l0O
t
times while E3 (90% of the E during pregnancy) is t OOO times.
. Function
i. Responsible (+Progesterone) for changes in genital tract & brehst.
ii. Value of E3: 24 hrs urine E3or serum E3 @etter) in diabetic preghancy
(daily), IUGR (every 2 days), postmaturity (raising or plateauing values)
z.Nutrition
') Placental transfer is by:
,,. Simple diffusion for H2O, electrolytes
.t Facilitated diffusion for glucose; amino acids & lactate
+ Active transport for vitamins, Ca, iodide, Phosphorus
+ Endocytosis for immunoglobulins and lipoproteins.
s7#

s. Borrier function:
It prevents passage of many organisms to the fetus, but many organisms can pass as
o Viruses rubella, chicken pox, measles, mumps, CMV, polio.
t) Bacteria treponema, TB
o Parasites malaria, toxoplasma
g Drugs: anticoagulants & oral hypoglycemic

1. Endothelium of the fetal blood vessels (contain no smooth muscles)


2. Connective tissue s s, Cyto & Syncitio-trophoblast
3. Cytotrophoblasts s om the 5th month & syncitiotrophoblasts
from the 6tn month e placental barrier

Abnorma I ities of the pI acenta


In the shape of the placento:
o Bilobate: the placenta is made of 2 lobes connected by placental tissue.
I Bipartite: Made of 2 equal separate lobes connected by membrane.
+ Succenturiate olacenta:
- Presence of accessory small Part .
- it may be retained in uterus leading to PPH and puerperal sepsis
- Known by presence of torn blood vessels crossing the placental margin.
+
- The chorionic plate is smaller than the basal plates.
- This appears as a central depression surrounded by a thickened white
decidual ring (double fold of amnion & chorion).
- Leads to Abortion, Preterm labor, CFMF, IUGR, IUFD, accidental Hge.

a Placenta fenestrata: a hole is present @


+ Membranaceous placenta
- Chorion leave doesn't atrophy ) thin large placenta (15 - 20 inches). ,

- It doesn't interfere with fetal nutrition but can cause serious bleeding
(associated with placenta previa & vasa previa) & may not separate
' easily and manual removal is difficult.

q Placenta previa
+ Ectopic pregnancv.
+ On septum: abortion, preterm labor, antepartum or postpartum Hge &retention
f Lo^cerr bL b aceicf.--

e leclo\troc
Hzo

€e

Achvt bc"sr
l a.q-
(e rP
Aur)

OPinoc?Fosis
Abrrroraa ctL pfactaliaL shapts
r?

fl <"'. b ra,*Ce4,ut

Ciratntvallate 2L'
. ln the attachment of uterus
o Placenta accreta the villi are just in contact with the myometrium (<50% of
muscle thickness).
o villi penetrate into the muscles (>50% of thickness).
P[acenta incf€th the
o@thevilliperforatetheuterus&reachtheperitonealcoat.
. Abnormot cord insertioni
+ Mareinal insertion: Battledore clp,ilt +t2. placenta-
.) tion of the cord:
* Vessels are inserted into the membranes.
ir If the traversing vessels pass below the presenting part over the cervix
' thby may rupture ) APH of fetal origin with severe fetal distress.
Diagnosed by Kleihauer-Betke test )>> fetal cells.
+ :It is usually asso.'ured with placenta membranacea.
fn,the sizL of the plocento:
r Large i4 Rh and DM, syphilis, twins, placenta membranacea,
- Syphilis: large, pale, friable, endarteritis obliterans, Spirochetes
- Rh Isoimmunization: large, pa1e, edematous
t Small in IUGR or infarcts ) placental insufficiency.
Inforction of the plocento:
c' in mature placenta.
c due to spasm of spiral arteries 5upp[ying
intervillous space, leading to decreased placental blood flow causing
coagulation & fibrin deposition. This is frequently seen in pre-eclampsia.
Colcificotion: It is correlated with fetal lung maturity.
Plocentol tumor
I Placental Polyp: retained parts of placbnta tfter delivery
.) Chorioangioma: benign neoplasm of blood vessels ) Polyhldramnidb
Q Vesicular mole & choriocarcinoma.

a SAnonumous nannes: Placental dysfunction.


q Definition: it is the failure of the placenta to supply oxygen & nutrients
to the fetus & remove toxic wastes due to I in the placental circulation.
c Etiology: seefUGR
G The effect on oregnancu : IUG& fetal distess & IL]FD
q as IUGR

Page
10
Ab,vrmal atlacArnenl- r-
+5:9
)
\-
s
h

Dcrcref*
l r^-;-a

A6..orrrro.l- CorJ lnscdr'in i_

Vela^ento'ut
B"ll ledocq 6rJ inserh'o 4
ftecenl-a

-RoF\
Vara
r#i a frrvr'q

Ui"i' I ."1 fe+ot- d istrert


* Volume of omniotic fluid

x frspect
? Cleor in early pregnancy
? Opogue later due to vernix Caseosa, lanugo hair and epidermal cells.

? Woter:99Yo
? Orgonic constituentS: (CHO, protein, lipid, horrrones, enzymes)
? Inor:gonic tonstituents: Na, K, Copper & others
! Specific Arovity: 1010-1020
12 Reoction: slightly alkaline 7-7.5

+ fn the f irst trimester: ultra-filtrates of maternal plasma.


+ In 2nd trimester: Fetal urination & swallowing are playing a major role.
+ In 3rd frimester: Fetal urination and swallowing are the major regulators.
+ other sources
Transudation from fetal skin, placenta, cord, GIT & respiratory ffact
+ Then it is removed by transudation to mother & fetal swallowing.
+ A well bolonced circuit (5OO mllhour)
* Functions of the flF fluid
+ During pregnoncy:
tl Af protects the fetus from mechanical trauma.
t lf keeps the fetal temperature constant.
t {[ allows free fetal movement & avoids pulmonary hypoplesia
! ag provides a medium for the fetal excretiort.
t Af prevents adhesions between amnion and fetal Skin.
I Atr' is bacteriostatic.
o During labor':
@ Form,a bag of fore-waters which helps dilation of cervix,
@ Prevents direct placental compression
@ Washes the birth canal.

0 Amniocentesis & Amnioinfusion


.i. Volume: PHA, oligohydramnios
.3. Amniotic fluid embolism.
-l Meconium staining: indicator of distress ) meconium aspiration syndrome
* Amniotic cysts, bands (in severe oligohydramnios, may lead to amputations)

. Origin: from connecting stalk


. Contents: Ulhorton's jelly + 2 ortetrles (non oxygenoted ) + I veln
(oxygenoted )+ remnonts of ollontois * covered by omnlon (glistening)
. Length: It is 50crn
' Diometer:2 cm.
. Vessels are convoluted due to the greater length of vessels than of the
cord
* lnsertioo:€ccentricinT0o/o or at the center "central in}}Yo"
* frbnormolities
I. In the inrertion
(battledore)
oCentaf
o Velanerttous insertion (look page I 0)
2. !-n the lenglh:
a) lbo /on_a > 55cm leads to
trCord prolapsed & presentation
trCoiling around fetus )torsion & stricture of vessels
rue knots.
b) Too'short < 32cm leads to
alpresentatio4S, Non engagement
vulsion of the cord
ailure ofexternal version, failed forceps.
etal distress or rupture of cord and antepartum hemorrhage
cute inversion of uterus.
3. Knotr
' i,
trelnot: Fetus-passes through loop of cord )asphyxia. :

i. EalSg_knog: localized collection of Wharton jelly or localized varices. "Both,


no interference" with fetal circulation.
4. Congenital umbilical hernio & ringle qrtery, myxomo or Jbrcomq
5.
6. Common in DM may be associated with
CFMF espgcially in cardiovascular system, IUGR, prematurity.
True knot False knot
a The umbilicol vein:
t /r carries oxygenoted blood from placenta to fetus
? The vein ascends along the onterior obdominol woll to the liver
t Then divides into Portq! sinus and ductus venosus.
c The ductus venosusi it traverses the liver to enter to inferior veno covo.
G The inferior veno cevo,
| 'It contains mixed (oxygenated and deoxygenated blood), with
. S.V.C. (deoxygenaEed blood)) blood to right atrium
q Bbod ln the right otrlum lt hos ttlo pothutoys:
A. To, left otrium through foromen ovale. bypassing pulmonary circulation
Or
B.Blood in right otrium passes to right ventricle to the pulmonory ortery
& shunted through the ductus orteriosus to the descending oorto.
a The umbilicol orterles:
I Blood is returned to the plocento through the 2 umbilicol
'orteries that
ari'se from internol ilioc orteries.
*Chonoes ot birth:
The umbilicol vessels:
o They are occluded by spasm.
o Umbilical vein) ligamentum teres.
o Umbilical arteries)
The ductus orterlosus:
o Onset of fetal respiration after birth leads to expansion of the lung & J
pulmonary pressure with J in the flow of ductus arteiiosus & obliteration.
o PGE2 maintains ductus patency while prostaglandin synthetase
inhibitors close the ductus)
+ The ductus vanosus ) the lisamentum venosus.
* The foromen ovole:
o Pressure in the left atrium is f due to the I venous return from the lungs
o Pressure in right atrium falls due to cessation ofthe placental circulation.
o Pressure on either side of the foramen ovale is equalized) olosed.

l- HbF (h/gh oxygen carryrng capacitl)


2- | Fetal cardiac output.
3- f Fetal systemic blood florv rates
FeEaL Ci(culatfo,t

S\
s
\
v
.\-
:a
s=

I
fl
i
(
\

umbilical Veia D. VeAoS.it


(lj;. +eres) Ui vVtotut)
IJ Q,QrleriosvS

at
U
i
I fA$; ,i'liacq-
&
* Def inition : it is the physiological changes during pregnancy

Morning sickness, Change in appetite, longing & Sleepy or depressed.


Pituitory chonges:
- Enlarged (especially anterior lobe 2-3x)
- lGH, JFSH, f LH but tpnl (200-400m g/dl)
ThVroid chonges :
- Enlarged nfSH chorbn/c thlrotrzpln)
- TT3, T4 (total not free due toITBA
- tnun by 25% (bypregnancy not hyperthyrordrs@.
Poro Thyroid chonges
- Enlarged, E blocks the action of PTHI'h 14+r.
- Calcitonin is 1 & ionized Ca is not changed.
ndrenols chonges
- Little change in size
-f ataosterone, DOC, Renin, Angiotensin ll (free hormones are not changeo)
- Cortisol is not Changedl+ a-g

I st month
. f size, vascularity , tingling & numbness (1 sensitivity)
9nd months
o Pigmentation of breast sn6 61gs14.'.rjl dJ'! d;i,
. AAontgomery tubercles (/0-20 swo//en orifras of /actea/s orsebaceousg/ands)
o Increase nodularity & size of breasts.
3rd month
.-------{-_
. Pale &'yellow secretion (colostrums)
Sth month
. zry areola (pigmentation around the lry areola)
o Dilated SC veins, stria

o Respiratory Rate is not changed but ted depth


o J Residual volume, Resp. alkalosis (CO2 rvash)
o t tidal volume & respiratory minute volume
o Dvspnea (pressure & progesterone effect later lightening )
t Si Ze , Vuc^rla r;ly t fen s;ti y;t/
.t Pi? *rrn Eh;o

rtunt 1o^"/.

veint
D Veins
. Voricose veins due to:
- Progesterone relaxant effect on vessels
- Pressure by gravid uterus
- f Blood volume
Blood:
. Volune 40-50% (max. 34 wk lr1, l-rlr;

- (hemodilution) & J viscosity. Pathological if less than 10.5 g/dl.


. tWBC's + 16,000/cm3 (more marked peripartum)& ptatelets >> mildJ
. IESR (50mm lst hour due tot fibrinogen & | viscosity)
o lClotting foctors (7,8,9, lO, 12 & fibrinogen )DVT) & J factor 1l &
J fibrinolytic activity
. Minerols & vitomins lr+ 4-e:
- There is t demand for iron exceeds the amounts available in a normal diet.
- | transferring (total iron binding capacity)
- tcopper & ceruloplasmin (estrogen effect).
- f Fat soluble vit & J water soluble vit in blood.
frPex shifted upward & laterally (4th ICS instead of 5th )
Pulse:
. tto-tsb/min
o Heartsounds:
- Splitting of 1't heart sound
- Appearance of3rd heard sounds
o Murmurs:
- Soft systolic murmur
- Diastolic'murmur if occurs we muit exclude pathology
Blood Pressure 4\ ,B
o I In 2nd trimester
) Placenta acts as AV shunt
) Progesterone, estrogen & prostacyclin vasodilator effect
o fln 3rd trimester (t blood volume)
Cordioc output@@
o 30- 50Yo max 24-28 wk due to I stroke volume & heart rate.
o During labor (uterine contraction & loss of placental shunt).
o pa Jiir u"JJl j 4si,a eAl a.r
Moberna,L aJo,. ioa in C Vs :-
b oA'

Blnol ?rctturc
CoP
Btood
Ttx
VoL
( ern
p la,rn^a
, Plt-
WBc s

RBct
ctr
vib
/ n;n.
N-

) The kidney:
@ fncreose size by 1-1.5 cm.
@ f Renol blood f low (50-80%), lateJ. t6lomerutor filtrotion rote (50%)
@ JSUNI (8.5mg/dl), blood urea (2-4 mmol/l), Creotinine (0.46 mg/dl), uric
ocid (3mg/dl), if elevated it is due to hemoconcentration or a decreased GFR),
@ J Bicorbonote (20-24 mmolil), J reabsorption of glucose & albumin
(<300mg124h)
@ lCreaiinine clearonce 720 -160 ml/min
\> Ureters
@ Enlarged, f liability to pyelonephritis.
@ Dilated (P effecl, more on Rt. Side (due to dextrorotatcd uterug
) Blodder
@ f Frequency early (uternepressurQ & late in pregnancy (engagenenl.
@ Hyperemia and hyperplasia of muscles and mucus membranes.

) Pigmenrouon
':' Linea nigra, chloasma, nipple, areola
':' Due tot placental & adrenal steroids & MSH like effect of E.

Y Divoricotion of recti
Y Sometimes: falting of hair, palmar erythema & spider naevi

The mouth:
t Morning sickness ) nausea sometimes vomiting in the I't trimester
' Solivotion (Ptyalism) due to congestion of the salivary glands
' Longing ,..C1 (vitamin deficiency or Psychological)
' Hypertrophy ofgums
) The esophogus
' Heort burn (pyrosis) due to:
- P effect on the cardiac sphincter
- Delayed gastric emptying
Y Stomoch
' Indigestion, delayed emptying, decreased motility
o J acidity of stomach (hypochlorohydrio) ) regurgitation of alkaline chyle
) lntestine Constipation, piles (p effect, pressure of uterus)
Y Liver & goll blodder
' Tendency to cholestasis
a
IGlobulins (immunoglobulins, binding globulins) & j albumin
a
falk phosphatase.

) WeiOht Ooin 4-J*,ill 4i{-...


oIn I't trimester is lkg
ofn 2nd & 3rd trimester is I 1.5 kg "0.5kg / wk from 20 wks onward"

' Pregnancy is diobetogenic (anti-insulin action).


' There is tendency to reno! & olimentory glucosur:io & ropid ketosis.
Y Fot:
' tCentrol fot deposition
' fBlood lipids & cholesterol

' * ve nitrogen bolonce, there is I Kg increase


-
500 g for the fetus & placenta
-
500 g for maternal tissue deposition
There is increosed demond for minerols &vitomins:
' Increased levels of fot
soluble vitomins in blood,
' Decreased level of woter soluble vitomins,

' Serum inon stores may be depleted if no Fe supplementation is given

tlordosis: to compensate for the enlarged uterus


lMobility of pubic joint & sacroiliac joint.
Numbness of UL (due to dropping of shoulders & anterior flexion of the neck).

' tserum level of Ig G, A, Ig M


O.6 ,

-'igt ;
1. Vulval changes
+F f Size, vosculority, congestion ) Jocguemier sign (bluish soft vulva)
a Liable to varicose veins &edema-
2. Vaqinal changes
e J Vosculority -; Chodwick sign (bluish soft vagina),
e I Secretions ) acidic (f lactobacilli ) E effect)
e Thick epithelium (smear )tintermediate cells ) P effect)
3. Gervical changes
. I Vosculority S Goodell sign (bluish soft cervix)
. Ectopy due to eversion & hyperplasia of endocervical glands
. Formation of thick cervical mucus plug (operculum)
4. Fallopian tube
. tVosculority,stretched.
. Hypertrophy of muscles and epithelium
5.
tVusculorJ.ty, no ovulotion (hypoth.-pit. inhibition by placental hormones)
Corous lateum of pregnonc? f in size tilt 7th wks ) E, P, relaxin
- €strogen & Progesterone will be produced mainly from CL till Twks then
production is shared with placenta till 10-12 wks;,then CL will atrophy.
- Reloxin: a protein hormone of unknown function may have a role in ripening
of cervix & relaxation of pelvis at labor.

- CorpuS Luteum Cyst may be found in the l't trimester (f unctional)


- Pfegnancy Luteoma may be formed. It leads to f androgen production )
maternal virilization (reversible). The fetus can escape virilization due to
conversion of androgens to estrogens by placental enzymes)

6. Uterine changes
o Copociry: l}ml ) S t at term
o Consistency: Soft (increase vascularity 500m1lmin & amniotic fluid).
o Shope: Pear shaped ) globular (8wk) ) Then pyriform (2"0 & 3'd trimester)
o Positioni
[l
Dextro-rotated due to presence of sigrnoid colon.
@ Dextro-flexed ) left round ligament becomes nearer to midline.
o Blood supply; l0 times about 500m1 (10%of COP)
o ttleight: increase from 50 gm to 1kg
o Size
. 8wks (5cm) ) l2wks (l0cm) ) l6rvks (l5cm) 'lH

- 12 weeks: At the symphysis pubis


- 20 weeks: lower margin of the umbilicus
- 22 weeks: at mid-umbilicus
- 24 weeks: at upper margin of the umbilicus
- 36 weeks: at the xiPhisternum
- 40 weeks: descend due to engagement.
o illuometrlum
)( HvPertrophY: increase in size of muscular fibers-
)( HyPerPlosio: increaSe in no. of muscle fib'ers.
,\( Controctility:
- Palmer's sign: In 1't trimester, detected by bimanual examination.
- Braxton Hick's contractions later on, felt on abdominal exarnination.
- False labor Pain: Nea.r labor.
- True labor pain
- |fiese contractlbns ard thep/acenta/ ctTcu/ation towards fie fetus.
o Formotlon of LUS
@During Pregnoncy: From the isthm,us (is the region between the

Anatomical internal o, Abor" & histological internat os below)


@tt is 5mm and during pregnancy &labor it is stretched to l0 cm
@During lobor: istbmus * cervix
OPhysiologicol rctroction ring {groove between the LJUS & LUS, it is
not felt or seen (below the SP)),
@Pothologicol retroction ring: it occurs in obstructed labor; it can be
seen & felt (above SP qr.i ds ddl d.L5, 4iU ct o ).
oSoftening of LUS (Hegar's sign).
LUS UUS
Peritoneum Loose Adherent

Muscle layer Thin (2 layers) Thick (3 layers) t'

Decidua Less develoied Well developed

Membranes Loosely adherent Firmly adherent


Passive Active in labor
Action (Dilates & stretches) (Contracts & retracts)
7r"/ t o/ ilc ule r us ,
t-

0
t

- pltyrio/o7ict/ r
- 7 allot.l i ca I

@ Conkach"on Ccon.rkiclio^) ?,y,-


,g- .lY
.SS at e, -f ,na ef C r'f C.

F,ra r cle FLrrt .

!t
I
o Symptoms
o frmenorrheo (:V)
- Usually sudden (not a ture s/nPtuA, the abrupt cessation of menstruation
in a patient with regular cycles is highly suggestive especially when
delayed l0 days or more suggestive when a second menses is missed.
- About 20Yo of:women may report scant bloody discharge during early
pregnancy(implontotion & deciduol bleeding t+ cpt'r).
- But any bleeding during pregrancy is consldered abnormal.
.
. Breost symptoms a nipple tingling & breast tenderness
. Psychologlcol chonges. appetite changes, depression or insomnia
o Signs:
. Breost slons
o- Genltol slgns:
' Vulvo (soft, vtolet) -> Jacquemier sign
- Vogtno (soft, vlolet) ) Chadwick sign
-- Co:tx (sott, vlotet) -> Goodell sign
' Uterlne slgns
o Enlarged & soft
o Palme.rS-slgn:
@ On bimanual examination
@ The uterus shows painless alternate contractions & rela:rations
o H.egar's sig n-(sof-tening -oflhe.-istlrmus) :
@ lf 2 fingers are applied into the anterior fornix & the other hand
applied on the abdomen.
@ The fingerS meet in the area between the upper & lower segnrents
@ Done 6-L2wk
@ Before 6 weeks it is -ve due to firm isthmus
@ After 12wk) - ve as the lower segment is full of the fetus.

o DD: Causes of amenorrhea & causes of symmetrically enlarged uterus.


o Investisations:-
. Preononcu tests
A. Appears one week after the missed period
./ Aggludnation test: (500 mlU/ml)
./ Urine ELISA (50 mlU/ml)
Earty no
but it tFe
bladd

Mus
is {>
m

At terrn: th6 heed of tha


fetrrs <lescends into the
pehvis -) frequency
B.Serum tests ve I w before missed period i.e l'r day of implantation
the hormone at 5 mlU/dl
"zDetects
y Radioimmunoassay of p,HCG.
./ I.R:M.A.

y'Pregnancy, Ectopic, missed abortion, I.U.F.D & Trophoblastic tumors.

i.j. I
a
J
I 0 Done early in pregnancy or I
I
! + When diluted urine is used. t
!
i
a
! o Proteinuria, hematuria as hemoglobin is a protein & Pelvic TB !
I + Drugs stimulating LH release aJ penicillin and phenothiazines a
i
! + Immunologic diseases as SLE because IgM interacts with test reagents. !i
! + Perimenopausal women with high LH a
! + Excessively alkali
kaline urine |t-t
t o
! r ! ! r r r ! r ! t t r r r rl
UlS:,

G Pregnancy sac at (Sthw) c Cardiac activity at (7tbw)


q Fetal shadow at (6thw) g Sonicaid at (10h Week)

o Svmotoms
. Amenorhea, Breast symptoms, & abdominal enlargement.
. Quickenllg:" "First perception of fetal movement
.l In primigravida 18 -20 weeks
- + Inmultipara 16-18 weeks usll
o Breost E Skin sions.
a

!2 Utbrus is felt as soft peivi-abdominal ifiass ur,.JJs uaJ,.{.}.3 J:\g.


t :
,. Intermittent, painless uterine contractions
- Felt by abdominal pa ation
- It is irregular, I in frequency near term
I. Uterine souffie @@
- soft blowing nunnur.
- it due to gush of blood in uterine vessels
- synchronous with maternal pulse
Qesfoh'ooat f,ac frfot pulrnh:unn

(5 u) (7 ut)
o FetOl SignS l*..,1_l cpl_l r-iil
9 fnspection of fetol movements
_
I Polootion
-+
t
*Ballottement movement of the fetus in the amniotic fluid
o trnternal ballottement@ (between{6 . 28 weeks)
- The fingers of one handin the anterior fornrx irpwprd push the
. head )moves away then falls back touching the fingers.
- '
The otherhandis placed'ovei'the fundus'to keep it in place.
o External ballottement (> 24 wks)
- The 2 handsplaced abdominally on both sides of the uterus.

.3. f,'etal Heart Sound by


o Doppler (Doptone or sonicaid) at 10 weeks
+ Pinord stethoscope at 20 weeks.
* Umbilical (funic) souffle:
+ a sharp whrst/r'ngsound
+ it is due to gush of blood i4 the umbilical artery
+ or heard when a loop of cord is close to anterior uterine wail
o Synchronous with fetal heart sounds.

o InvestigationstPregnancy test & U/S (best, the safest, sure).

O D.D.
{. Causes of amenorthea & Causes of pelvic abdominal mass
*:
. Cause:
* Emotionol disturbonce as the intense desire for pregnancy, fear

"'lTH;ll',J"T';3ffi }il?.::,?i',H.T[i]?1i,T,.u,,
. Diasnosis:easv bv
- Pregnancy t€st:.-ve U/S empty uterus.
. Treatment:psychotherapy.
- }( Ihe.r:e-alelror.lte symptoms-hf. pregnancy,

X ButlAaf-si ure-signs of pr-egrancy--


l- Fetal movements .

2- Palpation of fetal parts .


3- Auscultation of F.H.S. or umbilical soufte.
X US & x-ray (fetal skeleton) are sure investigations .
o AIMS:
, qt i9 a. Drqgranl of preventive obstetrics 4i{i 4JS)
* Assess the maternal & fetal health status to give births a healthy infant.
+ To identify ond treot any medical, surgical and obstetric condition

* Estimotion of.the gestational age & expected date of delivery


.t Early detection of congenital fetal malformations & fetal therapy if possible

o Antenatal Care ingludeql


* History and exominotion
a lnvestigotions,
.. + Routine &screening tests
+ Other investigations according to findings
* Plon for a schedule for return visits
* Antenatal odvlce and reossuronce
* Preporotloa for delivery.

r Aim
G To detect any condition that leads to moternolor fetol hozords
i.e. to detect high risk pregnan cy (pregnancy astnciated with I
materna/ or feta/ /tazards due to certan r/* facto$:
r Risk factors:
l, Socioeconomic:
Socioeconomic status, parental occupation, psychological e.g. excess
anxiety leads to preterm.labor.
2. Demogrophlc fectors:
. Maternalage (optimal age is between 20-30 yrs), maternal education.
3. Madicol foctors
. Pre-eclampsia (PE), DM, Heart disease, Hyperemesis gravidarum,
Anemia, Renal, Hepatic & Thyroid disorderi, PVt: Respiratory disorders
. Personal history
G
. Ver//oung,liable to malnutrition, anemia, social disturbances, PE, small
pelvis ) difficult delivery (dystocia).
. 0/d: liable to hypertensive disorders, DM, APH, anemia, dystocia, CFMF
. Extremes of age lead t0 abzrt/'zn, IUCR, preterm /abor
QlVoritol stotus: if there is a period of infertility
C Poritu. hioher PNlvlR & MMfl in:
r trlderly I'G Z 35,vears f norvadays, liablu'lo
- Durirrg pregnancy:
or\hortion. pretcrm labor, hyperernesis -sravidarum. PE, antepartum
hemorrhage, chrom osornal irnonlal i es (Dorvn syndrome)
- During labor:
t Dystocia, prolongcd pregnrncy, rigid pcrineunt (needing
cpisiotoml,), highe r rate r-rf CS, gcncral anxiety irbout pregnancy
I Grandmultipara > 5 delivcrics: liable to
- During pregnenc!':
a anemia & other nutritional deliciencl', abortion, PTI.,
h-v"pertcnsive d isorders, APF!
- During labor :
i Pendulous atrtlonrcn & malprcscntation, osleonralaciac pelvis.
inertia lead to dvstocia. rupture uterus, PPH, operative deliverl,.

C frddress social conditions &environment


C Occupotion certain occupations have certain risk e.g infectious diseases,
anesthetic gases, radiation.
G Speciol hobits
. Smoking: increase risk of abortion, IUFD, IUGR, APH, perinatal death
. Alcohol: abortions, IUFD, IUGR, perinatal death, CFMF, mental handicap.

' Norcolics: fetal depression & addiction

Q Ploce:
' Pre vious ancnmplca ted hare deli vcna) reass ur in g
. /fprerbus conp/icatdlpne dcliverhs, better to deliver in hospital.
C Tine: for suceession ofdeliveries
' Raptdsuccession) liability to anemia &malnutrition
' Prolonged period of 2n'! infertility

' previous preterm labor ) suspacl materna/ disease, atcn'ne or cervaal factors
' previous postterm deliveries
C Weight of fetus
' Sna// for date /UGR) suspect maternal disease.
' MacrosomtT>> suspect DIvl
C flntenotol period
. Repeated hypertenstbn expects recurrence & superimposed PET
. Preytbus Dn/>> screen for DM
. Preyious APH or PROM>> may recur
Q ilode of delivery
. Vagna/(spontaneous onset, or induced ) search for a reason)
.CS
C- Tuoe of deliveru
. Eas/ vagina/ d€/iver/ expect another urtless other factors ocbut
. Breech may recut as in septate uterus
i. o If delivered vaginally easy is a good sign
o If complicated plan for possible CS
o Must deliver in hospital
. Forceps or ventose
o if easy & child well. . good
.

o if difficult or neonatal compiication ) hospitai delivery & suspect


cephalopelv ic disproportion
. .Cesarean section...high risk
c
G Neonote
. .Statut
o Alive & well or stlllblrth '''r. .r.dr.
o RDS, NND, ineubated or CFMF
. Wctgilif recorded
. Blwst or futtl€ lW

G inportent fot drting o[ ptegnaney (GA) &calculaliur Uf EDb


Q Must know if the nienses ore
. Avetage
' Regulaf
. lfsure ofdates ot not
r If pregnant on period of amenorrhea, or ditef COC

C Surgicol: previous operations


a UygS: may affect pregnaniy or fetus e.g oral anticoagulant
C Previous blood tronsfusion
C Presenca of ollergy to drugs
q .screen
C Hypertens i on.. investigate
q eFry,...screen
C Twins,..suspecr

. General
Y 3 looks:
' rthoPenia
' less than 150cm be aware of pelvic contraction
. f obese, be aware of DM, hypertension, macrosomia & dystocia.

lWoternol
Fetus Uterus flmniotic Qlocento 9reosts
Blood Extra
fluid
Fat
vascular
fluid
3500 3 500 1 500 1 500 1000 1000 500 s00
Total : 12.5 - 13 ke

Underweryr?t9 = But <2okglm2 Orerwerght? = eN{t > 26 kg/m2

3 Vitol doto

'normar"':;;::ff11i:ffi;::'""'"
: specially in infection or PROM
3 Colors:
t regnancy induced or hePatitis
o Cyonosiiin cyanotic heart diseases
o Polloranemia or bleeding during pregnancy
Y Suruey
. Heod:
- Elre brows for myxedema &leProsy
- Hair Ioss for. sYPhilis & SLE
- Cheeks: malar flush for mitral stenosis, butierfly erythema for SLE,
brown pigmentation for chloasma gravidarum
- Nose: saddle nose in sYPhilis
- Mouth: pallor jaundice ,cYanosis
t r thyroid, vessels, LNs
s7

Chest ond heort exominotionz


- Splitting 1st heart sound
- Appearance of 3rd heart sound
- Soft systolic murmur may be normal
- Diastolic oi loud murmurs ,...evaluate
Lciquer limbs for varicosities, DVT & edema,
G Edema may be normal if mild occurs during the end of the day and
' relived by iest.

Size: huge) twins or polyhydramnios


Shope: pendulous in P contracted pelvis
. S'trioe, Supropubic hair distribution: feminine or masculine.
Hernio, umbilicus, Pigmentotion: linea nigra

Fundol level (by hand or in cm above symphysis)


Symphysio - fundal height :

+ htcllonald nde: Weeks of pregnancy : SFH in cm X 8/7


+ Johnson rule : fetal rveight in grams : SFH - n X 155
(n= //
in engagd & /2 if not engaged)
+ Gravidografil progressive ] in SFH lcm/week after 20 weeks

Circumference around the umbilicus in inches: weeks of pregnancy

Causes of fundal level higher than period of amenorrhea:


1. Wrong dates or calculation.
2. Pregnant on period of irregular bleeding or previous threatened
abortion calculated as date of LMP
3. Multiple pregnancy. Macrosomia, Polyhydramnios
4. Concealed accidental hemorrhage.
5. Vesicular mole, choriscarcinoma
6. Turnors as f ibroids or ovarian mass rvith pregnancy

1. Wrbng calculation
2. Pregnant on period of amenorrhea.
3. Missed abortion, Intrauterine fetal death, lntrauterine growth
retardation
4. Oligohydraminos
5. Transverse lie
. Fundol grip:
-. Head....breech
- Breech....cephalic
- Empty ....transverse lie
. Umbilicol gripi
- For back & limbs
- In transverse lie ...feel head &breech
- For amount of liquor, expected fetal weight, local uterine pathology
. Pelvic orips:
- 1st pelvic grip: head (normal), breech, empty in transverse lie or
floating head (tiriengageh;.
- Znd pelvic grip: head engagement, position & presentation
(face...gcciput), flexed or extended.
Causes of non-engagement of head in PG
r Fetal Maternal
Large head, hydrocephalus Contracted pelvis
Malposition or rnalprescntation. Ttrnrors in pelvis
Multiple Pregnancy Placcnta previa
Short cord l-ull bladder or rectum
Polyhydramnios No causc may be found

. Sure sign of pregnancy qJtii: $Lr ful


a Fetal life
a Fetal distress
a Twins
a Position &presentation
a Progress in labor

Different sites of fetal heart rate (according to presentation)


Normal (OA) Between the ASIS & umbilicus
At the f lank, & highet than normal

NIA + umbilicus, NIP+ f lanks


Complete-;, above the umbilicus
FrankJ below the umbillcus

Transverse lie At one side of the umbilicus


@
Ahe firndus is palpated T:!le hands pdpaqg the ?: The head should bc
and irs cotrtents (here cohtoqrg ofthe utcrusr Dalpatcdi and ic should
the breech) identified. identiging tre back be noted whethcr it ic
elnd tic limbs. mobile oi 6xed inthc
bdm;
r Vaginal examination
Not always
. .ln eorltl preononcy for
iagnosis (Hegar's sign, ballot
n some complications as in ec
ny associated pathology e.g.
o take tervico-vaginal smear
. ln lote pregnoncy for
S.ro diagnose labor
Cto assess pelvis for CPD
C,A ny associated pathology

t
Q Elood tests:
- Blood group & Rh. "if Rh -ve
indicated", Hematocrit value or
. Serological tests: Rubella hemaggl
B surface antigen & tests of syp
l Urine ooolysis ond screen
- Bacteruria " mid stream urine" not
- Proteins
- Glucose

!
Etr Glucos e Screen, t,
tr CFMF, Maternal Serum AFP, Screen for chromosomal abnormality.
tr Infections: Gonorrhea culture, Chlamydia test, Tuberculin skin test, HIV
antibody & Group B streptococcus culture
tr us *f etal wellbeing
t-l l-t /

IJ M,E I 1T----;
?Freouencv:
--
Every 4 weeks until 28th weeks, then
.:.

Every 2 week until 36th week;


* Then weekly until delivery.
* More freguent visits in high risk group.
History:
.:. Eorly pregnancyl
* Pain, Bleeding, discharge (watery or infected), fever, dysuria, vomiting
.;t Lote pregnoncy:
* Bleeding, Contractions, cramping, pelvic pressure, ROM,
* Adequate fetal movement, symptoms of PE, or DM, any new problems.
* Desire of controcepfioh during delivery.
Examination in revislts
l. Generol: Weight, Blood pressure, edema
2. Abdominol: Fundal level, amount of liquor, presentatiort, position & FHS
3. Voginol exominofioh at 36 weeks "for contracted pelvis".

Routine:
@ Hb% is repeated at 34 week.
@ Urine for sugar and protein
Specific:
@ o fetoprotein (16 weeks) in suspected anomalies
@ Vaginal swab for bacterial vaginosis, GBS, Chlamydia

Graviditv: number of pregnancy.


* Paritv: number of delivery.
I)uration of pregnancy in weeks.
., Fetal Lie, Presentation, Positlott.
+ In labor or not
* Associated Complications (as DM OR APH)

N.B: How to test for syphtlis ?


l: VDRL, Wasserrrldn, Khan
is syphilitic due to false + ve in
a. Other treponema int'ection
b. chronic disease as malnriu, leprosy
c. Autoimmune disease as SLE, Rheurnatold urthtitis,
Antiphospholipid Ab $
d. rarely in nornral pregnancy
t. Confirmatory test :
a. Treponema pallidum imrnobilization test (usually not done)
b. Fluorescent Treponerna antibody test (ttsunlly done)
' How to test for Rubella ?
1- Tetrye IgG: recent infection
2- Titre IgG :
a. -ve: susceptible so avoid patients & vaccitrated after delivery not before
b. +ve: reoeat
- Rising titer ... infection - Same titer.. lmmunity
c. If inf-ected mother .

- In ISt I week ...5o/o risk - 8-12 week .....5Vo risk


- Above 12 weeks ...5Vo risk

* History:
+ strual labor interval
= 40 weeks, I0 lunar months, 9 calendar months + jdays or 280 days,
+ Date of Qttickening = 16 -LB wks in MP, 18 -20 wks in PG.
+
Naeclele s rule: E.D.D = LMp + 9 months + 7 days
'& The cycle is regular
* The patient sure of date
* The last cycle was of average duration & amount
* No contraception or lactation for at least 3 months before LMp
* It is applicable for 28 day cycles
{j Examinotion:
4 Fundal level =
./ 12 wks at symphysis pubis
/ 24 (20- 22) wks at umbilicus.
36 wks at Xiphisternum
+ S.F.H: McDonald formula: SFH in cm xBlT = GA in wks
Doppler at lowks
+ frbdominol girth ot umbilicus
q F.H.S: First heardby Pinord at 20 wks.
+ p/Vin early pregnancy, €ngogementin late pregnancy
* Investiqotions
I. U/S:
+ Croutn-Rump length in 1st L2 wks.
+ BPDfrom 12 wks, accurate till 24wks.
.) to estimate GA.
9. frnniocentesis: For fetal maturity [GA, Organ maturity]
3. X-Boy Inot usedL

a Corticosteroids(see PrL)
a Surfoctont theropu g after delivery (into the Endotracheal tube(ETT))

* Diet # L*, t-t+ lSlS:


Calories: about 2400 /300 kca/ / dal,:\)t\ : 30 kcal/k glday.
? Protein:
> 75-125 gm/d :1.5 glkg/d (add I kg to body weight)
D In form of meat or fish, eggs and I glass of milk daily
I Carbohydrate: 200-400 g t.i- , one gm of carbohydrate) 4 kcal.
t Lipid: Should be restricted to avoid excess weight.
I Mineral
> All basic mjneral needs (cxcept iron ':-rJt) are met by adequate
vegetable, fruit and milk intake.
) Iron must bc supplied as 30-60 mg/day + folic acid.
) If the patient can not'drink adequate milk, I to 2gm of ca is given.
) Salt: restricted only in hypertension
I Vitamins
> All vitamins needs are met by adequate vegetable andfruit intake. Many
physicians prescribe supplementary folic acid (0.5 to I mg daily).

Doily A ) s.000IU/d Nieotinic acid ) l.5mg/d

reguirements D ) 400ru/d Ascorbic acid (vit Q )


l00mg/d

81)lmg/d Folic acid ) 400ug/d

B2 ) l.Smg/d VitK ) PPH & fetal hg

* Smoking *Jt+ l+ i; C$r3Jt :


) Is restricted as nicotine, carbon monoxide and cyanide may lead to
obortion; preterm lobor and felol growth retordotion.
* frctivitU )Wl $ ty ct"l Llii,i'3 '
oids dangerous work or radiation or chentical.
* €xercises ciio 2e U4,+r)i :

drainage from LL, stimulate intestinal movements & help engagement


\7ANu =\aaz =srzznsizzasirz, sgz/=\Ig r\Ir:rzl \aE z, s\:/z sr;
=sg

', Air travel is better than train for long distances,


z If more than 6 hours, walking / 2 hours to avoid DVT
i Not allowed in late pregnancy or in repeated abortions 0r preterm labor.
# flothing r-*SJl tyL: p*l.eJl

i Comfortable clothes and avoid high heels which cause lordosis and tight
stocking should be avoided ->decrease venous return.

i No sea bathing ) ascending infection, Routine hygiene. Bath and showers


are allowed. Douching should b_e avoided or performed gently
i Hot tubs & sauna better to be avoided.
* egular cleaning and consult dentist if indicated.
* Breost l+ ae4-o :
> The nipples are washed by warm water and soap or boric acid.

x Sexual fictivifu lil Yl f y,g.e :


'a Allowed, some advice to be restricted very early or late in pregnancy,
hemorrhage, infections, ROM, abortions or Preterm labor
* Eowels:

fruits), regulation of bowels and exercises.


')Coffee/tea )
allowed but if in excess it causes irritability & J Fe absorption
* Worninc sumotoms *-ll
i Vaginal bleeding.

i Persistent headache, Blurring of vision, vomiting & Epigastric pain


fi-immediote core in the lobor word
' Keep under observation for at least one hour or more
' Transfer to postnatal ward after stabilization of blood pressure & stopping of
vaginal bleeding.
B-ln the postnotol word: Observation of
'r Pulse& temPeroture: every 4 hours for 24 hours then twice daily
':' Fundus ) palpated daily to check for normal involution
.r Lochio: checked daily to note the amount, color, odor
* Hb%rdone next day after delivery to exclude anemia
n The mentol stote or signs of depression (postpartum blues)
'r DVT:tenderness or swelling in the thigh or calf
+ The f etus:jaundice &condition of the umbilical stump
.r Dischorge if no complication after 24 - 48 hr
Glmmunizotion:
@ Durinq eroenooctt:
. Not controindicoted:

- Virus ) poliomyelitis (Salk), rabies, influenza


-. Bacteria ) meningiococci, plague, cholera, typhoid

produce antibodies):
- Tetanus
- Diphtheria

- Hepatitis A & B, measles, varicella, tetanus, rabies


Q Contraindicoted: onu live voccioe os MA/IR, uellow fgver
@ frfter lobor:
C Anti-D
' For non immunized Rh-ve mothers who delivered Rh+ve infant ) 300pg fM
' Or calculated according to Kleihauer Betke test
'Given within 72 hours from delivery
C Hepotitis B*va rnothers;
' The infant receives both active &passive immunization (Ig 0.5ml,IM)
C Rubella voccine:

' For non immune t (avoid pregnancy for 3 m as it is live attenuated vaccine)
D-lnstruction:
+Self care of the mother &her baby
*Advantages of breast feeding
+Contraception is discussed
*Coitus can be started within 3weeks
+Avoid contact with infected persons or carrier

l- History: vaginal bleeding 0r discharge, breast disorders, urinary or GIT


symptoms, backache usually due to strain of sacroiliac joint

2- Exominotion:
B Generol:to exclude hypertension & breast abnormalities
E Abdominol:for abdominal wall laxity, involution of uterus &CS
E Pelvic:
- Vulva &perineum: episiotomy, vaginal bleeding ol discharge, prolapse,
SUI.
- Vagina ...for vaginitis
- Cervix ...for erosions, cervicitis, lacerations
- Uterus ,.,for size, position, any abnormalities
- Adenxal for any swelling
3-Mongement of problems:

. Prolapse: pessary is used for 3 m till the supporting ligaments restore its tone
o Puerperal-retroversion: the uterus is corrected at first then a Hodge-Smith
pessary is left for 3 months

Cervical erosions (Ectopy): obseryed for 3 months as spontaneous cure occurs

Subinvolution of uterus

Postpartum psychosis (blues): transient depression in the lst wk postpartum


due to stress of labor, consult psychiatrist (self Iimited within days)

zas- ts- !s- :\7!s- !\7 s-- s--:s--!s


) Svmotoms
- Fetal kick chart = (Cardiff I ibjtr 4!
* Normolly: l0 or more k;cks l12hr
&lQNnOM/tl
s ABNORMAL ifif occurred
nnnurre/ in
in > 1?
12 hh.. -^ v ,vr uoser !,,rrv
- * Advontoges:
I Easy
? Cheap
? Good indication of fetal rvellbeing.

? Subjective (some t can t count or can't feel the kicks)


I Not sufficient alone in high risk cases.
? It is not accurate as the about 80% of the movements only detected by US
are perceived by the mother a.#
* lf : Fetal condition can be assessed by:
l. Cardiotocography (CTG) or
2, Bioph I profile.
t2
houru

D-ft

{;ta.t ff<K e kar P

I
F Signs
* fu,lo terno I weight goin n2nd half of pregnancy
o Normally:lDk{wkafter 20 wks
o In the absence of edem4 overeating twins or polyhydramnios
* lncreose in the fundol level(Symphysio-funddheight)

rr Gravidosram:
:-
a At20 wk = 20 cm & f I cm / wk ( after 20 weeks)

I McDonald'S formula
+ GA (in weeks) = Slmphysio tundal height (in cm) x$/7
r lohnson's formula
+ Fetal weight (in gams) = SFH - n x 155
(N= 12 cnr if unengoged heod or ll cm if engoged heqd )

* frbdoninol girth 614l t'r' ^ '


oCircumference at the umbilicus 36 inches at 36 w & 40 inches at 40 w

) Biochemical tests (obsolete)


.:.Human Placental Lactogen level & Estriol.

) Biophysical tests l. *l +=.

* ldeo:
r It is soundwaves between l&30 mega[fu (beyend thenumal hearing =20 &20.000)
9 The equipment send waves & receives dre echo by tansducer pfr on the abdomen
tlrough gel (waves can't pass air)
9 The echoes depends on dilferent tissue densities (water )black, bone )white)
e It is done by Sir Ian Donald (1958).
* Hozords:
! Safe for the mother & fetus in contast with X ray (causes CFMF or IUED, malignanry
later on & matemal gonadal affection)
* Technique z

? Two approaches
. IreEEeEdgaqlE3! (3-3.s MHz)
- Wde image but les sharp
- Needs firll bladder (to elevate theuterus to the abdomen)
.I@!(s-8MHz)
ore sharp but limited scanned area.
Th height of
the ftndw in
ccutimetres
uhould ef,ual
approximately
the weekt of
gclteuon,
* Uses in obstetrlcs
r Diagnostic:
. Eghrs: fetal life, site (ectopic), number (trvins), CFMF, biomefy
, Amnioticfluid: volume, turbidity (for lung maturity )
. Placenta: position, tumors, hemorrhage, grading 0,12,3 (maturity)
. Uterus: uterine anomalies, fibroids, remnants after delivery or abortion.
' Cervix diameter (forpatulous internal os )
? Therapeutic:
. Chorionic villus sampling (CVS)
. Amniocentesis,cordocentesis,needleguidedbiopsy
. Fetal therapy.
* Uses in Gynecology:
r Diasnostic:
. Congenital lesions (hypoplastic uterug agenesis, septate & bicornuate).
. Trauma:(perforatedruCD)
. Inflammatorln Clubo ovarian cyst, Chronic PID).
. Tumors: Benigu & malignant
. Insertion of contraceptive (assessment of site of IUCD)
. r Therapeutic:
. Ovum rekieval in ART
. U/S guided aspiration of cysts e.g.: abscess.

* ldeo:
t Dopplu is used to measure blood flow & velocity in different vessels
? The echoes reflected from a moving object is directly proportional to their velocity, thus
U/S reflects echoes from moving RBCs in differentbloodvessels
t It is the earliest diagnosis of placental inzufficiency
? I;n cases of placurtal insufficienry the blood flow through the placartal bed instead of
occurring in both systole and diastole occurs in systole only
o First decreased diastolic flow
. Then stoppage
o And at last reversal of flow, ocflrs in preeclampsia (this is a very grave sign)
. Done by Andreas (Johann S rl) Christian Doppler (l8r'.z)
* Tuoes of Doooler
. Sonicaid (Doptone): if flow is changed into sound) hearFHS
I Doppler: shows Systolic & Diastolicwaves.
. Colored Doppler: differentiates aftery from vein, systole from diastole
1 Duplex adds cross sectional diameter (esp in DVI) + US
z=\r\zzr\rg=\rvzlsrTza\Igz=sr7lz=s\7-l\v'-vzz\\7<

Rueool of l/""J fi-


( J'otl)
f="
Vza.]SZZ\\7z1]V,-a-Zz-]Vz-\7z-]V4\-Z.;:\:EZ a \:-r:-,z a \zz a S-SZ- a
=r,

* Uses in obstetric:
I Assess feto-plocentol circulotion in cases ofplacental insufficiency
' PET & ruGR (uterine, umbilical & cerebral vessels).
. Early detection before pathology.
r Diogrnosis of fetol cordioc obnormolities
r Diognosis of fetol molformotion.
r Diognosis of fetol onemiq: RH incompatibility (increase velocity)
: suggest the nature of suspicious swelling (benig, or malignant )

A. Determine btood ftow volume :Velocity X c,ross section al areaof the blood vessel
B. Flow wove form onolysis (all parameters I as gesationll age progresses): -
o Systolk,/ diostolic rotio (SlD ratio): - from uterine or fetal umbilical artery
o Resistonce inde:<: S -D / S
o Pulsotility inda<: S -D /mean

* Aeo n-il!.+i: ejY;


r FHR is controlled by the autonomic nervous system, the sympathetic is
the accelerator & the parasympathetic nervous system is the decelerator.
I , Vagal dominance occurs as the fetus getting mature & approaches term
i.e. the 4verage FHR is 200 at20 w & is 120-140 bpm at term pregnancy.
e CTG is graphic recording of fetal heart rate & uterine activity
r Monitoring is either external or internal (only during labor)

1st method: Score of 10


I.
. Tachycardia (>160) may occur in pyrexia, rnild stress and drugs
o Bradycardia (<120) in drugs, congenital heart block & severe hypoxia
2. Be o t- to-be o t vor i o b i I i ty
' /f is normol as fetal heart receives impulses from both sympathetic &
parasympathetic systems.
' Absent in distress, drugs, tachycardia, severe prematurity or sleep
. It hos 2 components:
o Amplitude (short term variability): it is the beat to beat
variability between electrical cycles. Normal range is 5-10 bpm.
o Frequency (long term): it has a frequency of 3-10 cycles/min,
amplitude of 10-25bpffi, & is reflected in the waviness of FHR.
!. flccelerotloos
I They are elevated to 15-25 bpm above the base line for 15 seconds
; Due.to fetal movement or contractions ) sympathetic stimulation
I Absent in hypoxia, drugs & sleep. :

4. Decelerotlons
r 15 beats or more below the base line
I Normally absent, if present = hypoxia.
. t Normally, the placental blood flow allows sufficient oxygenation of
the fetal blood to maintain metabolism during contractions.
ll If'blood flow is led with no 02.reserve -; anerobic metabolism g
lactic acidosis ) CNS & cardiac depression ) decelerations which
corrects on the resumption of placental flow after uterine relaxation.

Tuoes of decelerotions
) Early deceleration (Type 1 dips): Starts with the onset of the contraction
& ends with its end. It is due to vogol stimulation by fetal head
compression during contraction
F
Late deceleration (Type 2 dips): Starts after start of the contraction (near
peak), the nadir 1gtil corresponds to'the end bf the contraction & the recovery
occurs after the end of the contraction. It is o result of fetol h)rpoxio
F Variable decelerations: they are variable in duration, intensity & timing
but often coincide with contractions. They ore due to umbilicol cord
g-olT!.p-ressto,!. It indicates fetal distress if it is persistent.
F Prolonged deceleration: isolated deceleiation of > 2 min
* Interpretation
I Normal: 7-10 repeat in I week
r 6 terminate if mature & if im ture repeat daily
e < 6 terminate
Score 0 1 2
Baseline FHR <100 o>180 100-120 or 160-180 120-160 b/min

Amplitude <5 5-10 o>25 10-25 b/min

Frequency <2 2-4 >4 cycle/min

Deceleration Late>25 Vo Late <257o Sinsle mild variable


cleceleratron

Acceleratlon No reactive Atypical Reactive


* Reoctive (-ve):2 accelerations of at least l5 b/m in 20 min
* Non-reoctive (+ve): less thaa2 accelerations in 40 min.
. Results of CTG:
- Folse -ve: (5/1000))IUFD within I wk though the test is reactive
- Folse +ve: (50%) as in sleep do BPP, Doppler, amniotic fluid volume.
o Advaniases of CTG:
Easy tb perform
It is reliable, no risk & has no contraindication

A device generating 110 decibels at 10.000 Hz (artificial larynx)


Fetal stimulation with artificial larynx ) If occeterotions occurs )
reoctive

? Oxytocin is given to induce contractions, 3-5/10 min each lasting 40-60s.


tt This will cause hypoxic stresses to the fetus through ] of uterine blood flow.
I If fetal oxygen is normal, will not disturb fetal oxygenation.
these changes
i But, if fetal oxygenation is low, contractions will induce deceleration-
t CST tests the fetal oxygen reserve prior to development of fetal compromise.
!l Results:
. Neqotive: contractions adequate + no decelerations ) repeat weekly.
False -ve: 0.5/1000
Positive:contractionsadequatetlatedecelerationsin>
contractions. False *ve: 30% ) repeat +BPP & Doppler & AFI
a Suspicious: contr. adequate + late decelerations in < 50% of contractions
a Hyperstimulotion: contractions > 5/10 min or last more than 90 sec.
a Unsotisf octory: no adequate conkactions.
. Disodvontoges: Mdy induce dangerous fetal hypoxia.
t Contraindications: uterine scars, placenta previa, preterm labor.

NipPle stimulotion! Manual stimulation of the nipple for secretion.of


endogenous oxytocin. However this test is not-acceptable-socially

tl Fetal viability
!Z Nervous system is well developed.
a
Unusuol FHR potterns: either sinusoidol or fetol orrhythmios
fi"nitli c {tr;C f ocQr'
I €och receives o score of 2 or zero

Variables Normal (score 2) Score O

Fetal breathing 1 sustained breathing for 30 sec in 30 min Less

Felal movements 3 or more moyements in 30 minutes. <3


Fetsl tone I movement of texion after extension No

Fetal reactivity 2 or more acclerations in 20 minutes <2


Amnioticfluid A pocket ofat least I cm X 1 cm <lcmX1cm

I Besults:
r $ - 10 & amniotic fluid unaffected: Normal ) repeat in I week
. 6 & normal amniotic fluid: Deliver if > 36wks or repeat daily if not
. 4 or less or amniotic fluid affected: terminate
I Pitfolls:
. The BPP interpretation is dependant on the gestational age
. NST, amniotic fluid & breathing is more predictive than movement & tone
- Modified Manning omit (l) & the scoring is out of 8 & the results as above
- Another modification : do AFI (chronic distress) + NST (acute distress)

I Significonce of fetol testing: There is a controversy between those who


feel there is a clear benefit, others feel that there is only marginal advantage & by
the time the test is abnormal fetal damage has already occurred
| ffFl (omniotic fluid index): is the sum of the longest diameters of the 4
quadrants of the uterus (most irnportant in Postterm). N: l0-15
I High risk oregnoncv:
o Doppler for early detection
o CTG
+ 7ll0 ) repeated weekly
+6:
. Mature: TOP
. Immature:
- Repeat daily & give corticosteroids
BPP:
E if good (> 6): repeated weekly (especially in adequate AF)
El 6: mature fetus) terminate, immature) repeat daily
E .6r terminate
+ <6 ) Termination of pregnancy
I There ore high folse +ve results f CS without improving PNMR
t Fetol heort rote monitoring:
Graphic recording of fetal cardiac &uterine activity
A.
C Every 30 min in the I't stage and every 5 min in the 2nd stage.
C tt is heard immediately after contractions for 30 seconds.
C lt is not so accurate.
B. Electronic monif oring:
ri. Exte.rnalmonitoring: of both uterine contractions & FHS
(tocodynamometer) (fixed to the anterior abdominal
wall by a belt) on a tracing paper
Internal Monitoring:

)( Data are transmitted from


i. intrauterine transducer (for uterine contraction)
ii. an electrode fixed to the scalp for FHR
>( Requirements: CX dilatation, ROM, cephalic presentation
>( Advantages :more accurate > external (esp. for variability)
>( Disadvantages: May cause infection& trauma but more reliable.

+ Results :
A Abnormal FHR: No accelerations, Loss of variability, late
decelerations, Bradycardia
i Causes: hypoxia, sympatholytic drugs, congenital heart block,
sleep
rr i - - -r .i - rr- - -.t -- iil ;,ia- -- - -I.r - -
- - - - - -
* CCC of normal uterine contraction:
@ Start infreqyen! _shgrt l!y9d, ryeak (amplitude 19120 mmHg), then
@ Gradually increase in frequency, duration, intensity till reaching 3-
5/10 min each lasting 40-60 sec (amplitude 40-60 mmHg )

@ Two waves are recorded :

{ Asynchronic (o waves):irregular, low amplitude, high frequency


* Synchronic (B waves): regular, high amplitude, low freq'uency
---a-- --r i-- rr---- --- r I-
---
I
U rndicotions
i. Abnormal FHR patterns,
ii. Meconium stained liquor.
@ rtiethod
o Cervix is dilated enough 3-4 cm + ROM + well engaged head
o Fetal scalp is cleaned from blood
o Then sprayed with alcohol ) hyperemia
. Then incise 3-5 mm thr<iugh the amnioscope.
o Collect blood in a preheparinized capillary tube
o Maternal PH should be measured as it affects the fetal PH
o Results
\ 27.25 ) Nbrmal.
A <7.20 ) Acidosis.
\ 7.20-7.25 ) Repeat.
I Fetol pulse oxymetry:

I Scolp stlmulotion test:


_ If pressure on the fetal scalp ) acceleration : rio fetal acidosjs
?

F Indicator for fetal distress which may lead to CNS depression )


I Inte'stinal motility &, relaxation of sphincters.
) It may be normal in breech & post date.

l. Motemol Position chonge to remove cord compression (left lateral).


2. Stop orytocin + Orygen administration (to correct fetal hypoxia) +
intravenous hydrolion (to correct maternal hypotension) .
2.lf fetal distress persists, immediate delivery by C5 or low forceps(full
cervical dilatation + engaged head ).
4. Proper neonatal rcsuscitos-tion .
t Definition:Tapping of A.F for diagnostic or therapeutic pu{poses.
t lndicotions:
* Diognostic:
+ Early- i-n- P reEnancyi Geneti c amni ocentesis : I I 5- 1 Twks]
D Cells cultured for 3wks & subiected to :
' Genetic studiesto diagnose chromosomal defects [e.g. Turner]
& sex linked diseases
. Enzymatt'cstadtesto diagnose errors of metabolism [Gucher]
' KarloUpngor gene probing to identify fetal sex.

F Genetic amniocentesis is done if :


' Moternol oge > 35yrs gl1|> 3 abortions.
, Parents or previoul child with chromosomal abnormality.
) Early diagnosis of CFMF:
r By measuring alpha fetoprotein in A.F for ONTD.
+ Late-durjng- p,rc.gnancy
1) Estimotion of bilirubin in AF in cases of Rh iso-immunization

2) Estimotion of fetol moturity: Fetal epithelial cells are stained


with Nile blue sulphate, cells containing fat are stained Orange & those
' without fat granules are blue.
3) Estimotion of fetol orgon moturity: (Vide Infra)
4)Amniogrophy: to diagnose GIT anomalies& delineate GIT for IUPT
5) Plocento! functions: estimation of eshiol level * meconium stained AF.

6)C&S In cases of suspected IU infection. '

o Repeated tapnine of AF in chronic polvhydramnios.


g Induction of abortion by intra-amniotic injury of hvpertonic saline. urea.
O Intrauterine transfusion in Rh iso-immunization
' + tr'etal theranv: intfa-amniotic glucose for IUGR or PRL & T4 | lung
maturity.
r Technique:
* Stbrilization & asepsis.
* U/S for placental localization & identification
of amniotic fluid pool.
* AF aspirated by 20 -22 gauge needles inserted under U/S euidance either
transabdominal (better) or transvaginal,
* After procedure ) fetal monitoring + Anti D for Rh -ve mother.
Uilrasouhd trtsrrcdueer

PtBEf nta
Arnniotia lluH Amfiiofic {luid
vdlhdiawn
Uloru6
Publc bong
! Complications:
* Moternol:
o Iniury to Bladder, Bowel, Blood vessel.
o Sensitization of Rh -ve mother.
+ Fetol:
+ ROM & Infection.
o Iniurv to placenta )feto-maternal Hge.
o Iiliury to umbilical cord )fetal tamponade
o Iniurv to fetal vital organ ) Lso not done < l4wksl

* It is the main protein in the fetus


*Produced mainly in the volk sec. the liver & small amounts by GIT &.-plgggnt&
{.It is present in fetal serum. $
(excreted by fetal kidney) & maternrl serum

!Normal levels of -FP


* In fetal blood it reaches a peak between 13-15'b week (3mg/ml)
* After which the level decreases as pregnancy pro$esses.
* In AF level is exoressed in micrograms follows serum level and is 150 times
lower in AF than fetal serum.
*In maternal serum the level is expressed in nq, it rises gradually starting from
the seventh wk of gestation, its neak is at about 32wks then declines till term.
.l It has avery wide normal range so it is expressed in multiples of the medion.

lAIpha fetoprotetn
1) Wrong'calculation of date.
2) Neural tube defects (NTD), sacrococcygeal teratoma, Cystic hygroma.
3) Congenital renal anomalies,
4) Urinary obstruction.
5) Esophageal & duodenal dhesia.
6) Hepatic anomalies
7) Presence of fetal blood in AF [50 times higher in fetal serum than AF]
8) Multiple pregnancy or Ectopic pregnancy
!Atpha fetoprotein decreases (< 0.2 MOM) in:
* Missed abortion, IIJFD, vesicular mole
*Down syndrome part of the triple test (JcFP,E3 & IUCC) done at 16 weeks
routinely if maternal age > 35 years. The sensitivity t to 80% by adding nuchal
thickness (normally 1.5 mm).
t Interpretation & screenins by Alpha fetonrotein:
1- Start by measuring maternal serum AFP.
d If
elevated 2 MOM then proceed to step 2, while if it is 4 MOM = NTD.
2- Careful U/S is performed for: GA, CFMF, Ectopic pregnancy
3' If U/S reveals no abnormality
\ AFAFP is estimated, if found to be 5 MOM ) high chance of CFMF
{ So,high resolution U/S is performed again
8 Together with other biochemical tests e.g. Acetyl cholinesterose in
omniotic fluid.

1- 'bble stobility test:


+ AII sample serially diluted with saline + I ml ethanol95% in a ratio l:l
then l:2 and so on till l:5.
a Shake tubes for 15 sec. and then allow resting for 15 min.
+ If ring of bubbles appear betweeu AF'& ethanol in 2"d tube or more =
mature lung.
2- €stimation ofl / S rotio :- If> 2 : Mature lung
3- Detection of ft in ffF ; - Especially in D.M
4- = lung profile:
oLecithin, Phosphatidyl glycerol, Phosphatidyl inositol, Phosphatidyl
ethanolamine, & Phosphatidyl serine.
+ Sphingomyelin.
5- Becentlu---
+ AF micro viscositr:
1 AF is mixed with fluorescent dye that attaches to hydrocarbon of the
surfactant, then intensity of fluorescence is measured
+ SPectroPhotometly of 65O nm [absorbance of surfactant]
.) rfoctont / olbumin rotio [50 or more = mature lung-100%]

* Creatinine in amniotic fluid at term > 2mgYo.

* Amniotic fluid bilirubin:0 at 36wks


* Congenital "s ince birth" Malformation'faulty development'@
t lncidence: -
':'i{ojor molformotion ) 2 -s % t+ rr PJI
+iiinor molformotion ) l0%

-#

tructurol

t €TIOLOGV:
enetic (7.5%) & chromosomot (6%):
, o Autosomal recessive as polycystic kidneys.
. o Chromosomal anomalies (Trisomy 13, 18,21, monosomy).
o X linked as muscle dystrophy.
)Exog.nrurt
o Drugs: e.g. OCPs, thalidomide
o Infection: e.g. rubella, C.M.V. ,Irradiation & endocrinal (DM)
o Fetal hypoxia & mechanical (oligohydramnios ) talipes equino varus)
)multi-foctoriot
hrJiopothic (60%)
t Dioonosis
':'SymPtoms:
o Personal llistorv: age ( > 35 years), drugs
o *ve F'amily history
o Infection during pregnancy
* Slgns:
+ Oligohydramnios ) renal agenesis (Potter syndrome)
+ Polyhydramnios, unexplained IUGR
':' Investigotion: -

f-i
* Maternal serum a FP: f in ONTD & J in Dowo syndrome
* Double test: I a FP, IHCG
'* Triple test I a FP, I E 3, 1HCG
* Quadruple test: t o FP, I E 3, tHCG, f inhibin
2:Imggittg;.
G II/S,: it is done at (18-20 weeks) abnormalities of head, limbs & liquor
* Levet fI U5: for anomaly scan
+ 3D U5: for more details of CFMI
+ 4D U5: real time (live scan)
* Nuchol fronslucency 11-13 weeks (normally < 1.5 mm I in Down)
* Sensitivity 80% (triple test + nuchal translucency) in detection of Down
(, MRI. CT -y expensive
)

--,

a Done in IVF - ET
A Cells are taken from the outer cell mass in
A Only t-2 cells {blastomeres} at 6 or 8 or I0 cell stage.
e A hole in the zona pellucida of the morula to aspirate the cells is done by
LASER or chemicals.
A Cells aspirated are sent for karyotyping, gene probing, inborn errors of
metabolism to transfer only normal embryos aot-! u+ pl.Jt
?-Chorion i c vi llous sompl ing
* The aim is to obtain chorionic cells ( fetal in origin ) for laboratory study
e Done at8-12 wks either:
* Trans-abdominal or
* Trans-vaginal (guided by U/S).
r Aspirated cells are sent for
o Karyotyping & gene probing
o Determination of fetal sex
' o Enzymatic assay for inborn errors of metabolism, abnormal Hb.
A:
'Fetal cells are.obtained at an earlier gestational age.
'Chorionic cells divide very rapidly ) no need for lengthy cultures.
. Possible complicotions of (CVS):
. Fetal death (5%), intrauterine infections
' Feto-maternal hemorrhage & limb reduction l+ l+ l+ L#
3-frmniocentesis
4- Cordocentesis:
' The aim is to obtain fetal blood
' It is done by using ultrasound directed needling of an umbilical vessel at the
cbrd root (placental insertion).
. Possible indicotions of cordocentesis:
- Diagnostic:
) Fetal karyotyping.
) Other blood tests: Coagulation factors, hemoglobin level
) Cases with iso-immunization: Fetal blood type and Rh status,
Coombs antibody testing.
r Therapeutic:
) Transfusion of compatible douor blood
) Injection of drugs directly ) fetal therapy,
S-€mbryoscoptli an endoscope passed through the cervix under US guide
G Aim: to visualize the embryo to detect facial 6r limb anomalies
6-fetoscopyl this tube passed through ant. Abdominal wall under local
anesthesia under U/S guide to inspect the fetus.
C Aim: to take a sample of cord blood & to take biopsy from fetal skin & liver

d Chronosomol study: as in Down syndrome

d Gene'study: Its value in obstetrics is related to the study of inherited disease

d Serology infections fetol sexin sex linked diseases

* Prophylactic:
o Avoid predisposing factors
\ Avoid exposure to teratogens
\ Control of blood glucose
8 Fetal therapy "see later"
. Screening program "neonatal & adult"
I Oenettc counse/in_a about previous malformations & recurreoces
I Oenettb sreening. as congenital hypothyroiilism

+ Terminotion of pregnancy if major malformation (CNS, major CVS)

* Delivery in well eguipped ploces for immediate post partum, treatment is as

GIT obstruction, congenital diaphragmatic hernia

* Posf notol ossessment: by routine examination of the treonate


ultrasound
ffansducer
VA\7;:-\\zz::vz=:]vz-vzAIE-zavzl-:z'41rEz l-- : S-Sr- I
= -zz = -,rz

O Fetol Theranv ?S.


I Definition l.+ x-.:
+ The administration of a drug with the lry purpose is preventing or treating a fetal
disorder.
.l Meosures used to reduce tho incidence of CFfuIF
* Peri-concepfionol folic ocid to prevent ONTD till 12 weeks of gestation.
Patients on anticonvulsant should receive higher doses .

) Isotretinoin (mega dose), androgens, lithium


) Oral hypoglycemic & oral anticoagulants
* Controt of diseoses as diabetes mellitus & low phenylalanine diet in PKU

a Drugs used for fetol theropy:


*Digitolis (0.25-0.75 mg orally daily) used for fetal supraventricular tachycardia
n Indomethocin to treat Polyhydramnios.
* Vitomin K to prevent or reduce ICH,
* Antibiotics if +ve vaginal swabs (J I.[N sepsis)
+ Antenotolsteroids to prevent RDS
'3' Low dose ospirin & heporin to treat IUGR
':' Thyroxin for congenital hypothyroidism.
* 6ene theroPy: still experimental

t Closed fetol surgery


fntrouterine blood tronsfusion in cases of RH iso-immunization
.:.
Pi;.otoil cotheter is introduced in the bladder in posterior urethral valve.
':' Shunt procedures is used to treat hydrocephalus
{' In trvin to twin transfirsion syndrome, is
done DJ-[4!$E,B

a Ooen fetol suroer


l't open fetal surgery (1982) was for obstructive surgery the most importairt
obstacle to the success offetal surgery is the preterm labor & amniotic fluid leak
al e tioloou:
j Folic acid deficiency
I feotures:
1 More common in girls
.;. Frog face deformity
.:. Adrenal & pituitary hypoplasia
.;. Associated GIT anomalies (mainly esophageal Artesia)
10 €ffects..PHA, Shoulder dystocia, Preterm, Postjterm.& Mal-presentations
Q lnvestigotions:
t U/S, f a-Fetoprotein (maternal seram & anniott'c f/ato), tcholinesterase enzyme
A Preve n ti on :Pre-conceptional folic acid 5mg/d
A TTT:
+ Elective abortion
.r Folic acid to avoid recurrence

to arcumu/atbn of t/te CSFrh fie yentnT/es & sub-arachnordspace


I Feta/headdue
A €tiology
* (Arnold Chiari $)
*Coneenital infection (CMV / Toxoplasmosis)
;Chromosomal abnormalities (hisomy 18, triploidy, X-Linked trait)
.:. Multi factorial oriein

i Diognosis:
.:.U/S 'cerebral cortex compression, PHA', cortical thickness.

10 Complicotion
1.. Malpresentation & obstructed labor
I TTT:
4 Early & severe: )Abortion
+ Late:
+ ) destructive
operation.
+ If there are no other CFMF* corticsl thickness > I cm )CS * shunt
operation
I Definition:
'i' Defect in the spine due to faihre of fusion of the 2 halves of the vertebral arch
I Types:
1. Soina bifida occulta
2. Spina bifida cystica:
o Menineiocele
o Menineiomvelocele
Q Mvelocele
/ Associated anomalies Arnold Chiari (+ hydrocephalus later) &
mosomal defect (trisomy l8)
I Treotm
'i' Cover the lesion with sterile non adhesive dressing + Consult netuostugeon

1:*!isr.sg.g.p..Q.g!x
4 De fi n i ti onrabnormally small head

4 Diognosis.,decrease occipito-frontal diameter OFD & BPD


4 Co mp i co ti ons: mental retardation
I

!:.P:yg!.9.P.*.#.?.!!.
J frbove lleum
o Causes: Esophageal & duodenal afresiq Pyloric stenosis, Jejunal/ ilealAtresia
+ Clinical picture: Vomiting & abdominal Distension after labor PHA
I Belota lleum
+ Imperforate anus -Anal Atresia -Hirschsprung's disease
+ Geneialized distension of bowel loops on U/S
ITTT: - Surgicol repoir
2- Cleft lip & cleft palaE a.rt)+,,1 Jalt

I. Renal agenesis: potier's $ ) oligohydramnios, IUGR, facial abnormalities


2. F pispedivs, hyposoedius
?. Ambieuous senitalia: in chromosomal abnormalities, congenital adrenal
hyperpl asia, exogenous androgen
4. Obstructive uropathy: PUJ obshuction, uretbral valve, urethral atresia )
oligohydramnios * dilatation of urinary tract
+ TTT inutero vesicoamniotic shunt then surgical correction after labor
5. Undescended testes: orchipexy at ly
t
oThe defect in closure may involve the lower part of abdominal wall only, or
bladder, urethra and penis, and/or clitoris
t MSAFP, high % of associated cardiac & other anomalies.

: Ompholocele (exompholos): Congenital herniation of some of the


intraabdominal contents through the umbilical ring.
. Ectopio vesico: The defect involves the bladder.
+
. lmmediote core;
' Do not clamp protruding mass, Clamp the umbilical cord few centimeters
distal to the swelling.
. Keep the hernial sac moist and warm, avoid trauma & infection.
. Empty the stomach of air with a nasogastric tube.
. Surgical corrective repair
? icol
? VSD, ASD, PS, Fallot tetralogy

s Definition
-# Generalized skin edema (> 5 mm by US)
+ Fluid accumulation in ) 2 serous cavities
+ Placental thickening > 4 cm
o Couses of fetol hydrops
* Immune hydroPs fetolis. Due to chronic intrauterine anemia.
+ The well-known example is Rh iso-immunization.
olisl It has a high incidence of mortality.
+ f,'etal cardiac arrhvthmias e.g. supraventricular tachycardia.
4 Fetal structural cardiac anomalies e.g: hypoplastic left heart,
+,i
+ Intrauterin e infection s
/ Due to chronic intrauterine anemia e.g. parvovirus infBction.
{ Liver affection: toxoplasmosis, rubella, CMV, congenital hepatitis.
/ Cardiac affection as coxsachie virus
+ Chromosomal abnormalities e.g. Turner's syndrome, Trisomy l8 or 21.
+ e.g. a-thalassemia (notp). "
+ Twin-to-twin transfu sion.
IT III II' III II' III III
..
-Ii

trrr rrr r-r rrr rrr rtl


-rr --J
I - Gostroi ntasti nol disturbonca

!. rl{orning sickness
!. Gingivitis: hyperemic gums that may bleed with the use of a tooth brush.
!. Pilolism: excessive salivation. It manifests especially with smoking.
!. Heort burn:
lDue to relaxation of the cardiac +/- pyloric sphincter (pnssare effectl
lTreated by antacids, more frequent small meals, avoidance of spices.
! fndigestion. Flotulence:
d hypochlorohydria (regurgitation of dlkaline chyle into stomach)
I Constipotion:
! Due to progesterone effect & pressure by gravid uterus

9 Treated by | fluid intake, eating whole meal bread (not white bread)
! Henorrhoids: usually regress after delivery but not completely

2- Uri nory disturbonces


g >E .l r-l-rr g.ll:
r Early weeks of pregnancy (the uterus is still a pelvic organ)
I last weeks (due to engagement)
O Stress incontinence (50%): due to loss of posterior urethrovesical angle.
3- lrlu scu I o-sk e I e to I d i s tu rbo n ce :
$ Bockoche:
O common in the last trimester due to
* Increased strain on the lower lumbar spine
+ Relaxation of the pelvic ligaments &muscles).
OTreatment:
* Avoid wearing high heeled shoes
* Exercises to strengthen the back muscles
!. Leg cromps
d Due to:

' Sudden involuntary pumping of blood out of distended venous


sinuses into the calf muscles f "ill9lj5;*i:.
' The stress on leg muscles of carrying the extra rveight of pregnancy
' The pressure of the growing baby on the nerves and blood vessels
d Prophylaxis :
. Raise the foot of the bed about 25 cm above horizontal.
. Stretch & exercise
. Avoid standing or sitting in one position for long periods of time.
. Drink plenty of fluids.
. Massage your legs and apply heat.

s Round ligoment poin:


o Sharp groin pains due to spasm of the round llgament associated with
sudden movements (especially on the Left side because of dexhoversion).
o Prevention: avoid sudden movement.
o Treatment: local heat, analgesia.
4- Skin chonges :
s Strioe grovidorum
s Sweoting & feeling the heot (due to I peripheral circulation & VD)
5- S-Neruous system
$ Insomnio due to the large uterus, leg cramps & backache
g CorEol tunnel syndrome: due to edema, disappears 2 weeks after delivery,
s Plociditl (calmness) & drowsiness: due to increase progesterone
6- Cord i ovoscu lor sV mptoms

s Voricose veins
* Treated by:
a Patients should sit with their feet elevated whenever possible.
t
Nylon elastic stocking should be put in on the morning before
- getting out of bed & removed on sleeping.
t Complications: DVT which is rare & treated by heparin.
!
* It is due to the altered cardiovascular dynamics & the introduction of the
placental bed into the vascular system.
s
t' Due to I estrogen ) water storage in the ground substance of connective
tissue & mechanical obstruction to the venous return from LL

7- Voginol dischorge
s Leucornheo (due to increase estrogen) treatment frequent wash oithe vulva.
D.D. ROM.
Early pregnancy Antepartum hemorrhage Post partum
hemorrhage
Mancaas athen P/acena/ futn-placenhl
ILocal I Placenta 9tr'.trl: r.st o Atonic
O Hartman's g
orevia orevia Traumatic
sisn o hbruptio @huoture uterus O Retained
o olcidual placenta @Excessive olacenta
hemonhage show o brc
@Marginal sinus O Acute inversion
hemonhase
@Local r.rl.,

t{ Definition LsF*s i
? Termination (or interuption) of pregnancy
0r
? The attempt of the uterus to expel the products of conception
? Before the age of fetal viability {20 (or 28)week - FW= % (or l)kg}

* Period of fetal viobility:


Gestational age at which fetus is capable of extrauterine existence
20 wk in USA & 28 wk in developing countries
The smallest weight recorded was 375 g.
* rlliscorrioge is a nonprofessional term used in pregnancy losses.

tl Clinlcol types ,Jt*:


* Spontaneous threatened, inevitable, incomplete, complete, septic, ,missed &
cervical.
+ Habitual 3 or more consecutivti spontaneous abortions (If not consecutive.it's
called recunent).
o I spontaneous abortioa +l5-20%o,
+ 2 spontaneous abortions+ 2-3% &
+ 3 spontaneous abortions+ <17n
.
+ Therapeutic Teimination of pregnancy before the age of fetal viability when the
continuation is dangerous for the mother.
* Elective /i//W// termination of pregnancy due to a social cause (not religious)
I Arl;vtry oS f{Su4
rt,,r: 2nJ
'-ry
eqrl p-era.
ult

io at lal;r
! lncidence:
tr 12-20% of all pregnancies
fr True incidence may be 50-80 % due to
. Subclinical (pre-implantation) abortion before patient recognition
. Not all cases are notified especially illegal
l.- 80% occur in I't trimester (mostly at the 3'd month due to I in progesterone
from CL while placenta is not fully developed, the window gap)

o Age of couple
o Balanced translocation carrier
o Previous abortions & malformation
s Couses:
A. Maternal
- Accounts for most of 2nd trimester abortion

t. Diseases as PE, cardiac, chronic renal failure

z, /nfectbnSTORCH EB:
erial
- Syphilis, mycoplasma, ureaplasma, Chlamydia

- Listeria monocytogenes: listeriosis, meningitis & spontaneous abortion.


- Any organism causing high fever as typhoid fever
g vlruses: acute viral infection, CMV, herpes, rubella
7 Protoz.oa: toxoplasma, malaria
The mechanism of abortion in infections
l - The organism can cross the placenta & affects the fetus.
2- Fever ) release of prostaglandins & stimulation of uterine contraction
3- Malaria can obstruct the placental blood vessels

r AuLoimnuuity
-
SLE: Vasculitis & placental insufficiency.
-
Antiphospholipid antibody syndrome: antibodies against cell
membranes & AT3 )
thrombosis & PE
AIIo lmnunity as Rh isoimmunization.
HLA eharing (due to lack of formation of blocking antibodies g;tiYl 6t3;;
r Corpus luteum insufficiency (commonest endocrinal cause)
- The diagnosis of luteal phase defect is difficult
- It needs an endometrial biopsy to diagnose it (retrograde diagnosis )
- Wide range of normal progesterone level.
a PCO
- tAndrogens
- Premature luteolysis by high LH

' Others: tJthyroidism, DM


5.8u9
* As cytotoxic drugs or chemicals as lead, heavy metals or radiation.
6. Irauna.'
* Direct or surgical (removal of corpus luteum by mistake in appendectomy)
- Local (Fresh fetus)
L Patulous internal os: either
! Congenital or
! Acquired (conization, cauterization, amputation)
2. Bicornuate or septate uterus.
l. Snrall uterine cavity: Fibroid uterus, Asherman syndrome
4. Uterine overdistension: as twins, polyhydramnios
5. Fixed RVF (14-l6wks fixed time of abortion)

B. Fetal
-Accounts for most of 1st trimester abortions ) malformed fetus
r. Genetic:
r 50-60% of lst himester abortion.
e The most common abnormalities are
- Trisomy (52%)
- Polyploidy (26%)
- Xmonosomy (15%).
+ Bliqhted oyum: (Anembryonic pregnancy l+ re'dle").
No fetal tissue by U/S > gwls homogenous & structurless sac
There may be a yolk sac, but a fetal pole is not seen.
Due to severe chromosomal abnormality + non development of inner cell mass
Anembryonic gestation is suspected ifno yolk sac is seen when the gestational
sac > l0 mm, or when no fetal pole is seen when the gestational sac is > l8 mm.
Evidence supporting the diagnosis of anembryonic gestation would be a clinical history
of a more advanced gestationalage than what is seen, the presence of bleeding and
cramping or abnormalities of the gestational sac itself.

Z. Circumvallate placenta
3. Twins, PHA

[. Chiomosomal l- uterine:

2. CL Defects - Anatomic defects of the uterus


3. Infection - Uterine over distension
4. Immunologic 2- Circumvallate placenta,
5. Anatomical 3- Syphilis and Rh isoimmunization
6. Maternal systemic disease 4-'Trauma: directly or indirectly as
irradiations >3 0 0 0rads

y Pothology

+ Hemorrhage in D, basatis ) detachment ) uterine sontraction ) expulsion


of the pregnancy sac either as an intact sac containing the embryo or in
fragmented parts i.e. embryo, chorion, decidua.
+ Missed abortion: reorganization of the hematoma ) keep on viable placenta.

+ Chromosomal Anomalies ) hydropic villous degeneration (not VM) occurs,

+ Amenorrhea) bleeding ) then Pain


o 0r Arnenorrhea ) then ROM )
then pain

L Differentiol diognosis l+ l+ er :
r,. EP-VM. Membranous dysrnenorrhg! (pregnancy Test ) -ve),
-i
ott

.;.Lgcai causes: known by speculum examination as ulcers, tumors


.r Implantg$on Dleedinq (Hartman's bleeding)
!
Due to erosion of some vessels 1 week after fertilization
g Importance: cause an error in calculating the EDD
.1 Degif, uaf Hemorrh as g.
g Scanty bleeding at time of menstruation that stops at 12 wk (when decidua
parietalis & capsularis fuse).
!
Due to minimal shedding of the decidua.
F.,tAolog?i

l|f lrinetfii 2al T?imcthr

TAc,,
&ntrac h'oAs

A-> BlrrC,y *fun

, 2- Diognosis ofgostoEiottqry -Ug_C_&


: 3- Diognosis of fetol life (if fe
--l-
llfe is
L jU=_ o go;g gr-q1s;gd_ob_ortion). __
t _ _4-_Egoli-v?
Dioonosis of_c9rge-$olrnolformo[ion.
----:--:------ ---r I
,__ 1-_D_to_gr.o_ll_s_oJ9t!gltlgg ov:um._
l- _ E :D_!-";a"!E eG.,!ljete-p'eg1'o,n!,[ _ _ : _ :-__.,
: : _ _:
1_ _ Z-_D_1"_gno-sls_of
vejgglo_r1n-o!9.- _ _ _ -_ _ I

' 8- LocolizoEion oF olocenEo


r Definition J;..t-i:Jt 1J$ ru,
- It is an attempt of the uterus to expel the products of conception
- It is due to partial separation of the fertilized ovum from uterine wall with
slight hemonhage into the choriodecidual space
- It may succeed & the patient aborts or fail & the patient continues pregnancy.
t Synptoms#-ll,;n ulltr r.l1
- Amenorrhea t symptoms of early pregnancy
- Bleeding: mild
- Pain: lower abdominal, colicky fftl+J J+!Jld o-,x
!- Sgns
- Signs of early pregnancy
- Abdominol:uterus conesponds to the period of amenorrhea
- PV Ex: the cervix is closed ls u{e
\ fote
* Con tn ua pregnan cy 7 0 -80% i. e. l! rgate n ed abortion.
* Bleedtng ntreases, inevitable abortion
* Fetus dns lut retained. missed abortion
* lnfection nccan septic abortion
x lnvestigotions
- Investisations to diasnose
o US: +ve fetal life (TV[JS 6wks, TAUS 7wks, Doppler at l0 w).
o B-HCG: repeat in 2 days (must be doubled), if -ve = missed abortion
- Inv, for the cause: S. Progesterone, DM & Thyroid function tests
LTreotnent:
Consenrative
t Rest:
- Physical: rest in bed till bleeding stops for I week
- - Psychological: May give sedatives as valium 5-lOmg/d.
- Sexual: Avoid intercourse & PY examination
g Vitamlns and minerals (iron)
g Progesterone (contoversial) only in cases of luteal defocr because it
- tmissed abortion (masking effect)
- Virilisation of fema!-e fetus (not with natural forms)
- Preservation of a CFMF, even a VId, maybe retained.
r Antisnasmodics . Br symDsthomimetics or Ritodrine after 20 wks
t Anti D in RH -ve
"Def;,ufidn A
c r7^ft6ms A
'tg-P
.S;gns.q l

o fiLJs f krcL I
-

. t'Av; C;t'ffie
t\o . ltl 1 Lt€*efccl
-.-
\)
qrlcl;caI
__, fu(j;c*f

-Anh'D
tteE
S7r-\7z-\7r=l

Terminftion
I -lnevitoble if >9 of dte following qitaria ore present:
t) Severe bleeding (especially.if > 6 hrs, prolonged >7days)
e) Pain: severe persistent lower abdominal cramps in spite of analgesics

3) Rupture of the membrane


4) Dilatation & effacement of the'cervix.
S) Partialprotrusion of products of conception
2-furned into septic, missed

* Definition
- It is a type of abortion which cantt be prqvented
- There is complete separation of the fertilized ovum or the placenta
- There are progressive cervical dilatation & fetal expulsion.
\ Symptons
- Amenonhea + symptoms of early pregnancy
- There may be collapse
- Severe bleeding
- Pain
I Severe
1 Colicky due to uterine contractions
r Lower abdominal
1 Backache = cervical dilatation 9 sacral pain.
!- 5ln9s
q Generol
- According to amount of blood loss (pale or shocked)
6 Abdonrino!
. Tenderness
- Uterus conesponds to the period of amenonhea
6 P\V
- Cervix opened
- There may be prolapsing products of conception
\ lnvastigotions:
- It is a surgical emergency ) no investigations except routine pre-
operati ve investigations.'
2 Treotment oYi Jly
. ReSUsCitOtion dr,Aill\,".':<'.j {rgr

1. Oxygen
2. Restore circulatory volume
' ? Trendlenberg position
? 2 wlde bored cannula
? Fluids
{ Co'l I oi ds as albumin , glucose 57o a.{apJg0
& Crystalloids as NaCl 0.9%, ringer lactate
\ cross match for possible b'lood transfusion if bleeding is
brisk or if the iuitial hemoglobin is less than l0 gn/dl
3. Drugs as dobutamin
4. Eliminate fts cause
5. Reassess the patient; HCV > 30%, UoP > 30 mU h

Terminotion
. lst trimeste[
@ Evacuation and curettage: suction & curettage ars most effective.
@ Oxytocin before E & C I the possibility of uterine perforation.
@ A sharp curette is used to ensure complete evacuation after the
suction.

- ?nd Trimester
O Ecbolics (orytocin or PG),
O Administer IV drip with l0 units of oxytocir/ 500 ml.
O The oxytocin contracts the uterus, aids in the expulsion of tissue or
clots & limits the blood loss.

Antibiotics : Reduces possibility of postabortive infection.


. Anti D if Rh -ve

l Definition: Part of products of conception is expelled

I Symptoms & sings: As inventible abortion but

' There is history of passage of some of the contents


. Uterine size < period of amenorrhea.
. Treatment: As inevitable abortion
r Definition: The whole product of conception are expelled

r Signs
@ Bleeding: mild
@ Pain: little or absent
@ Utqrus smaller than period of amenorrhea
@ Cervix is closed ) Good prognosis

InVestigatiqns
O U/S: clear endometrial line + empty uterus.
Treatment
! Nofurther treatment
! Some may give ecbolics & antibiotics
! Some do D&C to ensure complete evacuation &decrease infection).

(Carneous F.sJMissed mole = FleshY mole = BIoodY mole)


r Caus'es Retention of non viable products of conception due to:
:- viable Placenta
- Exogenousprogesterone
r Symotoms lj14 g++ht lri-ii. &6+Lr
- Loss of symptoms ofpregnancy
- Cessation of fetal movement (if felQ
. Breast (milk) discharge: may be present due to decrease estrogen,
normally estrogen blocks action of prolactin on breasts during pregnancy.
- Brownish (prune t,. lyjuice) vaginal discharge; due to dissolution of
".retroplacental hematoma (hematin = destructed RBCS)

- Pain is unusual
r Signs
! General: no general signs ofpregnancy
! Abdominal:
o J Uterine size less than period of amenorrhea
o ['HS are not heard q+
! Vaginal
o Cervlx is closed and firm
o +/- Dark brown blood
a Complications
* DIC Oypofibrinogenemia)
- It takes > 4 wks to occur (slow)
- It is due to liberation of thromboplastin from retained dead tissue
It consumes fibrinogen by 50mgldllw till < lO0mg/dl in 4 weeks.
.i. Infection & septic abortion
a Investigations
* US: collapsed sac, no fetal pulsations
* BHCG: no doubling & becomes -ve in2-4 weeks
oao

o lN PR€GNANT 400 - 600 mg/dl


o lN NoN PR€GNANT 200 - 300 mg/dl
: o CRITI(AL L€V<L below it DIC occurs is 100 mg/dl
+ Tieatment
O A((ORDING TO FIBR'NOGGN L€V€I-
<1100ry P-ll@ry
+ TTT DIC: cryoprecipitate,fibrinogen, FFP, Fresh + Treatment
blood. Heparin (only in dead fecus, intact circulaiion,
main indication is slngle fetal demise). ! 2od tri: ecbolics
+ Then terminate ) Antibiotics

N.B. Cx abortion

Definition Products of conception are Implantation


retained in the cervical canal

Shape Banel shaped cervix

lnt. os Opened Closed

Ext. os Closed Opened

Pain ttf t
Bleeding t ftt
Ecbolics/ dilatation of the - Cauterization of the bed
Management - Under running sutures
cervix
- Folley's catheter
- Pack, circlage, embolization
- If failed hy sterectony
C,$
qb crh ort _/
t5l
GI
G Sqfficdborffiorn
r - Causes: Superadded infecdort oft any type of abortion.
Criminol obortions ore considered septic
.o
o 0tottt +ye staph, stteptococcl especlelly group B (cBS)
'- o 6rgn -ye. t,coll, Proteils, hlebslella
o Aruannn anerobic Sttept, closttldla & bacteroids.
o
o. lrstfuinents, sanltary pads
o Etldogenous,'orgafllstfls ptesetrt ln fetnale genital tract
o Henitogmous(rar\, ftoin e septlo foous e,g appendicitis

Symptoms Signs
* Seotic * Abortron Generol: Fever, rigors, tachycardia,

- FAHM
Jaundice
Abd: lower abdominal tenderness &
- 'Acute Abdotnen
- rigidity, Thoracio respiration & tender
Offensive discharge
uterus rnay be felt
- There may be
PV:
trristory of a trial to
- Offensive discharge/bleeding
:induce abortion by
- Physometra (uArne crepitatrbn
irairained personnel
due to infectlon by Chstndt'an/
- Foreign body
- Swelling in DP : pelvic abscess

r Complications
t. Lo,eal:
: d Endometritis, myometritis, perimetritis, Parametritis, salpingitis,
salpingoophoritis, pelvic peritonitis & pelvic abscess
l( General:
' d Generalized peritonitis, pelvic thrombophlebitis, septicemia, pyemia

,r( glglr! &tlyrq i


Renal Failure

pre renal failure from dehydration .

Renal failure due to toxins or drugs used in treat ment .


n,-oh $ij

Seph'ce,nu, fYtna
li
DIc

:1
\

felilc

ocK
Lunj d n
/ RRDs
t

rlotr ( s IRs)
d Liver cell failure, hemolysis (clostridium & strept), septic shbck,
ARDS, DIC & multi organ failure.
(Systemic inflammatory response syhdrome = SIRS)

I U/5 dead fetus or incomplete abortion

t To know the etiology


O OENETOI IWBCS, fESR, TCRP
g Cultures vaginal discharge, blood (at peak of fever).
* To know the complicotions:
I Orgons LFT, RFT, DIC
q X roy:
Abdominal: (Perforation)air under diaphragm)
- Pelvic: (clostridium) Physometra.)
'.:.

-{ Treatment
.t Hospitolizotion. resuscitation if need,ed

+ Antibiotics (in hiqh doses in combinotions)


. 9 Gram+ve: penicillin G, or qephalosporins
o Gram - Ve I gentamycin or tobramycin (less nephrotoxic)
o Anerobes I Metroni dazole, or clindamycin
o ln gas gangrene : antitetanic serum
'o Till C & S) Specific antibodies
n Evocuotion of contents
1tttrimestet : Suction eyacuation to avoid
!' Perforation of the soft uterus
S' Spread of infection by opening sinuses
! Injury of Nitabuch layer) placenta accrete in next pregnancy

2nd trimester) ecbolics & if failed hysterbtomy


o Sgvere infection) hysterectomy en toto
S Clostridium, shock not responding to antibiotics, perforation
! Old age, completed her family
'TEofmenf

@ Rrrrcital,on

eb,rrh'on

@-trufmcnt'f

I Gnf Ucat;6n

W,J, TN, *)-rr *U, /


* Close observotion: in the ICU in c licoted coses
sr Vitol doto: blood pressure, pulse & temp
sr CVP especially in renal affection, urine volume, repeated
function tests
SJ

ST

o f tissue perfusion
o Stabilize lysosomal membranes & endothelium
o Increase BP &myocardial contractility
o Restore sensitivity to catecholamines
Treotment of complicotions
+ Pelvic obscess: drain by laparotomy or posterior colpotomy
+ Septic thrombo-phelipitis: heparin
+ Generolized peritonitis: laparotomy, peritoneal toilet & drain
o Renol foilure: dialysis

')',EI9: fibrinogen, FFP, fresh blood


o Circulotory collopse: vasopressor & sympathomirietics drugs
+ ARDS: assisted ventilation

l< Any bleeding with pregnancy.


t( Ectopic pregnancy.
l( Rupture membrane (ascending infection).
ft Virgin.
l( Histoiy of contact bleeding
*' w l+D ColVqoria
r;..-.. .-; J a--1-/
stt 5

U a-
(E
q.
T
l- ;1 -r,
3 1 4
{ f
PoSt abo111'64
3 o \
S. \o
!'- t-
o- A g
2 € d _&. u
r
.T J
, s+ )frim<sF.r \ \ I I I I \ \
u\4 +fioleJ[Zr I z I \
dlf.tct tegtic au ' \ I I I q t I
s Definition:
o 3 ormore zuccessive sponhneous@Oabortions(somesay2)
I If not succasive it is called repeated or recurrent abortions
* lncidence of obortion:

- Riskof abortion: l0lo - Once:29/o


- lVisu 26%Q-3%ofcommunity) - Thrice:32%(<l% ofcommunity)
e Cat*s: 50% qe idiopohict+pft

*They rsually lead to 2m himesteric abortions (usually Asymptomatic not diagnosed

before multiple abortions).


.:'They constitute:
+30% of ?d trinestrrc abortions d. PJll
+15% of lst tnnestnc abortrbns

) €tiolow
*Congenito!
Qlncreased muscle tissue in cervix > l0% or poor collagen type.
ssociated with uterine malformations as septate, bicomuate, hypoplastic utenx
iethyl-stilbesterol @ES) exposue in utero &Jtl,/ gi'a''r 6Lraid fut
*Acquired
o Due to obstetric trauma
I Forceps or ventouse or breech exhaction before full cervical dilaation
'r Manual dilatation of the cervix
oDue to svnecoloeic trauma
t Dilatation of the cervix excessively or too rapidly
l Hi$ amputation ofthe cervix' Cone biopsy ofthe cervix
I Clinicolpicturo:
.a
- Painless effacement & dilatation of the cervix
- Uterine confactions are late & not very painful
- PRONI followed by rapid delivery of a fresh abortus with minimal discomfort
. The obortion ocans in descendingfoshion = at 7 m then at 6 m then at 4 m, otc.
9 lnvestiootions:
+Durins Dresnancv
. $ Seria I U/S examination (better done trans-vagina lly)
$ To determine length (2.5-3 cm) & width (l c of internal os
! Herniation of fetal membranes.
.r In the non-pregnant state
! HSG t firnneling (loss of uterine waist)
t Ability to pass Hegar dilator No 8 or hysteroscope No 8 with no
resistance or pain
! Pediatic Foley catheter with lml inflated balloon can be pulled tluough
the os without resistance
I Treotment:
o
o Trachelonhaphy may be done in case of cervical tears
+In the pregnant state: circlage (Circlage oW 'S q',is")

1- Voginol circloge (success rote 85%):


a- Modified Shirodkar
/ Purse strins sutures (4 bites) around the level of int. Os using Nylon or
Dacron tape (Mersilene r.r ) using aneurysmal (Shirodkar r^r+) needle deep
to the vaginal mucosa.
, ./ Avoid3 &,9 oclock sites related to uterine artery to. avoid its injury
/ Ends of the tape are left long, hanging from posterior cervical lip to be removed
{ not done, a piece of fascia lata was buried
underneath the cervical mucosa to be delivered by CS

b- +McDonald's operation (easiest)


. Purse strins sutures around the portio-vaginalis no incision of the vagina or
. ' , dissection of the bladder .r$ sLll .fi cl^sr
2- Trons-obdominol circloge:
/Through laparotomy -; delivery will be by CS (permanent)
/Indication:
l. Mullerian fusion defect & diethyl stilbesterol uterus.
2. Failure of previous vaginal circlage.
3..Irreparable deep cervical tear.
+. Viry short cervix, cervical conization
r If failed before 28wks hysterotomy must be done (a great disadvantage)
ShiroJ l(ar C irs/a1e

l_(CDoildtd,, C;rchre

I lanr ot b.lo *-r ; naL C'fc/*tc


,/

Three levels of cetclate suture5:

Transabdomlnal e.clage

shlrodkar,r suture

McDonald's iutu.e
Timing of circloge: f2-f6 wteks (risk of malformations is gone)
Post operotive core:
- Anti-prostaglandins, p2 sympathomimetics, progesterone
- Antibiotics
- Intercourse is forbidden
A, Removol of the toPe ) 2 weeks before EDD (about 37 weeks).
s fndicotions of circlooe
t- Cervical incompetence
2- Other uterine malformations as septate, bicornuate uterus
3- Multiple pregnancy
4' Cetvical ectopic 9fgljl
5- Placenta previa rg{lil
fndicotions of cesorean section with circloge
I. Abdominal circlage
II. Original Shirodkar
II. Associated.indication
Comp!.icotion of circloge:
? Iniurv to bladder, Abortion or PTL
ROM ) tape must be removed & manage
"
? Infection ) tape must be removed & terminate

o Uterine hypoplosio:
' Usually leads to abortion in ascendingmanner
' Treatment:
' Pregnant: vaginal circlage

' Nonpregnant cyclicE+P


o llterine molformotiors (bicornuoto 30%, septote 25%)
S During pregnancy: vaginal circlage
-!. ln the non pregnanil Metroplasty (surgical removal of the septflm or
unification of the uteri). Done only if there is recurrent failed circltge as
mefioplasty leads to seva'e adheslons
o Submucus fibroid:

-!. Pregnant: circlage

!. Non pregnant: myomectomy


Fired frVF:
s Abortion at 14 - 16 weeks
! Treat ttre cause of fixation (venfosuspension not fixation as it leads to psterior
sacculation)

Congenitol flshermon syndrome (inaoubrina synechio) ) dissection of


adhesions

c/P ,NVESNGANONS TREATMENT


Diseases, HTN, {.History of the t RFT,LFT {' TTT of the
Renal cause cause

Immunological *History of * SLE: ANA, Anti dsDNA tttt Low dose


(sLE, APL $ SLE, DVT .i. APL$: aspirin,
- lupus anticoagulant {. Heparin, CS
- Anticardiolipin Ab
HLAsharing * HLA typing *lmmunotherapy
Rh isoimmun *Failure to * Amniocentesis
take Anti-D
prophylaxis
.i. Arterial & * Assay of clotting factors .i. LMW heparin
venous
thrombosis

Endocrinal: * History of * Thyroid function tests, * Thyroxin,


DM, the disease OGTT, S. progesterone insulin,
thyrotoxicosis progesterone

Infection * History of .l Serology .l TTT of


Syphilis, CMV infection Syphilis, CMV

(other infections are not believed to be a cause of habitual abortion)

tFetal (4 - 107o) * History of {'Karyotyping * No TTT or


(genetic) CFMF AID

o Preventlon of obortions:
g Most abortions cant & shouldbeprevented (as in CFM$
9 Empirical teainent of idiopathic cases:
! Progesterone
s Low dose aspiriq / low molecular weight heparin
s Folic acid

rN/zA\%
o Definition
n dn outoimmune disease with formation of antibodies against phospholipids
' ! It may-be lry or 2ry (with other collagen diseases as SLE)
o The syndrome is ccc by recurrent
-- s Thrombosis (arterial, venous)
s Fetal loss (abortion,IUGR, PTL,IUFD)
s PIH<20 wks (usually severe) & plaantal atruptton
o Diasnosis
--
g Serologicol criterio:
- Anticardiolipins antibodies (ACL)
- Lupus anticoagulants (LAC),
- Chronic infections, malignancy, stress
- May be normally prqsent in low titers in aboul?%

s Soecific clinicol criterio:


Z 3 successive unexplained abortions <10 wks after exclusion
genetic or endocrinological causes

2l unexplained death >10 weeks of a morphological nor-rnal fetus

2l Preterm Labor <34 wks due to PIH or placental ins'ufficienci: ,

g Diognosis) I serological criterion + I clinical criterion


o Treatment:
s Asoirln 7imsld
$ Heparin 5000 units SC twice daily or LMW heparin (clexane)
. 30-40 mg once daily
- No need to monitor it as it does not change PTT,= no bleeding
however it is monitored by factor Xa
s Corticosteroids: no more used
n Under trial : immunogldbulins & oral anticoagulantsc)rl.l.l.,/
I ) Personol history:
!- Age: chromosomal abnormalities, DM, hypertension.
! Address: rural areas (bilharzia$, slum areas (toxoplasmosis)
!- 0ccupation for exposure to infection as medical personal & teachers,
exposure to radiation, exposure to toxic substances as lead .

!- Special habits as smoking, alcohol, pets (toxoplasmosis).


1 LMP: for pregnancy dating

2) C/O:
O Abortions > 3 (2) times = repeated
j) History of present pregnoncy
- Symptoms of abortion:
' Amenonhea, pregnancy symptoms, bleeding, pain, ROM
- Symptoms of complications:
' DIC, fever
4) llenstruol history
o LMP & dates for gestational age
o Premenstrual spotting ) lutealphase defect
o Menorrhagia ) fibroid
o Hypomenorrhea ) hypoplastic uterus, Asherman syndrome.
5) Obstatric history
A. Previous deliveries
.) For history of maternal disease.
o For history of traumatic delivery as forceps, ventose, breech,
postpartum hemorrhage from cervical tear.
o For'history of failure of Anti D injection in Rh-ve.
a For history of premature deliveries, forhistory of SB, NND.
B. Previous abortions
t- Timing:
' o l" trimester abortion are usually due to ovofetal causes
. 2nd trimester abortion are usually due to maternal causes
2- Tf the fetus is
\ Macerated : suspect general case
\ Fresh : local cause.
\ Ma tformed ) fetdl cause
3- Abortion
- ln descending manner : patulous os, Rh
- ln ascending pattern 8S hypoplastic uterus
o RvF
{: m t n la'' to,J: # :, : ffi :,iJ" ;ffi l,: il":,|J"'
"
s- Speciol C/P: in patulous os (painless, rapid)

6) Post history:
o MediCal for maternal disease as DM, thyroid, heart disease, drugs
o Surgical & gynecological operation on the cervix
7) FomittlhistorY for hypertension, DM

! Generol; hypertension, renal disease, thyroid, SLE


! frbdominol swelling: fibroid uterus, ovarian swelling
1) locol L

- Uterine size, position, local pathology (bicornuate 2 bodiei)


- Cervical tears, dilation, and condition of membranes

1. General cause .

- Endocrinal :GTT (DM), progesterone level, TSH


Urine for DM, albumin, bacteria

,r - Immunological.blood group, Rh, SLE, APL


, Infections. V[)RL for syphilis, tests for toxoplasma, Rubella,
- Organs. renal & liver function tests
Loca I cause:
- Ultrasonographlfor pregnant & non pregnant

- HSG & hysteroscOplf6l local causes in non pregnant state


3. Feta I cause:
Chromosomallstudy (of the abortus & parents)
- Postmortem picture of the abortus
1.
- PreconcePtionot: correct any detected cause
- During pregnoncy
t Rest (no travelling or intercourse) + good nutrition
L Vitamins, minerals, no smoking
L Folic acid before conception & for the I't trimester
2, IfeAJ-g-enera.l--Ae-USes as toxoplasma is treated by sulphadiazine in pregnancy
& in non pregnant either sulphadiazine * pyrimethamine or spiramycin
3. Trpa! s circlage

l- Bisks from tho obortion itself


A. Ilemorrhage: which may be severe
B. Blood clots in the uterus )
severe cramping.
C. (in criminal abortion )
D. Shock may occur due to hemorrhage, pain or infection.
tr. Pelvic infection (sepsis)
.F. RH isoimmunization
c. Infertilitv.
H. DIC in missed & septic abortion.
I. Increased risk of breast cancer
2- Bisks from onesthesio
j- Bisks from surgery: cervical tear, uterine perforation & infection.
4- Post obortive bleeding
o Immediate
A. Atony
B. Retairred parts
C. Laceration of the cervix, uterine perforation.
D. Hemorrhagic blood disease.

I. Retained parts (may from a placental polyp )


II. Infection.
lll. Associated gynecological cause.
5- Choriocorcinomo
Definition:
o Termination of pregnancy before fetal viability to save maternal health.

lndications
Maternal
Fetal
Ceneral Local
i Severe diseases not o o Blighted ovum
Acute PHA
responding to ttt (HT,
Q Cancers
+ CFMF & teratogens (STORCH-
Renal, Liver, Hyperemesis) o EB, inadiation..,)
Incarcerated
i Breast cancer, melanoma 0 Missed abortlon
i Mental retardation
RVFgravid uterus
+vM

Def inition:
o Interruption of pregnancy due to social (non medical) factors
. Illegol uit6+yt+ t+ Cr* $lt rlilt y'
. Elective Ft{+ylilt+ Cr.,*.tt $Jl,/
? Methods used:
l- Uterine stimulation ) purgatives, methergine
2- Intrauterine manipulations to induce cervical dilatation or ROM
3- Usually done by untrained doctors in septic conditions

4- Common complications:
- Genitaltrauma as perforation, Asherman

- Infections & septic shock


5- CIP & treahnent as septic abortion

Vocuum ospirotion (Karnan rannu/a)


- Advantages: less time, trauma & perforation, bleeding & anesthesia.
I Dilototion & evocuotion
I Hysterotomy
I Hysterectomy enToto
.I:!i.9!=.
,D+C
Sucfion ?vaevili^
. H'Trferob*7
. ilys/Trrctomy ?nfotr;
edicol:
o Oxytocin:
, I amp (10 u) + 1000 ml of lactated ringer, Start infusion at 0.5 ml/ min
(50 mu/min) the rate is increased every 20 - 30 min up to 200mu/min

+ PG (E2, F?a)t intra-amniotic or extra amniotic or vaginal or oral routes Used


before'the oxytocin or before the curettage.

e 20% salinel 30% urea/ glucose (50%).

I Not done because they may lead to


. Hypernatrenia
, Hyperttoleni4 water intoxication & lteart fai/ure
, 'Myonetr/a/
danage (due to extrarasation/-> D/C (t/rronbop/astn
fron danaged trssues)
, lnfection fueritonitb & septrc s/tock)

+ Anti progesteronq.
I it blocks progesterone receptors. Oral intake of 200- 600mg
followed 36 hours by oral or vaginal 400- 800 micrograms of misopristol.
, it inhibits endogenous progesterone synthesis.

a Vorious combinotions of the obove.


o
be sensitized.

After 12 wks

Oxytocin, PG's

a
R.0,1\4
:- D&E
a
Intra-amniotic hypertonic fl uids
. Menstrual extraction a
Hysterectomy
. Laminaria
. tents
,,,!.r a
hysterectomy en toto
Definition: Implontotion outside its normal site (uterine cavity)

Sites:
9 Uterine: Cervical, Rudimentary horn & Angular
o Extro-uterine:
q Tabe 7-99%l: interstitial 2%, isthmus lTYo, ampulla 78%, fimbrial end 5%
+ Abdominal, @[an-.!;!%, & intra-ligamentary
lncidence:
- l-3% lts
- l0% maternal mortality (/0 trnes ragrhal delittery & 50 trnes nducedabortr'ortS

- Increased nowadays (4 folds) due to o: :

. 1STD with better antibiotics


1 Tubal surgery, M
(GIFT,ZIFT)
. Contraception as
* Progestin only pilts lJtubatmotilitv)
* IUD r'l--l-r

g Salpingitis, Jtubal motility (if + P)


../ lt can prevent intra but not extra uterine pregnancy 6.lYl
-:
. Infertility $0%)
. Term pregnancy (30%)
1 Recurrent Ectopic (20%),
a Abortion (10%),

CauseBt
- Causes in the hrbe:

! gongenitot: hypoplasia, diverticulae & absent cilia (DES exp6sure).


! Tr,o9motic' , i,

r Surlery near the tubef appendectomy so ectopic is more ln the Rt slde.


. Surgery on the tube: tuboplasty, tubal ligation

! Inflommotorl@si
e Salpingitis (>50% of causes gchlamydia is the most common)
6 Appendicitis.
Co,
#
use s
YUl?t
in //'
O

Cons
-l.
-Eo, (;L :::l
- H igrofiion(ff:
s- NeoPlosn tumors in the broad ligament )stretch the tube
I Miscelloneous: Endometriosis, methods of contraception& IVF.
- Causes iri the ovum:
o External & internal migration.
+ Early appearance of chorionic villi or early disappearance of ZP
. Generol pothology
- Tube:
o Any part may be affected especially ampulla ) enlarged, vascular
o Rarelv: bilateral ectopic or heterotropic (intra & extra-uterine) l/30.000
(l/7.000 - l/4000 in IVF & ET)
o Can not reach > 12 weeks due to
I Limit.ed tubal distension
o Poor blood supply & nutrition
o Thirrn-Qr decidua (ovum penetrates deep in muscle)
- 0vary: It contains corpus luteum of pregnancy.
- Endometrium:
o Symmetrically enlarged up to 8 wks (hornona/ effal
o No villi
o Arios Stello reoction
fftypiccil focol odenomotous hyperplasio
s- In lo- 15 % ofoases
s- It is due to extreme progesterone * estrogen effect.
!- It is found in ectopic pregnancy, twins, vesicular mole, normal
pregnancy & in those using high progesterone.

+ Pelhp-logy: The f ertilized ovum becomes embeddzd in the tubol woll


+
'/ Symptbms;

o May be earlier if in the Isthmus or disturbances occurred


before missing a period
o Or later if cornual (2-16 w) or rudimentary horn (l6w)
+il1 +,
(ne.m
tulltl.<
/nla"'.\\
+t
+++

Frr h lizeJ ov,,';


rod
^,,-'
?r/ fiuJ frla,a", rnfirc/c
" h.,
ct?n. *yrJrr Ctn (o eJt
I
I Sigiis:
General: p'1ss
Abdominal: slight tenderness in one iliac fossa
PII:
o Extreme tenderness on cervical movemenl (jumping sign).
o Tender adenxum
o Mass may be felt in one adenxum (< 3cm)
To diagnose the undisturbed ectopic, it needs high level of suspiciorl
(ectopically minded l.s r4.' uls).
CombinaUon of history of risk factors * *ve pregnancy test * pain in
one iliac fossa is very'suspicious
The most common cause of acute abdomen in childbearing period is EP

+ Tubol nrole:
The fertilized ovum dies with blood accumulation (hematosalpinx)
Fate:
r Tubal obstrtrction or
o Blood becomes absorbed.
Tubol obortionr
o Contraction of the tube ) expulsion into the peritoneal cavity with:
oma (if mild hemorrhage) or
I Pelvi0 hematocele (if moderate hemorrhage)

T*ml::$'B I

peritonitis or drains thrdugh the


posterior fornix
, (ifsevere hemorrhage).
9 The ?boitibn may bd complete dii ds or incomplete 4r.tr .l*lLi 1..ll
Tubol ruptura:
o The tube may be eroded and ruptured
. If ruptured in peritoneal cavity
,Intraperitoneal Hge + Zry intra-abdominal pregnaticy
.
) Intra-ligamentary hemorrhage + 2ry pregnancy.
Clinicsl picture:
o
r'- Svmotom:
Amenorrhea:
'.'. Short period (usually 1 missed period)
.f. OR absent if disturbance occurred before missing a period
(vaginal bleeding is mistaken as a menstrual cycle). This
occurs especiolly in isthmic pregqoncy 15 ril; r
* Rarely, symptoms occur after 6 - 8 weeks.
Pain:
. . Dull oching in tubal mole
. Colicky in tubal abortion
. o Stitching in tubal erosion
. , Stobbing in tubal rupture,
' . Acute obdomen & righ? shoulder pain in intraperitoneal,,
hernorrhage.
Bleeding:
o Due to I of B-HCG ) JE&P ) separation of decidua )
scanty bleeding ,3J+f eJt $ ,J+ sll Cr.r# ll r.t ofu
o Sometimes a decidual cast Cli may be passed
Fainting attacks, nausea, vomiting (intraperitoneal hemorrhage)

General:
r Various degrees of shock 1tf, Jef, oliguria)
. Difference
. .t of BP > 2Ommhg on measuring it while the
patienf ii sitting & Iying doivn,rJrh .rrij rltir 6l UtnL^
Abdomen:
. Tenderness, rigidity & rebound tenderness over lower
abdomen
I Respiration is mainly thoracic ( J, abdominal movement)
PV: (very difficult & may t the disturbances)
o Cervix: extreme tenderness on movement (jumping sign).
o Adnexo: tender swelling in one adenxum & arterial
pulsation may be felt.
o Douglas pouch: boggy swelling in pelvic hematocele
)
l. Pain:
* Acute sal?/ngttts: 11s amenonhea, no fainting, fever, pain
usua I ly bil atera l, leucocytosis.
*
* Acate appendicitn no amenorrhea, vomiting, pain usually
periumbilical then at Mcburney's point. .
* AcuteP.ye/onep/tritit loin pain radiating to the groins with fever
& urinary symptoms
2. Bleedine: from abortion & vesicular mole:- .

+ Acute fulminating type


O pathology:
9 There is massive intraperitoneal herilofihage with shock & collapse.
o symptoms:
I Short Period 6f alnetrorrhea
o *t:t
,uuuro severe abdominal pain
Shoulder pain due to diaphralmatic irritation by blood.

! Vaginal bleeding
o Collapse
o signs:
* General:
o Shock not proportional to external bleeding
* Abdominal:
o Tenderness, rigidity, rebound tenderness t{ls 0hll
o Shiftins dullness (rare & late).
o Cullents sign
- Bluish discoloration around umbilicus
- It is due to absorption of blood in peritoneal cavity
by lymphatics) rr
'i' Vaginal:
oDifficult due to marked tenderness
olt is easy if the patient is shocked
O D.D: acute internal hemorrhage & pain e.g rupttrre spleen
A. Chronic Pelvic hematocele
9 Pothology: Blood in DP ) organization, infection or rupture into the abdomen
I Synptons:
! (amenonhea, pain, bleeding)
t uria, Dyschasia, deep pain & dyspareunia
9 Signs
! Cened pallor, tachycardia, jaundice, fever
! Abdoninal pelvi abdominal mass ) ill defined, tender & cystic.
g Pt/.
The cervix is pushed anteriorly, Os is directed downwards
- Boggy swelling in DP

9 DD:.
a RVF uterus: differentnted b!
r Bimanual examination
r Direction of cervix
r Culdocentesis.
+

B. Advanced abdominal pregnancy:


: l!! (very rore, tubes &ovories must be nornol)
.fu to tubol rupture
! The tube ruptures and the trophoblast acquires attachment to a neighboring
structure such as intestine or omentum & the placenta attaches to it.
! Fate
- Zrlruptures, IPH & Infection
- CFM|SO% OR fetal death followed by calcification(lithopeCiol )

' Continue to tern+ false labor pains) fetal death& internal hge

! Symptoms
, Hrstory of drsturbed Ectoptc(amenorrhea, pain, bleeding)
, PanYague abdominalpain, acute abdomen & false labor pain
, Feta/noyenentsmay stop
u Signs
g Abdoninal abnormal lie & fetus is felt easily. No uterine confractions
g t[. small uterus away from the fetus
o
proceed to
laparotomy.
9 If not do laparoscopy cr.rilr$ l+rtl
9 Early diagnosis needs
e High level of suspicion (ectopically minded).
. So usually discovered accidentally during routine U/S follow up of pregnancy.

l.Pregnoncy test:
A. Urine pregnoncy test:
7 Slide agglutination with latex detects 25-50 mlU/ml.
? ELISA is more sensitiye 90% detects 10-25mlU/ml
B. D ,-H66. :

!. Earliest diagnosis of pregnancy at 5 miu/ml (mostsewtrrQ


!- *ve lweek post-fertilization
!- Normally doubles every 36-48 hrs
!- Abnormal rise < 66% in 2 days : EP or non-viable intrauterine pregnancy
!- Biochemical pregnancy: means detection of B-HCG >l0mlU/ml done
twice before the expected missed period.
. !. There is a lag (about 2 weeks = the wndow gap) between biochemical
pregnancy 3rd wk & TVUS visualization of the gestational sac (5th wk)
2. U5:
1) Vaginal (more sensidve &specific)
! Ectopic:
' Gestational sac with fetal echoes outside the uterus (difficult)

' So the diagnosis depends on €XC/uSt'1nof the presence of IU sac


) Intrauterirte:
. frT 6 wks:
- Gestational sac (echogenic rim of l-3 mm sunounded by
.. a sonolucent area formed by the choriodecidual reaction)
- At this earlv time. it is difficult to distinguish
an IU sac
' from the decidual reaction ofan ectopic pregnancy
o
frT 7 weeki: fetal pole + cardiac activity very confirmatlve
2) Abdominal: detects gestadonal,,sac at 6-Tweeks
3) D0PPLER: ring of fire pattern in the tube (plaeenta all round),
TIf-
I

sf,
5 S
\ (+\

e F
T e,
3)
\
fl
\
t. I
f )
A

il
C-.
.1or
i
s
p $
n

I
r- o
.l i'i
1tl!'l r
- \
+

$i+$
xi sAt
a \
,l

F
\ P.i
b:
"lrt
1r) rn
s,
h.r
'p
-

b P\-ttto o
T
.i
\ G) n P
rD
ci

,.i
\$ $
t S.
3
Combining p -HCG & US: Ectopic prepancy can be diagnosed
o If p-HCG > 6000 miu/ml + empty uterus in trans-abdominal US
+ B-HCG >2000 mir:/ml + empty uterus in trans-vaginal US
+ That level of HCG is called the discriminotion zone

3. lop o ro s c op lt :(di agnosti c & th erap euti c) p.a .Eir *p


r .Lii$.
r It is beneficial in query cases, but 4-8% of early ectopics may be missed.
r Also it excludes other causes as ruptured ovarian cyst.
r It is the preferable therapeutic route in the haemodynamically stable patients
r Not done if intraperitoneal hemonhage is evident

4.
o This detects hemodynamic instability
o lnterna/ /rye b suggested by I inhematocrit in absence of external bleeding.
oAcutedropismoresignificantthaninitiallyIowreadings.
5. Culdoceitesls(tapping of Douglas pouch)
loody fluid (non clottable): pelvic hematocele
*clottable blood means:
- Aspiration from a blood vessel (false +ve),
- After brisk bleeding
.i.0ther fluids: c Iear in (ruptured zyaflan cyr\,pus in (ah@t$
6.Serum progesterone:
r Normally > 25nglml lsensitire gB%)
g < 5 ng/ml = abnqrmal pregnancy (atoptT ornonnab/e lUpregnanc)

7.
o D&C: no villi, Arias Stella reaction (may disturb an early healthy IU sac)
+ EUA )may t the disturbances
+ X-ray: in abdominal pregnancy
8.ln query coses:Hospitalization & follow up of:
. Symptoms (pain),
. Signs (detectable adenxal swelling),
. Inv: (TVUS, B-HCG, hematocrit) )stilt query: laparoscopy, culdocEntesis
9. I nvesti goti on of obdoni nol pregwncy :
Lateral view X ray
o 0verlapping of fetal & maternal skeleton [Dixon's sign],
o Hyperfl exed fetus (intraligamentary), hyperextended (intraabdominal).
U/S (best): show fetus outside uterine cavity
* Prophyloxisl avoid the causes l+ t+ l,r +,{-
* frctive trootment

Resuscitation : (should go hand-in-hand with the surgical treatment) {t.t{t,rfF#l)e

A [V fluids using two large-bore intravenous cannulae.


A Catheter is placed in the bladder to monitor urine output.
A A blood sainple should be obtained for typing & cross matching.
A Transfusion of blood or other plasma expanders. The operation is started as soon as there
are signs ofrgsponse to resuscitation.

1. Laparotomy:
y' It is suitable fe1 3 hemodvnamicallv unstable patient
I or if facilities for Iaparoscopy are unavailable (experienced staff & instruments).
y' If the situation is acute, providing hemostasis followed by conplete or portiol
solpingectomy is the treotment of cholce.
2. Laoaroscoov:

It is the preferred surgical approach in a hemodynamically stable patient.


.l
=Less invasive,

- -allowing a quicker recovery & earlier return to work .trlill pjl;Jl tltrl...;p
-It gives a definitive diagnosis, although 4-8% of early cases are missed.
I Drsadrnnta{a
- Difficulty in controlling hemorrhage
- Needs special skills
- Some are not ideal candidates (obese, previous abdominal surgeries)"

$ Don't miss to inspect the other tube (unhealthy or congenitally absent)


$ Salpinsectomv:
H Involves removal of the entire tube on the affected side.
H Indications:
- Pr0servation of the tube is not possible
- Recunence of ectopic pregnancy in the same tube.
b Conservative surgery:
t Indications:

' If the situation is not acute (mild case)


' Only one tube is present
' In low parity
- Every attempt is made to conserve the fallopian tube, or part of it.
- Definitive reconstructive surgery should not be attempted at the time
of removal of the ectopic pregnancy l+ {.tr-r 4}ii.

! trpes,

Partial salpingectomy Fimbrial Salpingostomy Salpingotomy


(segmental resection) XX evacuation: )0( /

o Excising the involved milk the gestational a linear incision in The same as
6 portion of fallopian tube. tissue out of the the antimesenteric salpingostomy,
H

o
tube border.of the tube to but the
q) create a new opening incision is
H (stoma). reoaired with
fine
o Isthmic unruptured .The sac is near o
It removes a small monofilament
ectopic. to the ostium, sac in the distal sutue
o (apngostony n tltl o not done as it:
1/3 of the tube.
o nanow region resa/t u
u6 Jrecurrence points
o
nanowinfi
compared to 'Bleeding
r5
o Ruptured ampullary are cauterized
salpingostomy
H pregnancy in an
unstable patient Jpersistent o the incision is left
when fertility is trophoblastic to heal by Zry
needed. tissue. intension

,t.
Z'.

o Drain by posterior colpotomy br abdominal approach

Delivery of the fetus


Delivery of the placehta:
U If attached to unimportant shucture remove it
U If attached to important structure ) cut the cord short, antibiotics &
Methotrexate & leave for spontaneous absorption
! Methotrexate (50 mg / m2 or lmglkg) IM OR IN THE TUBE
(lf p-HCG doesn't .,1. by at least 15% between day 4 &7, or 2"d dose is given)

? Indicotions:

'::':Hiffi"
*

I ,Complicotions: stomatitq pfeunsftq I ftaerenzymes, pefucVan n lst


feu, trays
. Others
I Mefiprostone (RU486) ' antiprogesterone
, Actinomycin D
) PGFza (In the sac either laparoscopic or U/S guided)
! Anti D. if RH -ve
! D&C may be done to remove the decidua & . bleeding
s Later on after ectopic pregnancy rlsll lit l+ #-
n Treotment of Chlornydia
s ControcePtion: avoid IUD & POP
! Pregnancy ofter ectoPic:6oYo
s Recunrence:20o/o

l. Rudimentarv horn:
o The sac is media lo to the round ligament (tufatpregnaoq/ ts fateraQ
o Rupture occurs at 16 - 20 rveeks4js,ll c.lJrill drlt <-1'.
o Treatment: excision of the horn

2. Cornual (angular) pregnancy:


@ Occurs'at uterine orifice of fallopian tube
@ Late diagnosis 14-16 weeks with more bleeding
@ TTT is by wedge resectiont+r*o or hysterectomy lllc
d.sl {ut c-? lto

'---t;

=
Gtvic.L

o ther
3. Cervical pregnancv
\ TreotmenE
- hysterectomy (usually severe hemonhage) or
- Suction evacuation & to I bleeding
* Suturing at3,9 o'clock
* Silk sutures around the cervix (as circlage) with Hegar dilator in the canal
* Balloori tamponade by inflation of Foley catheter 30ml
* Bilateral uterine artery embolization by gel foam
- methotrexate local injection in the sac

4. Ovarian pregnancv
I Usually Zry totubal rupture
, lry is diagnosed by Spieg elberg criteria ,

- The tube on the affected side is normal


- The gestational sac occupies the site of the ovary & contains ovarian tissue
- The sac is eonnected to the uterus by the ovarian ligament :

I TreotmenE, as tubal pregnancy.

5. Heterotropic pregnancy
!- Intrauterine & extrauterine pregnancy
L Increased after WF (U4000 -1/7000) .
tr Treatmenti
- Surgery \,

-- Concentrated glucose in the sac


- Avoid medical treatment
+ Voginol bleeding is mild & not donoerous:
- Due to rupture or abortion & death of fetus
- This leads to decrease HCG with degeneration of corpus luteum
- This leads to decrease estrogen & progesterone ) withdrawal bleeding
t The decidua may fall in one piece called decidual cast (abortion e sL)
t Couses of obsent omenonheo :
' When rupture or abortion occrus before the missed period
' Usually in cases of isthmic pregnancy
Couses of obsent poin:
' disqovered too early.
If
Couses of obsent bleedina:
' When the corpus luteum is yet has not been affected
' : (very early detection before decrease IICG)
Do we remove ovoru with the tube :
' some say yes & some say no, but mostly it is no
a No due to:
l-Normal organs should not be removed
for hormone production.
2- The ovary is important
3-The other may be diseased later on
Yes due to:
A. To force the ovary to ovulate monthly with fpossibility of pregnancy
B. Migration may occur ) ectopic

Am...bl...pain
Usuallypresent
Colicky+/-backache' ::!' I Colic, dull, sharp J+5

Not proportional .Jillg4lrrl


TTR,RT .L.bFrl

Swelling + tenderness
BEXIGX HlLlGtAtf l.q, nrgr 4-#li]
Vesiqular mole Metastatic Locally malignant
: @ydatidiform mole)
t Choriocarcinoma s Placental site tumor
! Metastasizing mole s Locally invasive mole

I Definition
.i.

' Trophoblastic (cyto & syncitio) proliferation


' Avascular chorionic villi
' Hydropic (eF) degeneration of chorionic villi stroma.
I lncidence
* 1y'1000 pregnancies (p..lJt .rl3 ct {+r 1- pJtl
I Tvpes

1- Complete mole 2- Partial mole


lncidence More common Less common

Karyotype 46 (all paternal) Triploid 69 (extra set is paternal)t5r.9


Cause I 46XX (90% o .Bj- FrlJ I ) or u 69+XXY (70%o) OR 69+XXX (2770
(Andro- 46XY (10%) ) all paternal U due to fertilization of an ovum with

genesiso)
U Fertilization of an oyum with an an active nucleus by I sperm then
inactive nucleus by I sperm then duplicates or 2 sperms ) There are
duplicates or 2 spermg fetal tissues
Mac Uterus: large, full of vesicles (2mm - 2 cm + pedicle & filled rvith clear fluid)
Ovory: Theca lutein cysts (* 6 cm up to l0 cm, present in 5O%;o of cases, due to t
HCG so regress Lr "..lll after treatrnent by 2 - 4 m)
No fetus only vesicles Malformed (riploid) fetus, usually aborts
in midtrimester arlc &l oiLJ. + placenta
Mic o The chorionic villi are: oSome chorionic villi are: grt .rJtill-,
avascular + hydropic degeneration + trophoblastic hyperplasia
4 No. fetal tissues gThere is fetal tissue
Malig 5-10% 2-3%
change
Tro pho blas h'c prb ti tb rafti a

hy Jn pi , dejctrc mttn
Co,r. plc t< rno lq farfi'at rn

lf, ;:"'t *;LLrott


A*t nacl;*
,5t *t:
d Yunt
lfyil Aet acA'rrc. n relcrb

4b x((q ry (,o't)

6gxlx 2tt"
dl X{I 37.
6q YYY Jy',
1. Benign
2.Invasive mole (choriadenoma destruens): it perforates the uterus .i.e.
locally malignant (rarely metastasize)
3. Metastasizing mole: usually metastasizes to lungs & resolves with
treatment
4.Recurrentmolel-2%
I Etiology :
l< Unknown but theories:
+ A pUnory pocVta abnormality (error in fertilization)
* factors (| carotene).
I Risk factors :
1. Agq <20 or >35 (only in complete moleofx)
2. Nut.ritiorla I vitamin A (carotene) deficiency,
3. Socio economic
d More in the Far East {Taiwan 1/80} & less in west (I121OO)
d May be due to genetic factors. Now it is proven equal
4. Recurfenc
I Clinical picture :

* Amenorrhea * symptoms of early pregnancy


* Vaeingl Bleedine 97 %@ most common presentation
* Rarely discharge of vesicles ( qiagn
Pain:
t Dull aching due to distension
t Colicky (uterine abortion ofivesicles)
t Stitching in uterine erosions
t Stabbing in perforation
. tAcute abdomen in coinplicated ovarian cysts
* G0%) + {with no fetal movement)
* Others:
tEarly (severe) preeclampsia (but eclampsia is rare@)
t Hyperemesis gravidarum.
tt t Hcq
A.A.rV

o'
fata
, €xc?.y.ive eUa
enl-*1*mcal-
. DIe
. r?Ds
, .,11rt r
'J)5
r; !,rr
Ce,r Ji
n
" -l ^rbl

nla?,i+
, Veti clce
trll ,, F tt ccT
Ihv
,v
t- 1) i- CXR -B;oftf
{5tt ur t^f
ff l. - - K1 lcrccli,,y<rt"h 6oatra-cep1rd.
"ff
* Uyperthyroidism due to:
tF lncrease human chorionic thyrotropin
* Cross reactivity between TSH & HCG (similar a subunits)
Usua I ly biochemica I (1 T3 & T4) > clinica I
v Respiratory distress $ (embolization, PE, hyperthyroidism, heart failure)
v DIC
I Siqns:
fr-Generol:
+ Signs of early pregnancy
o Hypertension (PE, hyperthyroidism), shocked (severe bleeding)
g Tachycardia (hyperthyroidism, bleeding, anemia)
B-fr b d o m i n6l 61il[fi..p3 .LJor .
o Uterus
' DoughylU+c (vesicles + no fetal parts)
' Enlarged > period of Amenorrhea(50% of cases or lower than the
amenorrhea in 30% & corresponds in 20 %)
' Absent FHS, no fetal parts (except tf partid or twins ),
ballottement
+ Ovary: Theca lutein cysts
C-Voginol:
o Discharge of vesicles(diagnostic)
g No internal ballottement

3 As missed abortion, Diagnosis is by histopathology

lnvestigations r,qarg'

* U/S (almost diagnostic):Snow storm apPearance al'liltia€Hl


t Doppler: no FHS
I If Partial rnole cjlir*'Jl'lir dt-33 UI;_.jS5!l: focal cystic spaces in the
placental tissue, increase in transverse diameter of gestational sac
* Biopsy g@: villous pattern * avascular + trophoblastic proliferation
* HCG: > 100.000 miu/ml. It is more important is for follow up.
* Radioloev:
e Chest X-roy for metastasis in invasive mole or choriocarcinoma
r ploin X-roy : no fetal skeleton
r Amniogrophy : honey comb appearance+Lll & y1 tdlir Pll
* Coagulation profile. urine (ketone bodies in hyperemesis)
I Theatment:

A) Generql
g Correct the general condition: resuscitation
+ Treatment of PE, Hyperemesis, RDS
B) Euocustion of the moJe

/.Suction evacuation by 12 mm cannul2lq ar."l3


2. Oxytocin is only given after starting suction to I hemorhage
3. At last, sharp curettage to remove any residual mole
4. Don't forget to
' Send the specimen for histopathology
'
Anti D
5, Note thqt:
' Induction of expulsion by PG or oxytocin ) f risk of embolism
. D&C) perforation
q Hvsterectomv en toto:
a- When preguncy is not desired in patients > 40years.
b- lt doesn't prevent metastasis) need follow up by HCG
c- To I risk of choriocarcinoma (35% at this age) so give Methotrexate
d- Theca lutein cysts are not removed surgically except if complication
occur e.g. torsion or rupture
q -
a- Only in severe uncontrollable hemorrhage & partial mole with bleeding
b- Ensures complete removal but disseminate vesicles & Ieaves a weak scar

A) Chemotherqpy (J embolizotion, recurrence & locql involion)


o It is not in theroutine management of vesicular moleO@
o
(=critenio of possible development of choriocorcinomo)i
1) HCG : rising or plateau HCG, *ve after being -ve
2) Biopsy shows avillous pattern (=Choriocarcinoma)
3) Vaginal bleeding
4) High risk group: (single course at time of evacuation or hysterectomy)
1- B-HCG >100.000 miu/ml
2- Excessive uterine enlargement.
l- Theca lutein cysts > 6cm
a- Age > 40 years
B)Follow up:
1) Weeklytill -ve(<5mlU/ml )for 3 successive times
2) Then monthlvfor 6 monthso (l -2 years e$ l)s)
C)Controception:
@ OCPS till end of follow up then pregnancy is allowed (Baniers can be used
till HCG becomes negative l-ill r;l3,rf 4+J.s. (J". ). .
@ Avoid IUD(it causes iregular bleeding misdiagnosed as choriocarcinoma)

I Complicationte. :

Generol Locol
PIH <20 weeks 25% Hemorrhage & infection

Hyperemesis gravidarum 25% Perforation


Thyrotoxicosis 5% Malignancy (chori ocarcinoma)
Pulmonary embolism &DlC 2% Recurrence 1-2%

0 Natural historvrs. :
I) Spontaneous oborti on
2) lnvosive hydotidiform mole is the most common form of persistent GTT
3)Development of
* Risk factors include:
A HCG > 100.000 miu/ml
a Excessive uterine enlargement
a Theca lutein cysts > 6 cm in diameter
A Advancing age: older patients are at high risk

N.B: Indication of Hysterectomy en toto


1-Some cases of septic abortion (life threatened infection as clostridia)
2-Vesicular mole in old patient
3-Cancer cervix or cancer ovary with pregnancy

Q: what is D.D of partial vesicular mole ?


@ Twin pregnancy: in which one ovum gives rise to a normal fetus, the
second ovum gives rise to a complete vesicular mole.
@ Hydropic degeneration of the chorionic villi takes place with death of the
fetus (missed abortion).

)
S7#

Malignant tumor of the (chorion) trophoblast


^

7. Vesiculqr mole (50%)


2. Abortion & Ectopic pregnancy (25%)
3. Full tenm fetus (25%).
Any case of Zry PPH is considered choriocarcinoma till proved otherwise.
It is theworst Prognosis 6r" to late detection

A- Plocentol site tumor (rore)


. producerhuman placental Iactogen
111pL)
. Microscopically: intermediate trophoblast
' Treated by hysterectomy (locally invasive & chemo-insensitive)
B- Locolly invosive mole t (15% ofter molor evocuotion)
. Symptoms: vaginal bleeding
. Sisns: Uterine subinvolution rvith theca lutein cysts
. Diagnosis:
- Persistently high HCG
' Definitive diagnosis is by biopry (villi invading the myornetrium).

C- Choriocorcinomo: (5%)
. Tvnes.
- Gestotionol. after VM, abortion; Ep, full term
- Non gestoElonol: germ cell ovarian tumor
o Metastasis:
' Lungs (80%), vagina (30%), pelvic (20%),liver (10%) &. brain (10%).
- Characters of the metastasis:
l( May be larger ihan the 1ry, multiple hemorrhagic
lt Rrgr.r, spontaneously after removing the lry
l( In the lung it has a cannon ball appearance
Ge,rfa lbnal fuFhoblarh'< 17'

*yn alTnaa)
a l.

Ves; oila( rnol. Fo*dla.t Jit< T-


+Bragn
C-hari o CArrCiAotr.
+ invDiR
r Aefo'qht;Zin3
Gr.,,toh'oa..e-
horio Qanci /(

G^
( o(ar ran-l
dTT v
0
o-t
o\ $ (
q,?
{
t;o gI { ,{
-t,(-,:'
q
U a-a 2 a
d f [I J'
"L; r
B*ai1a t tl I I , I t ,l ,

aali1lr,of ll I I ,|-rltr
. Pathblogy:
A. Mac: ulcer or mass or intramural (-ve D&C)
B. Mic: sheets of malignant trophoblast + Langhan's cells * hemorrhage
C. Sp@: direct, lymphatic, blood (most common)
D. The cause of death is: hemorrhage & infections

I Prognosfic Criteria :
/- Ae
!- Antecedent pregnancy
3- Pre-treatment HCG
4-
J- Previous failed chemotherapy
6- Number. site: size of metastases
B.Low risk ! 4; moderate risk 5 - 7, high risk 8 or more
I EIGO classification
fill ore subdivided into:
Stoqe 1 tumor is limited to the uterrs
a. No risk factors
b. I risk factor
Stoge 2 tumor extends to genitol organs
c. 2 risk factors
Bisk foctors ore:
Stoqe 3 metastasize to the lung
g HCG > 100.000 m iu/ml
Stoge 4 other metostotic sites t Interval b.etween, the previous
presnancy & start or TTT > 6 months

I Symptoms :
o (VI\4, EP, abortion, full term)
o Voginol bleeding:
- Most commotl symptom
- Inegular & persistent.
- weeks, months (up to 2 years) after TOP
Amenorrheo ( increase HCG)
o Swelling: abdominal, vaginal, vulval, metastasis
o Poin: acute abdominal pain
- Invasion ofthe uterus
- Complications of theca lutein cysts
o SigFs of metostosis: cabhexia, hemoptysis, jaundice, I intracranial tension
P ro 7 sh'c crif,eri q
^o Jrr--t--II--Ir-Ir-r!
IOrlr tl
, r - r - - r r r+ r i Elr r - rI- r - rl - r
- -

;:#[:"*,r";*iu - J .#!-'*#",
-
-l-*.^-l -
--
-
-

I
-.-+-1-'l-5- -l-r=nl - tit ]!-i
i
lstortofchemotheropy :
T]
: :----:
----:
,o-t.J
a,__ t I tE II I r l- -
- ----t --- - -

1n'co
t
rtuiL) _ _ _ _ _ _1 -.10' I_r9-9o-l
-
_:rg _l - - - -

dineI .s lS-+cm I =5.m :


i I t r I r r
-- - - -
--t
- - --l -
:

'' j -,;,,r-! -
s--.;
-!?t,t,-l *a'!
r r kldne1 ltntesttnat! [inoer
-l' -i-
r -- - -l-o--; - -rE- -l
;

J riril6.-r
-"rT"-."1G"-r -
J -]
-
Ip'i;;e-,,ilir,2,i;t
--
I I -:_ _ - _ _:- y:ra=;_ily,-i

[n6 Ju^h {;sk 5- -1

H;7x /isk >8


o 6enerdl:
- Anemia, cachexia
- Hyperthyroidism (chorionic thyrotropin & TSH like action of HCG)
o Abdominol exomination: f uterine size, theca lutein cyst, masses
o Voqinal exominotion:
- Metastasis: (highly vascular) in fornices & sub urethrs a.r4-r gt uyl
- Enlarged uterus
- Theca lutein cysts of the ovary
lnvestiqotions:
o To screen:
- Very high HCGUp to millions (also diagnostic & for follow up).
- It is betterthan D&C
- May be -ve???
o To dioonosis:
. D&C:
. Should be done for every case of bleeding after TOP
. Sheets of malignant trophoblasts = avillous pattern
o May be -ve in intramural type (Mil & Doppler con detect i
- Biopsyfrom any metastasis
o To know stoge:
- Endoscopy, X-ray, CT, MRI: canon ball in chest
- Broin metostosis MRI & t HCC in CSF
- frodio immuno locolizotion
o Preoperotive
.4

I Follow up

o HCGt
- Plateau or rising HCG
- Becomes *vg after being -ve

I Voginol bleeding
o Curettoge'l
Pre TTT investiootions:
o CompleteH&Ex
o HCG level, CBC, RFT, LFT
3. Low Risk: <4 (good prognosis
)90 - 97% cure rote)..
o Methotrexate (50 mg IM) alternating with folinic acid 6 mg
o Actinomycin D (10 - 13 Mg /kg /d IV for 5 days)
4. Hioh Risk: > 8 (poor prognosis >75 - gO% cure rote)
ide)

5.

6.
t Stage I -; when pregnancy isn,t desired
t stage II. III ;, adjuvant to chemotherapy (j dissemination of tumor)
! Resistanceto chemotheraoy:As in Placental site tumor
S
7' Excision of locolized rnosses {Thoracotomy, laparotomy (hepatic resection),
craniotomy, vaginal masses)

OCPs during follow up.

!. Avoid fUD

S : resistant or severe hemorrhage


t as it interferes with the delivery of
optimal chemotherapy
r___r_

especially with the oral route.


;
__ ___J
, Definition:
a Bleeding from the genital tract occurring
o after the age of fetal viability (20 128 week of pregnancy) &
o before delivery of the fetus
? Couses

A-Obstetric:
Maternal Fetsl
Placental Extra-placentel Vesa prevh
1. Abruptio placentae l. Runtured uterus [RUl (The only cause
$ccidental .hemorrhage ) Excessive Show:
2. benign ofAPH of fetal
Commonest cause condition with no origin).
2. Placenta Previa complications & the diagnosis
'
(nevitable hemorrhage I is by exclusion ofother causes.

B-Non Obstetic
Local genital causes as cervical erosions or carcinom{incidental hemorrhage)
^

Definition :
i It is bleeding from the genital tract after the age of fetal viability 20128 weeks till
before delivery of the fetus due to implantation of the placenta in the lower
uterine segment below the fetal presenting part
Etiologv unknown btrt theories
' Deficient decidual reaction in UUS
- More in MP, old age
- Pathology as scars, tumors
' Enhanced hrbal motility
' Abnorrnal placenta:
- Delayed appesrance of chorionic villi
- Deliyed dlSa1ip""."hcA of Zona pellucida
- Large placenta (twins, DM, PHA, RH, placenta membranacea )
- Persistence ofvilli in decidua capsularis (chorion leave)
' Recurrence: (4-80lo)
Ca*rfes oP APH

fc fal.

futrt-
,M

K,t fetot,
floctolal

6uoin tkw

lor^7
, TyPeS 4-r{ra.trrLatl ,l-a!,'yl

Lower Placental edge is in the LUS but not


I Low-lying placenta 60%
reaching the internal OS
Marginal placenta previa 30% The placenta is at the margin of the (l.OJ
1 Partial placenta previa 7% The I.O. is partially coyerd by the placenu

4
Total placenta previa 3o/o The placenta covers the cervix totally

o Palhoaenesis
$ Placenta is inelastic (cantstretch) so bleeding occurs during
t-Preenancy,:
gDue to stretch of lower uterine segment (shearing effect)
gBleeding is severc as thc LUS lacks the o can't
compress the torn vessels.
gPeak incidence of bleeding is 30-34 wks @

gFirst bleeding episodes are usually mild o


2i
gDue to cervical dilatation.
gRarely may occur for Itt time in labor (rv moy leod to fotal hcrnorrhoga).,
o Symptoms
rr.-eBleedins
+ o cgntpctions) except a if th labor-pain.
.i due io shearing effrict bn f the LUS, except if occurs
after intercourse or PV "'l'.'

* Recurrent
j It is recurrent because the formation of the lower uterine segmcnt
occurs along the 3'd trimester, except:
. If placenta is just reaching the LUS or
r Labor occurred, or
. Severe hemorrhage ending either by termination of pt'Egnancy or
. patient death.
i.
+ ce st !!4s weeh
Z. Nq pain
a Collaose
g.+
S7r--zr-\i-r-t\:z%

Signs
. General Related to the amount of hemorrhage (anemia or shock)
r Abdominal examination:
* Fundol Level corresponds to the period of amenonhea (f iu DM,
twins & J in IUGR or transverse lie)
* Fundol Umbilicol & pelvic qrip soft uterus, not tender, non engaged
head, malpresentation

ln placenta previa, Malpresentations are common (30%)

. P/V: Contraindicated
+ Except if the patient is in labor & minor degree is diagnosed by U/S
+ Aim to determlne the possibility of labor
+ In the operating theaterrvhich is ready for immediate interference by CS +
available blood (even under stream ofblood transfusion).
+ This is called a double set up technique (with 2 teams; one team for
examination & available one for immediate CS)
c Placenta if felt will be a fleshy spongy mass
. Auscultation:
. Well heard {normal except in severe cases (major degrees) in which more
than l12 of the placenta is separated).

' Stallwothy sign: bradycardia when the patient is allowed in labor with
placenta previa marginalis posterior (placental compression)

t fetal complicotion
i'. PNMR is 5% especially in prematurity
'i' Due to poor fetal blood supply ) IUFD, IUGR, Prematurity & Neonatal death
* Some reporfed CFMF fdl +tis
o Allotemol complication

b-During pregnancy
t
Preterm labor (spontaneous or induced)
tr. Malpresentations & Non eflgagement.
.! Antepartum hemonhage & anemia or shock.
c-Durin&labor
r lststase (4p)
? Prolonged labor (intrapartum hge ) inertia & malpresentations)
t Large bag of fore water
PROM
'? Prolapse of the cord
al?ra
^!
Jt,t"i-Aufp('

?PH

Purffu/|ur\ 4 S
a 2nd stase: obstructed labor
I 3rd stage:
$ lry PPE 0" 24 h) due to
1- Atony due to poor contraction of LUS + poor maternal
condition
2- Retained parts : placenta accreta (due to poor decidual
development).

3- Traumatic due to friable LUS (wet paper LUS)


! Air embolism as the placental site lies near the vagina

d-Durinenuetuerium (4S)
) .'
l.Puerperal Sepsis due to
I Poor general condition, retained parts, placental bed is near to the
cervix, PROM and t infection due to surgical interference
2.Sub-involution (retained parts, bad general condition, infections)
3.SecondaryPPH (from 2nd day to 42nd day)
4.Secondat' anemia

o- lnveslioation :

O U/S (the only & best methqd used)


* Hish accuracv (98%) (Diagnosis, degree, accreta & associated conditions).
* Placen,tal misra,tion: growth of LUS >UUS) uppurrntplacental migration
with disappearance of the placenta previa or J degree (serial US /2wks)
* Lorv lving placenta is diagnosed in 45 Yo of 2nd trimesteric pregnaneies; of
these ) 90% disappear (so incidence is higher at early gestational age)
* Some sav that TVUS is better as it determines the exact site & relation to
cervix but PV is contraindicated

O Speculum examination r*-: to seek vaginal or cervical lacerations

$ 0therl+ oy*-
* Blood grouping, cross matching
* Kleihauer - Betke Test & APT to exclude Vasa previa.
a
xV
HPluin x-raY :

E Sqft:.QIacental shadow
E hard
/ Placental calcification after 32 wks
/ Degree of displacement of presenting part &. pelvic bones
HContrast X-ray: cystography & barium enema (degree of displacement of
presenting part & bladder or rectum), Amniography, angiography.
Hlfnt : very accurate but expensive
HThermography : more temperature over placenta
HRadio active isotopes

oTreotment

7 lf the patient is not in labor &


r No fetal distress &
7 Immature fetus &
r tvlild hemorrhage
{. Aim: keep the condition under control until fetal maturity
2-Horv6ljl-,
I Site: in the hospital till delivery
, The patient:

, Fetus: fetal wellbeing, corticosteroids for t fetal maturity.

It PV examination is contraindicated in:


I- Any bleeding with pregnancy
2- Virgins
3- ROM before 36 weeks
4- Ectopic pregnancy (t the disturbances)
5- History of contact bleeding
l-When
I If the patient is in labor or fetal distress or mature fetus
? Severe hemorrhage
2-IIow
I Resuscitation d|4i:ll-t cist + consent hysterectomy + blood preparation
r Deliverv either
)( Vaginal eL3:-)11 ) AROM * oxytocin
? Only in lorv lying placenta rvith mild hemorrhage (also in placenta.

. No other indication of C$ as fetal distress or malpresentations


. 1" stqg.e: continuous fetal & maternal monitoring

" ;
"
: T:,,
'
fu*,;il*ril t##",,
weakens the patient but PPH kills)
: ;

)( Cesarein section cr'cliJl g:;

O LSC.S is thi rule:


' Better control of placental led
' Leave strong scar, allow vaginal deliver next time
'. ' J l.ncidence of placenta accreta (the placentd usually implant
, on UUS next pregnancy)

O If placenta if found a.nterior:


* llncise the placenta & deliver the baby through it
'! Or reach round it till head is felt
o
' ) Ecbolics + massage+ hot packs
. ) Under-running sutures
) Cautery
:) Balloon tamponade on LUS
') Uterine artery ligation
) Bilateral internal artery ligation
) Supravaginal hysterectomy if all rneasures fail

)( Supravaginal hysterectomy
UII
I Care of the neonate
c Definition
*tt genital bleeding after the age of fetal viability 20128 weeks, till before
delivery of the fetus due to premature separation of a normally situated
placenta
o lncidence
* 1/200 - l/500 (commonest cause of APH O@)

ldiopathicis the commonest couse of accidento{ hemorrhoge


1, Poor decidua
a Multi-parity, Advanced age.
a Pathology in UUS septum or f,rbroid or scar

I Circumval I ate placenta


I Recurrence:
i One attack ) 5-11% &
& 2 attacks )25%
3. Blocd vessel iniurv:
I Smoking, r! $ vit C & vit K, alcohol
Folic acido,
I Short cord, Sudden decompression of the uterus as ROM in PHA or
delivery of the l" twin & Trauma (As accident or external version)
' PE (most important),DM, Obstruction of the cord or IVC
o Types
@

* The lower margin of the placenta separates and the blood trac.ks dorvnwards
(between the mernbranes &uterus) to escape through the vagina.
* The very mild revealed is called mrrginal sinrs hemorrhase
@ Conceoled hemorrhoge (10%):
t Severe hemorrhage doesn't escape externally due to atony of uterus,
adherent placental margin, adherent membranes or well-applied head
on cervix or intraamniotic hemorrhage
@ Combined or mixed type (60%):
* Usually starts concealed then become revealed
APH
altl;, I Jelitey
frt-t
)
+- ' ""
ftenofir' /b!'rof;'
a ilo rnoCh f''fiateJ '''"
/

Dec,Jr'*
V*
o Pothologyet4t
t Hemorrhage into the choriodecidual space ) separarion & dissection of the
placenta

! Consumption of the clotting factors by the hematoma ) DIC + FDp


tocolytic, anticoagulants & fibrinolytic atonic PPH & renal failure.

! The blood dissects the myometrium & leads to subperitoneal hematoma that
may rupture) intraperitoneal hemonhage & end to rupture uterus (Couvelaire
uterus = black eye uterus = uteroplacental apoplexy)
o Clossificotion
Closs
r LrgJJ
I i
l OP '.
a Fetus
avavJ a Shock
J..vra, iDrc
a vrt

r.,.,,. 0;-F:;;:-r;;
a

;=-tt:::-;-,-]:-l
l- a-a-r

:_-_;_:-1,-
|
I
! mildest I It
I
the
's asymptomatic, diagnosed retrospectively by !
! presence of retro placental hematoma I

1
Ctoss t 2 luld i Living !
-ve I
ic I

I Closs ll lmoderate ! Distressed t I -ve I -ve I


I I t- |
-i-evere- t

i -
i--Derd--l---ne
-:-sd"$- -i- -D;ad- - -
-
- -l' - -ryr- - -l i--ff---i
i
L----g --J-
l-!-- il*- ---t --__J
o Svmptoms
A. fraveolad;) bleeding +/- paln
B, Conceoled:
1) Bleeding (early absent)
2) Pain (sudden, severe. continuous abdominal pain)
3) CollaEe (hypovolemic, neurogenic due to tonic uterus)
o Sians
A. General:
freveoled; according to amount of blood loss
Conceoled',
'f' Shock may not conespond to the external bleeding, (hemonhagic +
neurogenic)
'!' The cause may be found e.g. PE (but blood pressure may be
apparently normal i.e. hypotension due to shock is masked by
PE)decapitated blood pressure. so hypovolemia is better
detected by central venous pressure & UOP monitoring
* Signs of complications as DIC
t1,4b:)t JtLl ,

.tfifier)nr.
y'iucln
(/ Y
,/-
, *
\t Gu,6"inio u/4r'*'
o t;7
'or'
But r (/-
s1

9,'frmni^oh t fril ernb'tlun


B. Abdominal examination
* Fundal Level
s freveoled: Corresponds to the period of amenorrhea
; Conceoled: higher than period of amenorrhea
.f. uscultation
; Beveoled: - Well felt & well heard
s Conceoled: - Less feltLtr+ii. FJll & heard (died or dying) * tenderness
& rigidity uterine basal tone i.e. board like).
C. P/V:
Controindicoted unless ofter €xclusion of plocento previo by US.
) Done for vaginal bleeding (dark, clotted & slight), cervical dilatation,
presentatioa, position, engagement & tense membranes (if ROM )
bloody liquor ) port wine)

. Fetal comqlicafion,)s.?sb. ;
IUFD, IUGR, Frematurity (spontaneous or induced), Neonatal death
Perinatal mortality rate 30Yo

o Moternal complication?s. :

2. : lry (atonic, traumatic & DIC) &.2ry.


3. Couvefaira rrferui (Utero-placental apoplexy): extravasated blood into the
uterine muscLlature & beneath the serosa lead to impairment of the uterine
contractility after delivery) prconhactile uterus ) severe PPH
4. DIC (Abruptio placenta is the most common cause of DIC)
5. heehoa sYndrome.
6. Shock
7. Pre renol foilure: DIC & microthrombi obstruct the renal capillaries, PE,
overdistension of the uterus leads to spasm of the renal vessels (Utero -
renal reflex) * ineonpatible blood transfusion.
8. Rupture uterus.
9.
10.
o lnvestigations-+(diagnosis is mainly clinical) :

1. us:
rto To exclude placenta previa
* To visualize the retroplacental hypoechoic area (Hematoma)
Rule out IUGR
2, Speculum examination: to seek vaginal or cervical lacerations
3. Investigations of the complications e.g.PE
4. oth..t
Fibrinogen, CBC & RH
a) Blood grouping, cross matching, PT - PTT -
b) Kleihauer - Betke Test & APT to exclude Vasa previa.
c) placing a specimen of blood in a tube

t If blood clots < l0 minutes g Normal


y If blood clots > 10 minutes ) DIC

o Theatment

a
A. As placenta prevra
B.

I Conceoled

. Bleeding is uguolfy severe + f elollmoternol distress + the potient


is usuolly in lobor (tonic utarus)
. ReSUSCitotign cJ#4illl+ ,.,i(ri3 44+

1- Oxygen
2- Restore circulatorv volume
Trendlenberg potition
;
S 2 wide bored cannula
p Fluids
\ Colloids as albumin 5%

\ Crystal l oi ds as NaCl 0.9%, ringer lactate

\ cross match lor possible blood transfusion if bleeding is


brisk or if the initial henioglobin is less than l0 grn/d1
3- Drugs as dobutanrin
4- Eliminags the carlse
5- Reass6ss the patient: HCV > 30%, UOP > 30 rnll h
r Terminotion Gfter correction of the coooulotion
ects
1 Cesarean section:
9 If the fetus is alive & very rapid delivery is needed
9 Correct I't DIC
* Vaginal deliverY oulill or:
9 If dead fetus + dilatecl cervix especially if DIC is present
9 'Easy (well-engaged head + t basaluterine tone).
9 Precaution: antishock measure + imprOvement of general
condition, close maternal observation, arom (relieves pain + l
intrauterilre pressure,?PC.|AF embolism & Couvelaire ) +
Oxytocin. observation of UOP, delivery not.exceeds 6 hours to lrisk of DIC

9 lst stageiontinuous fetal & maternal monitoring


o 2nd stage:usually rapid
9 3rd stage:guard against postpartum hemorrhage

! In Couvelaire uterus or atonic PPH (after failure of conservative


measures ) look placenta previa)

1- Sl,r--ock: Antishock measures ( dr+ll ji;1 ;


2- Couvelairg uter-us: Ecbolics if failed hysterectomy
3- Rupture- of -uteFUS.t fiyslerectomy
4- D-lC;
r Treatment of cause by delivery of paticat & removing blood clos
. Cryoprecipitate, fresh frozen plecma, fregh blood.
o IV fluids to help kidney to wash FDP
. H.eparin is conlgaindicated
5- fost:partum hemoruhage
6- Renal failure
o Treatment of cause as shock & DIC
o [f not enough forced diuretics
o If not enough hemodiaylsis

Treotment of the couse os PE


Care of the neonote
indicotions:
g Mild bleeding. the potient is not in lobor
o fmmoture fetus
o No fetol distress

o bed rest, good diet, sedotion, steroids &


I observotions:

- Moternol:
d US is important to assess the size & f of the hematoma
r Fetol well beint

1-causes of acute abdomen in late pregnancy (concealed or mixed)


2-causes of antepartum hemorrhage (revealed or mixed)

Mixed accidental hge


o Once, sudden o Recurrent
; History o llas etiology . causeless
0feedingl o Painful . painless
o Dark clots . fresh blood

oExamination o Etiology e.g PE . No etiology


o Shock > hge .
* General o DIC Shock: hge

.f. Abdominal
o Tender . Painless
o Hard o Soft

o No placenta o Not done


{. Vaginal o Cephalic r Placenta felt, Mal-
o Well engaged presentation )not engaged
o Normally situated r Placenta is in LUS

), ,>
,\-i. ;
, lrerge
I ttz
rane

Hb Vo 10-12 gldl l6-18 g/dl


Blood groups & RH May be different May be different

Microscop-1' Non nucleated RBCS Nucleated RBCS


Spectro photometry HbA TIb F

Kleihauer-Betke test
Ghost cells No hemolysis of RBCS
(acid elution test)

APT test ?(alkaline Blood + equal amount of. 0.25% NaOH

denaturation test) Brownish Remains pink


o Etioloay !
I. "commonest in the developed countries,'
2. Acguired heart diseases 93%
* Rheumatic heart disease (l!'lAT especially MS) "most common in
developing countries"
* Marfan syndrome (rare)
3. Others: 1%
& Arrhythmias, Ischemic heart disease & Peri-partum cardiomyopathy

t New vork heort ossociotion INYHAI Clossificotion tJ1ar.+g

o Acccrding fo the functioaol copocity of the heort


Grode Symptoms Degrx of comptomi*-
1 No symptoms Uncompromised

Mild discomfort on ordinary activities Slightly compromised


2
A-mild limitatiorl B- moderate limitation.

3 Marked discomfort on less than ordinarv activities Markedly compromised

4 D,vspnea at rest Severely compromised

o Drow bocks of NYHA:


7-
2-
t Moternol adoptolion in the C.V.S o*-l'$tr,ldlriin t

Veins
. Voricose veins due to:
- Progesterone relaxant effect on vessels
- Pressure -hy,gravid uterus
- J Blood volume

a Volume 40-50% (max. 34 wk t,r r^g)


a

(hemodilution) & J viscosity. Pathological if less than 10.5 g/dl.


tWBC's t 16,000/cm3 (more marked peripartum)& ptatelets >> mildJ
- Class;falion oF tleer/ d;teans

2 S - so'r,
**+.y
, (lorh c sfenosis wrll
shao lic,J;lalafoa
Tosl
, prorlltfic
.7ul^on,r1 ltTxt vrlrcJ
'QSDTYJD'?Do
. fo l(y gr', J;rellr2, . flortrc
f,ltacti r
uitl fforh'c Jlclrhzn
. l'lrr Fra reriltr,r re.l
aeeFc J;1.

t f{ahrnoL ala/afiti,r in CoVo [n )rd.,_,

r' I
crlancsit

BhoJ

('rvp"llatntecit
{"r,
| 'hSaia J?tultltut l

yl;l/;q oF s,
a
lESR (5Omm lst hour due tot fibrinogen & J viscosity)
o
f Clotting foctors ('7, 8, 9, 10, 12 & fibrtnogen ) DVT) &J factor I I &
I fibrinolytic activity
. Minerols & vitomins t.t-q2i.eg'
- There is t demand for iron exceeds the amounts available in a normal diet.
- | transferring (total iron binding capacity)
- tcopper & ceruloplasmin (estrogen effect).
- JFat soluble vit & I water soluble vit in blood.
> 4pex shifted upward & laterally (4th ICS instead of 5th )
> Pulse :
.1lo-l5b/min
o Heart sounds:
Splitting of I't heart sound
Appearance of 3rd heard sounds
o Murmurs:
Soft systolic murmur
Diastolic murmur if occurs we must exclude pathology
i Blood Pressure +tt{rJi,
o J fn 2nd trimester
) Placenta acts as AV shunt
i Progesterone, estrogen & prostacyclin vasodilator effect
o fln 3rd trimester (t blood volume)
> 9ardi?c ggtput s@
o 30- 50% max 24-28 wk due to f stroke volume & heart rate.
o During labor (uterine contraction & loss of placental shunt).

Prognosis dep.end.s gn
9 Age, parity, the functional capacity of the heart, previous heart failure,

4. Personal history:
e t Aeer& Paritv ) the worse the prognosis
o Occupation ) advise against marked physical effort
g Address ) rheumatic HD more in dumpy non sunny areas
o Smokins ) must stop smoking
1P\/C: cough, expectoration, hemoptysis, dyspnea, orthopnea, PND
.1SVC: congested neck veins, Rt. hypochondrial pain, ascites, L.L ederna
.1Low cardiac output: fainting attacks, fatigue
l,Infective endocarditis: fever, Rt & Lt hypochondrial pain, HF,
Hematuria
4.Rheumatic activity: carditis, arthritis, SC nodules, erythema, chorea
.;.Arrhythmia (palpitation), cyanotic heart disease (cyanosis, malar flush)
& heart failure

; for dating
previous HF in pregnancy

\r' Medical: attacks of HF, and duration of heart disease


r'\
\r' Surgical: valve replacement
t\
\l
l\
Drugs: long acting penicillin, anti-failure measures or anti-coagulant

Exominotion:
1. General examination:
a Consested neck veins (not reliable due to f blood volume)
A I,L edema (may occur due to pregnancy or PE)
A Enlarsed liver (may be difficult to palpate due to large uterus )
A Normally we can see signs of hyperdvnamic circulation as water
hammer pulse or capillary pulsations

Palpation:
) Apex is shifted to 4th space
) Thritl
Auscultation:
) Heart sounds: Splitting of Sl & appearance of 53
> ffi.rfilrie.orld h.u.
a Splitting of the l" sound
a Appearance of the 3'd sound
A Soft systolic murmurs (<216)
i Shift of apex beast from 5th to 4th intercostals space

3.Abdomen; IUGR, IUFD, and polyhydramnios


F
I - - -
f
-
I rr--.1
- - -
ic heort di ose & normol?
-

l: Symptoms
t D.l'spnea, LL srvelling, palpitation fainting attacks & malar flush I

l, Sions:
t Generolr I
I lower limb edema, congested neck veins, hyperd,vnamic circulation
!
I Locol
I
r Splitting of the l" sound & appearance of the 3'd sound
r Soft systolic murmus (<216) I

J
Sl Investigotions:
9 CXR * abdominal shield: cardiomegally
I ECG, ECIIO
9 Cardiac catheterization: if surgery will be done
o Investigations of rheumatic fever: AS0T, CRP, ESR
9 Assessment of F\VB
o Complicotions
@ Effect of pregnoncy on cordioc diseoses
A. Anhythmias. Rheumatic Activity (recurrence of rheumatic activity is rare but
serious if occurred)
B. lacterial endocardit'is (lEC after any procedure esp in pg,erperium)
C. Incrsased Cyanosis ,in cyanotic heait diseases
D. Decompensation (Heart failure & deterioration by 1 grade)
t Durins presnancv (28-34wks) due to:
. Max 1 in COp 30% (due to f blood volume * J peripheral resistance)
, I Blood volume (1 40.507s)
. COP = Heart rate (f 10-15 b/m) X stroks volume (l)
\ Durins labor:
. l't stage: pain + uterine dontractions ) lVR to heart ) 1C0P
. 2nd stage: as 1ttI bearing down
. 3rd stage: loss of shunting effect of the placenta (return of'500 mlblood
in the ,uteroplacental circulation, to general .circulation after
separation)
E. Embolism due to increase stasis & J clotting factors
'!S- tsg rS- lsg- rS- IS; s- S- :s- liri s- asg lsgz
S7*

i. Fetal
o IUGR, IUTD, abortion, prematurity
I
ii. Maternal
I Abortion (in seVere cyanotic heart disease)
I Prematurity (small fetus & soft cervix)
I polyhvdramnios (due
to congestion)
-l PPH @ypoxic myometrium + ergometrin is contraindicated)
I Puerperium) + S
. ltllonggement
@ of preqnoncy (in lst trimester
only'l + sterilizotion ifb-b.
1. Degompensotion:
I Grade IV & Grade III if completed her family
:aDrervious HF during pregnancy
Cardio-myopathy
I' History of rheumatic activity /lEC in the past 2years
2. Outflow obstruction:
I Tight Aortic stenosis
! Pulmonaryhypertension
I Coarctation of Aorta
3.

4. Collogen heort diseose: Marfan $ with aortic dilatation

@ Preconceotionol monooement

' Surgical conection if ueeded before pregnancy


@

cordiolooist).

* In special antenatal clinics (obstetrician + eardiologist)


* In the hospital at (30 - 34 w) then to plan labor (after 36 w)
b) When,
* EveV l-2 weeks till28 week
* Then weekly till36 week
* Then hospitalize
f Q Jae+te
Surnmdl
rOhr."
Pre..^oe0il fl^le ftqt.aL- nqb'q, L PosF naaoL

t Ut *t' if 6.ro _*;l


Sf jEoe x # rt5 - 6os
tol,-,wilt hotf.x - obs
- fedtrt /
---'
A.

C5
VD OC
(run)

,a
c) How
r Bed rest (10 hrs daily)
; Diet "f salt", weight gain should not exceed l2 kg+.a.
; Sedation , steroids to enhance fetal lung maturity.
I Drugs:
. Pneumococcal vacciry ) chest infection is the most common
cause ofheart failure
- Benzathine penicillin 1.2 million rul 2 weeks or daily dose of
peniciLlin V or erythromycin
- Class III & IY: Hospitalization & TTT of heart failure
A: analgesics "morphine"
B: breathe O2by IPPV
C; catheterization of pulmonary A
D: drugs, Diuretics, Dilators & digoxin

@ Intronotol monooement:
l< When:

-o Class I & II: leave for spontaneous onset of vaginal delivery (induction is
hazardous)
o Class III:
-- "-
a If Completed her family ) better to terminate (l't trimesteric only)
. Ifnot completed her family: continue the pregnancy in the hospital
o Class IV: control the heart failure then terminate
o Termination of pregnancv is only Itt trimesteric as tennination of a 2od
trimesteric,prggnancy is more,hazardous than continuation of pregnancy.

l< How:

$ Collogen diseoses as Marfan$ if having dissecting aortic aneurysm


$ Outflow obstruciion: Aortic Stenosis with post stenotic dilatation, lry
pulmonary hypertension, coarctation of the aorta
$ Cyonotic heort diseases as Fallot & Eisenmenger $
$ Associoted indicotions: fetal or maternal dishess,. prolonged labor,
placenta previa.
-I
I in cordiac
G 1st stagg I
r Semi-sitting + no bearing down
r Oxygen + antifailure treatment if needed.
I' . Epidural anesthesia except in pulmonary hypertension, aortic sten &
cyanotic heart disease (morphine 10 mg is better than pethidine 100 m no I
I tachycardia).
I
1.
i
II
piciltin IV or IM + 1.5 mg /kg gentamycin IM
I dose of aripicillin g hrs post_partum I
0 min in the 2nd stage.
I

24 breoth/min ) heort foilure)


ellbeing
I
I G znd stage: I
! g F_orceps or ventouse delivery can be used to shorten the 2nd stage
o Usually easy: small fetus + congested ripened ceryix I

G 3rd stage:
I
I
{ Avoid ergometrin fheart load due to VC + strong uterine contractioris &!
may t anhythmias) )
0,25 mg IM can be given.
I q Frusemide m?y be given I
I Guard against PPH
L - r rr r r rr r r r r I
-r ---
- of
3 Core neonote

o Loctotion:
, Breast feeding is allowed unless severely compromised
g Controceotion:
| ) thrombosis
lUP ) can cause ascending infection (cut threads short * aseptic
insertion + prophylactic antibiotics (categ ory l&2).
Best ore: mechanical methods, sterilization or POP
^Jy2z:j

t25nL/1,

. rih"la rluq
dllessMeAl-
oF fqB

,D qoayetiq
JooJ
. Lasi K
. ?pi lvtol Ac) '"
' (fre. /antic'"1')
LAtif

0- c {
U
o
U
L
o
0-
E
{
s *.
q {
0
I
t-
.1 i; L
(
6
I t tl I I I
CoafL rl I I I I I I

-trrLoltq q 1 l I I I ltl
Low risk
mitral valve Rheumatic heart disease Prosthetic cardiac valves
prolapse HOCM Previous IEC
Congenital heart Mitral valve prolapse * Complex cyanotic heart
diseases ASD regurgitation or thick valves diseases

t. Open heart is better postponed after delivery


2. Closed mitral valvotomy : can be performed in 2nd trimester if cardiac
catheter proved tight mitral stenosis (has little risk for the mother & fetus)

3. Valve replacement with pregnancy


o Tissue valve needs no anti-coagulants
o Metollic volve needs anti-coagulants
7 lst l2w: heparin 5000 ru SClSh for fear of CFMF
p 2nd & 3td trimesters:
- Worforin (marivan Smg) orphenendione (dindivan 50mg)
-Till 36th w or 2 -3 week before tobor shiftto heparin
- Stop heparin with theonset of lobor
- Heparin is resumed4-6 hr post pcrtum "the risk of PPH is gone"
- Shift lofer on to marivan (dindivan is contraindicated during
lactation)

Some give heparin all through Some grve oral anticoagulant all

x The risk of over or under control by hep


more serious than the minimal rec
o Doesn't cross the placenta,
9 Has an antidote (protamine sulphate)
risk of fetal affection due to
anticoagulants.
o Short acting (2-4 h)
Disodvontoges \ frntidote:
a
'Higher
risk of thrombosis, - FFP (rapid) or
a Thrombocytopenia - Vit K (safe for both mother &fetus )
! Osteoporosis (use calciparin).
a Na heparln is not stable at room
temperaturels)^:Jl sf Eis+ fiY
qb
lSl. Thrn . 1 trl Tri Lo b or
Hrfdrit\ No thiy1

ofitet of lalr,r

Mol tUt

Anti-thrombin Anti vit K

Onset aftor 3 days for 3 days

effdcts

. Optic atrophy,
. Chondrodysplasia punctata
ti:r;l'r\'- ' ,:. '. i trria'.

Side effects All ore less with tMWH


(hemorrhoge, The moin disodvontoga is their high
thrombocytopenlo,
S7r--zr---

1-Mitrol volve prolopse


.! Patholoey:'Myxomatous degeneration of one or both of mitral valve leaflets with
prolapse into left atrium during Systole.
9 C/P: Aqymptomatic mainly, or palpitation, dyspneq chest pain, syncope
! Treatment during labor: Controversial (most do not give antibiotics orcept if
associated with miral regurge or thickening of the valve)

I Definition: dilated cardiomyopathy >> HF in 3rd ftimester /puerperium


I Etioloev: unknown but may be viral infection, autoimmune
L Pf edispo sin g factors : hypertension, twins, genetii predisposition
I Prognosis:
Mortaliry in 25-50 %(tatal)
Mortality in next pregnancy is 80%
I Treatmer,rt:
Hospitalization.
Digitalis.
Decrease after load ftydralazin), decrease ppload (diuretics)
Heparin (risk of thrombus formation in dilated heart, it is ttre cause of death)

3-Coorctotion of Aorto
Definition: hypertension only in upper limbs, normal Aow pressure in lower limbs. It
*ay b. confined only tb left arm (coarctatiori of left subclavian)
Route of termination:
@ Vaginal delivery allowed
@ CS only in other obstetric indications

4-Morfon syndrome
' Etiolow: defective connective tissue.
l- Clinicd picture
@ Mitral valve prolapse
@ Mitrd valve incompetance
@ Aortic dissection (intimal tear) acuie chest pain +shock
' Route of termination is CS ' /
oF i,rfe"lion
o C/p Lr c-
-l'
e

{.)

auH
I

( JoP.
hr
r l,ahur
o Avoid qI
e cla

-t/o,nr.iliac I
?,u n'/ kn
- Iealfopria
- Br.tlGarl;rf B5c-;
y* * sloT Jig + K
, fABr phtqtoi
3- 0*;oLFar;tteh'on
Achin o€ ddgi!p:*
+rcinctruei< 4- G'J;'^/'f"Y
I sizc o?rte hc'rt
-coP
tF!iuresis
ve
lcout"t +,
L Obstetric conditions
. Morning sickness, Hyperemesis gravidarum
. Acute poinful conditions:, disturbed EP; VM, pE, pHA, pyelonephritis
2- Gynecological conditions as twisted ovarian swelling or red degeneration of fibroid
3- Medical conditions: Liver diseases as acute fatty liver, food poisoning
4- Surgical conditions: appendicitis, cholecystitis.

I perinition,
o Nouseo & vomiting in the I't trimester (max 6s-12'h week)
o It is usually in the morning, reloted to meals
g It is not affecting the generol condition,
I lncidence:
I It is very common 80% especially in P6.
!e tiotogy
g It is unknown
I Treotnentz
9 Reossuroncg ll'tr3l ' i:'r.'

9 Diet:
Small frequent meals: better dry CHO meals, I fat.
- Avoid recumbency immediately after meals
- Fe therapy is temporarily stopped (nauseating)

g Drugs: if not respondingAnticnrctics (lV, IM, suppositories)


)ron;!,y:*l\g_

I,oa s 5* - elJ //t.


-/
,*f- vh
Jtt -

I
Chorignic homone6 (oestrogeos.
progest€ronoB, chorionio gonadotrophinsl
I Definition
! Pernicier.t5 aj:#i!l vomiting with pregnancy

! It is affecting the mother's seneral condition

! It leads to metobolic imbolonce.

I Etioloqic theori?s fi
l. Type of potient
I More inPG & multiple pregnoncy
2. Psycholo0icol theory:
r Start only after knowing that she is pregnant
r Morq common in neurotic females
r Vomiting only in front of her husban(
I Isolotion, reossuronce & sedotives may stop vomiting.
3. Allergic:
r Against corpus luteum of pregnancy, s?X hormones

lMay respond to ontihistominics & corticosteroids.


4. Hormonol:
a Stimulation of the chemoreceptor trigger zone by
IUCi (as in Vesicular mole, tnins),1 Estrogen,
J progest eroneand J corticosteroids
tt3, T4 (hyperthyroidism), transient needing no treatment.

5.

1 Vomiting') hemoconcentration & J blood supply to the organs


et
1 Liver ) Fatty liver, centrilobular necrosis.
I ) Tubular necrosis.
Kidnay
1 Heort ) Brown atrophy, sub-endocardial hge
I Broin ).Wernick's enmphahp*y, petechial hemonhage &congestion
1 Peripherol nerves )neuritis
r Fundus exominotion )
Edema, hemonhage, optic neuritis & atrophy
a
p afAo/ot/ )t o )
inv + e/p
iKei t^ccf
+
5"{t1.s,,,,*, phtrol rrtr^rit.;
I

oplie ehophT

d.iso rderr e. g,

- frfa 0,.c. de rir { :

- e leln(1te irr.b^tc{qq
Excessive vomiting
. Aliover the day.
.Not related to meals
Emaciation, oliguria, conStipation & | weight.
9 Signs of dehydrotion
: Vital data: I blood pressure, I pulse, I temperature
Eyes; sunken, jaundice
Inelastic Skin & dry tongue
CNS:
. Peripheral neuritis
:
Wernick's encepholopothy {rofing eye movemen! drowsihess;
amnesia & hallucinations - due to vitamin Bl defioiency),

a lnves'liootions :

t- To dioonose the condition


r urine analysis for chlorides, albumin, ketone bodies (bed side test).
- CBC increase Hematocrit value,
2- To know the etiolooy
r T3,T4
- U/S to document pregnancy, exclude V.M. Twin, ectoDic.
3-T
- Kidney function tests: Creatinine, urea, uric acid.
- Liver functions: JGOT,
c SGPT, alk. phosphatase, bilirubin
r Serum electrolytesi Na, K.,
r ECG. EEG
Fundus examination

I t. Intrahepatic cholestasii
I .. Hepatocellular (WI, oi6hosi$ |
ofpregnancy
Acute fatty liver of pregnancy
: : Drugs r
o Treatment

e Aiethod:
o Hospitalization:isolatioa (no visiton) g)l^Jtsf &ij Pl
o Bed rest& reassurance
o Diet:
- Mthing per os (}rlPO) + IV fluids till 48 hrs after vomitiug stop then
o Start gradually by clear liquids + CHO solid meals
. Then semisolids & fluids at last
- Fluid chort: Input [fluids givur], Output [urine + VomitusJ
- )
if failed totol porenterol nutrition +vitamin Bl (thiamin)
o Observations :
- Vomititg: frequency, timing, color
- Vitoldoto:?, T, BP

- Urine: volume, analysis

- Orgons function fests ) biweekly


- Fundus exominotion ) weekly
e Drugs (no dntg approved by FDA)
- Sedotives; phenothiazine (chlciroprom azine, neurazine)
- Antihisfominic; Promethazine (phenergtrn), Navodoxine (meclozine + Bs)

- Antiemetics: Metochlopramide )dopamine antagonist [primperan,


Plasill, cortigen Be, Motilium (domperidone)
- In resistont coses: Zofran (Ondansetron:5-HT blockers) +/- steroids

r Indicotions:
g Persistence of vomiting
g VD: t HR > 100/min, |systolic < l00mmHg, t T > 38"C in qpite of neaturent
g Absence of chlorides in urine.
g Progressive rgr8l, liver, CNS, orretinal affection in qpite of treabnent.

r rliethod
Before 12 weeks: suctioh evacuation or D & C
After 12 weeks: may needhysterotomy due to bad general condition.
I Definition
r Chronic metabolic disorder affecting CHO, fat & plotein metabolism due to relative
(type II DM more common) or absolute (type I) insulin deficiency in response to a
-f
CHO load (or impaired effect of insulin at cellularlevel).

) lncidqnce
r 2-3% of all pregnancies
r 90% Gestational DM
r I't most common metabolie disorder & 2d most common medical disorder during
pregnancy
r Banting & Best used insulin for I't time at 1923

I Clossificotions

Closses
t. llodified Priscillo White Clossificotion I 978
Closs ffl Diet controlled gestationalDM " Fasting blood sugar < 105 mg / dl

Closs ff2 Gestational DM requiring diet & insulin (Fasting blood sugar>lO5mg/dl).

Closs I DM ofonset at age > 20 years and / or < l0 years duration

Closs C DM ofonset at age l0 - 19 years or l0 - l9 years duration

Closs D DM of onset at age < l0 years and / or > 20 yeus duration. t


background diabetic retinopathy

Closs F Neplropathy

Closs B Proliferative tinopathy or Viheous Hemonhage

Closs H Ischemic art Disease

Closs T Renal nsplantation

9.

a Tvpe l: Insulin-dependent DM, when not pregnant (IDDM)


A Type 2: Non insulin - dependent DM, (NIDDM)
A Type 3: Gestationaldiabetes (GD)
a Tvpe 4: Impaired glucose tolerance
&arn t' e linicaL el^rtai/;cahon t-

lor.,r ris.lC ( no tia.f or 6W oTt,


A;9 h ritK ( if . a.moc^hleJ u^.tL
maf. or 6r*J c.-fL)

clarmlT 3-
,W
f DD F( .;Jtho".l- enJ
"3o^ deafft
pv a _ Slatl< Crn qfix b.lic
aalroL )

- t^ sTable C - no Rf1-;6n'."1-
b
otetal'o\c aalnL)
Claraff' rDD^{ c'i'lt eal orJan dc.^y<
ail+^ ?^^.Q o. relin^l- C PD€) ,.
R^,^"r

fr; Jc;ito- rilhifr CJn


^;fc^h'04
:- €esf Ar <Jf)
A2 Diel- r- inJull.
D o

<U
P^fh1
opat\
rantlt-.^rL
Comporison of Type I ond Type ll Diobetes filellitus:
TYpe I (IDDM 1O%) Twe II (NIDDM 90%)
A Formerly known ps juvenile-onset DM r Adult-onset DM

I Thin patient r Obese patient

A Autoimmune (viral) destruction of the A Familial tendency (multifactorial)


pancreas ) Jsecretion ofinsulin. ) tissue resistance to insulin.

A Patients are prone to severe A Patients are not usually prone to


r hypoglycemia & diabetic ketoacidosis ketoacidosis (DKA)

I Potential diabetes: she is at risk for developing DM.


* *ve family history, previous delivery of a macrosomic baby (> a kd.
* Obese patient (> 120% of ideal body weight), Grand multipara (> 5 deliveries).
* History of previous unexplained IIJFD or CFMF or repeated abortions

A Latent diabetes: DM on stress (Surgery, psychological, pregnancy so gestational


DM is type of it)

A Subclinical or Chemical diabetes: DM on investigations


A Clinical or overt diabetes: Symptoms + signs * investigations
I Glucose metobolism during pregnoncq
* Maternal i
t Normal pregnancy FBG is 60 - 90 mg / dI & I hr post-prandial: 120 - 140
mg / dl. (transferred through the placenta by facilitated diffusion)
r There is a bi-phasic effect of the pregnancy on glucose metabolism

I st holf of pregnoncy 2nd holf of pregnoncy


Insulin sensitivity
Insulin resistance
(due to J 4uconeogenesis, I glycogenolysis
(tE, P, CS, HPL &prolactin)
& telycosen deposition bv E & P)

y This exploins thot:


. J insulin requirements in fi
% &increase again in 2nd % of pregnancy
. The gestational DM is not apparent until 24'h -30th wks
. Screening is done at24 -28 weeks.
. Also patients with GDM don't have CFMF
6rc.te
$r;t; t^lt J A:ffisron
?"ce-Ptor
+ tJa

fnsuL; n in s.rli n
Stt'si;;vib, Gs.stanc
'ltt YL oF 7"7 ', a,tJ b '€ Pr7
* Fetal:
r Fetal blood glucose levels are l0 -20 mg/ dl lower than maternal levels
r Fetal pancreatic secretion of insulin & glucagon itarts at 12 weeks
) Comolicotions

* Pregnoncy is diobetogenicb.)s. j13'^,, b:-riJ:


o Explanation:
- DM may appear for the l't time
- It becomes ilif{icult to be controlled
- Diabetic complications may appear & become aggravated
(renal&retinal &DKA)
o Causes:
- tplacental hormones (E, P, Corticosteroids, HPL & prolactin)
- Insulinase activity of the placenta
- Relative deficiency of vit. Brz & chromium (lost in urine)
I

6 Pregnancy:
- Glucose passes to the fetus by facilitated diffusion o
- Gestational glucosuria (|renal threshold for glucose n = 180
mg% becomes L40 mg% during pregnanry).
- Associated renal & alimentary glucosuria (rapld absorptfon
of glucose g high rapid peak but return to normal
promptly)
-
Morning sickness & vomiting g, starvation ketosis
o Labor:
- Due to uterine activity
i Puerperium:
- Loss of placental hormones
- Breast uses glucose to produce lactose
, of DH
Com ?l
s@

FN?N, DH
- Quri afl , P;atsfog enic
l^abo/
AA/rv . hffotl,Tccmia
" ptulonlcJ
. obsfructcJ
g "tcs
" 9s
T

T
,u
t!
I
d4-lr
l,,t*i Y
Materna l

Irreversible glycosylation of fetal proteins


-
- There is | 3 times in the rate of abortion

I preteralabor due to:

u- Over distension: Macrosomia, PHA


g- Due to complications
-Avold Bz sympathomim etlcs, cortlcosterolds are reladvely
contraindicated.
I Polyhydromnios33% due to
Malformation: GIT, Renal, open qeural tube defects
Fetal polyuria
Large placenta
Commonest cause is idiopathic (90%)

' Plocentol AbruptieTl(vasculopathy & PE)


' Plocento previo (arge Placenta)
. Preeclompsto}S% "due to vasculopathy, JPGI2"
' Py elonephritis & wlvovaginitis
It is due to I cell mediated immunity (TB follows DM as it shadow)
Here only give regular insulin).
oDuring labor
o Preterm labor
o PROM: fetal & maternal infection
o Cord prolapse
o Prolonged and obstructed labor (due to macrosomia)
o lry PPH: retained placenta, traumatic & atonic
oDuring Puerperiqm (4S)
I Secondary postpartum hemorrhage
I Secondary anemia
I Puerperal sepsis (due to prolonged labor)
I Subinvolution of uterus
I Pulmonary complications
I Defective lactation
f[. Fetal complicat,ions
1. IUFD ( 5% r+ a*- *_!r) due to:
9 PE, Abruptio placenta, CFMF
. e
Hyperglycemia */- ketosis or hypoglycemia
o
Vascular affection ) chronic placental insufficiency
o
Sudden unexplained IUFD. Occurs at 36 weeks & repeat at the
same time. (once mature ) terminate)
2. IUGR (20%) due to chronic placental insufficiency in long stantling
DM
3.
a t3tirrr.s the normal (n: 2-3%)
a Especiallv if Hb,q.r. is increased> l0%
a Due to tglycemia, Jglycemia, hyper,ketosis during organogenesis
a No diagnostic malformation
a Most common one is CVS (10X increase: transportation of
great vessels, VSD, ASD, coarctation of aorta)
a 2nd most common is open neural tube defects (5X increase:
anencephaly, spina bifida, meningiomyelocele). To orevent it

a Others: GIT, renal, skeletal


a N'Iost characteristic ) caudal regression syndrome (sacral
agenesis syndrome). It is rare due to vasculopathy of fetal iliac
vessels.
4.
S Definitiofr:" -

weight > 90th percentile


4.5 kg in non diabetic &> 4 kg in diabetic
t Pederson theorv: (f insulin, GH, CS)
Due to fglucose & amino acids in mother ) hyperglycemia
& | amino acids in the fetus ) finsulin (istet cell
hyperplasia) & growth hormone from fetus ) marked
anabolic effect
This is associated stimulation of adrenal cortex ) fsteroids
) lNa &HzO retention.
It marked on the shoulders (more insulin receptors)
s The Nervborn is large heavy plethoric with cushingoid features
! Complications: shoulder dystocia
ilt.

g Cyonosis GDS):
o Timing: 6 - 8 h after delivery (D.D. transient tachlrynea of
newborn)
o Cause: I fetal insulin -;
inhibition of steroid action on pneumocyte type II
) surfactant especially phosphatidyl glycerol, so is the best
I it
assessment of fetal lung maturity in DM (better than L/S ratio).

2. Joundice
.
Cause: physiological, prematurity, polycythemia, cephalhematoma &
Oxytocin (dlsplaces bilirubin from plasma protelns)
. Management phototherapy or exchange transfusion.
3, Po[cythemio (33%): HCV > 66%
. Cause: tErythropoietin by chronic intrauterine hypoxia
. Complications: hyperviscosity $ ) Renal & mesenteric VT
. Management exchange transfusion with plasma or albumin
4.
* Cause:f fetal insulin so when maternal glucose is removed ;r
hypoglycemia
.!. Management: IV glucos e l0% & avoid hypothennia.
g. Hypocolcemio (< 7 rng/dll & tetony
. Cause: due to funcdonal hypoparathyroidlsm (f maternal Ca r.tll vtis &
IMg dependent adenylate cyclase -;, I activity of PTH)
. Management:lY l-2 ml of calcium gluconate l0%
E,
o Cause: as Ca
o Management: Mg So4
7. Cordiomyopothy
o Cause : thickening of the septum of the heart
o Manasement: beta blockers
g . PNMR (4-10%l due to
b Causes : CFMF (40%),Prematurity & other complications
-
g Birth injuries
O Cause: shoulder dystocia (wider than the head )

lg. CF,t F: The most common cause (40%) ofPNMR


tt. fnheritonce of DM
DM is diognosed in mony times during pregnoncy for l$ time
- History is suggestive (symptorns ore guery) but investigotions ore o must

L Method
t
o Value:The most accurate & tbe most specific
g Other names: Glucola test, & O'Sullivan test
g Method: Give 50 g glucose then measure blood glucose after lhr
g Results
- If < 140 me / dl =no further testing as it is a normal result
- If > 140 me / dl : require confirmatory test (3 hrs GTT)
L UCoS€3*ve if > 120 mgldl
3.
* Random blood elucose "< 200 mg/dl"
* Fastins blood slucose less than 126 mgldl fiJl r-rES

* Glucose in urine (rvorst screening test)


.-. By reducing agent:Benedict, Fehling,
. False *ve ) Gestotionol ( | Renal threshold to 140mg Yo), renal
& alimentorv glucosuria, lactosuria, Vit g Solicvlates (this
interferes with Benedict test so better use strips).
. 0r better oxidizing agent Clinistix (Glucose oxidase), Osazone test
(sugar is precipitated as crystal ) specific to tlpe of sugar).
Il. Timins
r In low risk ) 24 - 28 weeks. Universal screening i.e. done for all
pregnant women without any complain as recommended by the ACOG
r In high risk(potentially diabetics) ) done at I't antenatal visit to be
repeated if normal at24-28 weeks. Risk factors include:
1. Maternal obesity or age >35 yrs.
2. +ve family history.
3. Chronic hypertensron /renal disease f'sill 'irlS
{, History of
'Gestational DM /Impaired GT/idiopathic PHA
'Fetal macrosomia, CFMF, Unexplained IUFD
f n(esb'9 a-hbor fDA

6'^
\--7\---- -ts
d;t t
- low p,ik, -st7l '3oqTT-
?q_ zg
Ih, P*7^t;4
- c1- Ll an

u.e// *,y
I -Indications:
o Xt is the only confirmatory test for DM
E It can diffeientiate between DM, alimentary & renal glucosuria
2-NIethod:
I Oral intake'of 50 g "UK" or 75 g "WHO" or 100 s "US,A" glucose in 400
ml water, over 5 minutes
I Detect blood sugar level eyery hr for 3 hrs
3-Precausions:
* Daily l50gm CHO diet is allowed for 3 days before the test.
* Fasting over night (8-12 hours).
+ No srnoking is allowed at least l2 h befofe the test.
4-Interpretatjglrs:
Norrnol Curue
Blood Plasma DM Benol olimentory

F'BS <90 < 105 At least 2 readinss are All Normal fasting
1-Hr < 165 < 190 higher than normal readings Itr is high
2 -Hr < 145 < 165 :
Class A1 Normal FBS are 2nu h is normal or
3-Hr < 125 < 145 Class 1r2: |FBS normal low
3'd hour is normal
Urine -ve -Ye *ve if> renal threshold *ve only *ve at ltt hour
glucose at peak

t One abnormal value = impaired glucose intolerance. Manage as A1


t Normal values are to be repeated at 3rd trimester in the high risk group
s If GDM: start diet therapy & insulin is started if FBS >720 m1yo

It Glvcosvtated HbAlc (N: 5-8%)


o if < l0% allow pregnancy,> 12 n/o in eariy pregnancy )1% of
CFMF
o reflects the control in the last 2-3 months
l: Glycosvlated plasma proteins. fructosamine: Reflects the control in
last 2-3 w

. Renal function tests, PE


' tr'undus examination: background is not dangerous but proliferative is
an indication oflaser but if still progressive consider Tcrmination
Fetusi weekly biophysical profile starting at 34 w
l) Control:
1 Glycosyloted Hbnc:
- Allow pregnancy if it is less than l0%
- If > 12% ) more liability for CFMF
2) Complications of DM
1 Control of Complications "as LASER for retinopathy" before ptegnancy
1 Advise against pregtancy (eveo theropoutic obortton moy be done) if:
- Marked renal or progressive retinal affection, ischemic heart disease
- Hb.uc >12%
3) Shift the oral hypoglycemic to Insulin
rInsulin hs better control .
tInsulin doesn't cross the placenta so less fetal hyperinsulinemia & complications
I Oral hypoglycemic moy be terotogenic if used during the I't 8 weels

1) Control:
A.Sife:
o Done in a specialized antenatal clinic(obstefrician, physician, dietitian)
B. Diet:
o Must be done in every case@
o Sufficient alone in impaired GT & Al GDM
o Caloric needs 1800 -2400 Kcal / d (30 KcaUkg/d * 300 Kcal in 3d trimester)
o In the form of:
- z}%proteins better high biologicalvalue
- 50%=200-250 g@ comflexCHo
- 30% Pbiy-unsatrrratetl fat
g Diskibuted as
- zs%breakfast, 30% lunch, 30% dinner.
- 15% mid-morning, mid-afternoon, & bedtime snacks
g f Vitamin Bl2 &Minerals (Zinc & Chromium).
C. Exercise: physicat activity should be moderated,
ManngemeaL or 7), 11 .
freaucyfro.l.lrtfenof irtfrn nat-al--
-L , hsfnafuz
@c t- QestaA'onal

prcyil.

@ a^2t @ hosp o 6i)

@ tA;fi @ inrctjrah'anr a -Jtlt


@ Are oF up
D. fnsulin theropy l+ ee

I Class 41 : Diet control only (insulin if FBG> 105, thpB) 140, ZhpB) 129;
r Class A2: Diet + insulin

,- - Reolmen:
o Dose
6 0.6 U / Kg in 1st trimester
\ 0.7 U / Kg in Znd tdmester
\ 0.8 U / Kg in 3rd trimester
o Method:
{ 2 infe*ion regimen:
- 213 in morning (NPH: Regular = 2: l) before breakfast
- ll3 in evening before dinnlr (NPH: Regular = l:l)
A thrlce daily:
- in resistant cases
- NPH at 5 PM is given lhr before bedtime snack
better control of nocturnal hypoglycemia & FBS
\ Continuous infusion pump: Not better than injections

U Urine
- Should be free of glucose
- If present ) | conesponding insulin (but gradually)
$ Blood, check
- Fasting BG, glucometer (cap. blood estimation, better)
- thr-pps should be < 140 mg/dl
After each meal until control ,then Zlw then llw
I In class B-T:
- Reglnen: no change in previous dosage (if sugar is controlled)
- frdlustnbnt os closs fl2
The patient must be
Warned against qmptoms of hypoglycemia (headache, sweating,
palpitation, hunger, epigastric pain, dizzines ) if oocuned any CHQ
(candy or biscuit) +J the conespouding insulin dose.
It is given SC in the abdomen, arms, thigh & buttocks r"l. -ill{
Patients are to be taught with variation of the injection site.
IsuL'n fr^F'l
6e= p eal-

L/s
t/s 1lH
?frn
2) Hospitalization
f Eorlyto calculate & a-djust the insulin-doSe (for better control. ICFMF)
r Lote (after 36 w) for planned delivery
t At ony timeof complication
Antenotol visits/ 2wks till 32nd wk, then /wk till 36th wk then hospitalize.

3) Investigations
o. To detect mofernol complicotions
? Renal & liver FT, fundus examination, investigations of PE,
I Culture and sensitivity for urine & vaginal infections.

b. Ante-notol fetol surveillonce


* frssessment of fetol molformotions
.!. Maternal serum alpha fetoprotein (for ONTD)
.!. US (18120 wks for CFMF)
.!. Echocardiogram (20 -22 wks for congenital heart diseases)
* frssessment of fetol growth obnornolities
.!. US ) abdominal circumference (for macrosomia or IUGR)
' + flssessment of fetol wellbeing
-!. At 26 -30 w: Daily Fetal kick counting chart till delivery
E At 32nd w till delivery: CTG, BPP "most commonly used" &
Doppler
* frssessment of fetol lun@.!_tJt
.!. Phosphatidyl glycerol (best)
.!. L/S ratio> 2.4

1. Control during labor:


IIV drip of glucose if hyperglycemia occurred give insulin p."ill +tis or
I Drip of 500cc glucose 5Y" + 5 units or regular insulin over 5 hours
)
rKeep moternot eugtycemio (80 - 100) to avoid fetal hypoglycemia
I Elective CS or induction: The morning dose is withheld or halved
2.Timing of delivery
1,. Closs f I: allowed to complete 40 weeks until spontaneous onset of labor
(provided there is no bad obstetric history )
2.Closs frZ, B)T:
- If well controlled, no maternal or fetal complications: At 38 - 40 wks

- If notonce fetal maturity (37 -38 weeks) ) induction of labor


3. Diobetics: should not be allowed to pass dates (40 weeks)
4. ln cose of repeoted sudden unexpected IUFD
- Termination earlier than expected date of IUFD l-2 wks
5. lndicotion for terminotion before 37 weeks:
1) fetal distress
2). Marked maternal diseases: renal, retinal, ischemic heart disease

3. Method:
':' C. section if macrosomia or previous history of unexplained IUFD
* Voginof. AROM * syntocinon * intrapartum fetal monitoring

4. Care of newborn: (in NNICU + EXPERT NEONATOLOGISTI


I Suction of the oropharynx + mointoin adequate air way
, if RDS develops or immature ...ossisted ventilotion
s Cord blood sqmPle {glucose, Hematocrit, bilirubin}
, Hypoglycenio: give IV glucose l0 %
, HCT (>66%) ) renal vein thrombosis: exchange transfusion with plasma.
. Hltperbilirubinemio : phototherapy or exchange transfusion
o TetonV : give IV calcium gluconate.

z.Gestational DM [A2):
r Stop insulin with 4 times daily glucose monitoring
r If once blood glucose > 200 ng I dl) give regular insulin
, If persistently high ) give biphasic insulin
s. Pre-gestational DM:
; eive holf the tost dose of biphosic insulin
Adjust blood glucose level between 80 - 200 mg / dl
"
n.Breast feeding:
Loctqtion is antidiabetogenic
I Allowed except in severely complicated cases.

s. Contraception period:
? Physiological (lactation), barriers & sterilization are the ideal
? mini-pills & progestins only methods are allowed
t COC are contraindicated in those with vascular changes
t IUD is contraindicated in those with depressed immunity; otherwise
(aseptic inseilion, antibiotics & cut threads short) can be used (cat.l).
,St

{ a rJ f ,r
{o s
t
U <E (5 3 E q
o- a H
Dt-( u
o o
(L
.a-
f
cl
o-
I
a c
)
o
S-o
C, '-
.qU
o-
-!
G J
not lrulil try iaohi z z L z I I I I I I

trcql?l bv iatqh:,n z z .,
L \ t t I
!
t
I

Dnthgc. cacalL, 1/\ L 3 L t


1 \ t t I I

A;sh'f "6
L r-,'+t
Qcsf Dt1
o Glucose intolerance
' If there is only one obnormol volue in the 3 hr-GTT
' These patients are still ot rlsh formacrosomia & PE

' Treatment: only dlot confiolbut recheck FBS &lhr postprandial every 2wee
o Gestational DM
- CHO intolerance recognized for the lst time during pregnancy & disappears
afterpregnancy (whether insulin is used or not for trCotncnt)
o Postoartum conseqlrences
o Risk of type II DM (50 % may develop overt DM within 20 yrs ) ;
o Recugence of GDM (reported in2l3 of cases, . in obese wometr )
o Tvoes of insulin
t Beef lPork /Human (mixtard) better
the neutrol protonlno of Hogedorn
-NPH is
Onset PeoR Duration
Short ocling (regulor. senilentd ll2h 34 G8
I nternedi ote (I,IPH. le nt) 2 &lz tG24
long octing (PZI. Uluolentl 8 12'.16 24.32

t Tuoes used:
.
combination of Crystalline + NPH (intermediate) e.g Mixtard
.
Long acting insulin: are not used as they give poorer control.
o Somocvi nhenomena
\ lnqeosd nornlng FBS
1 Explainedby nocrurnol hypoglycamiofollowed by exaggerated counter-regulatory
mechanisms ) increase FBS
\ Treofipnt decreaseMH of 5-PM
o Dawn ohenomena
s lnaewd munlng FBS + obsent noctttrnol hypoglycenio
i- TteotmenEincreaseNPH ofS-PM
o Tvoes ofketone bodies
Ketoacetic acid, acetoacetic acid, B-hydroxy-butyric acid
- is the most recent classification of DM r^+. &?
c What
t Gestational DM (class A)
O mOu with no end organ damage (class B, C, D)
C IDDM with end organ damage (class F, R, H, I)
o What are the tvpes of GTf?
r

l- 0ral GTf.
2.Cortisonestressed0Grnitisnotdoneinpregnancy(pregnancyitseIfisstress).
3- IV GTT: indicated when the patient has nausea &vomiting

o Tvpes of diabetic comas


--
- Diobetic ketoocidosis (most common)
- Hypoglycemic como
- Hyp.rglycemic hyperosmolor non-kefotic
- Uremic & cerebrovosculor stroke
o What is the management of DI(A?
A. Prophyloxis: is the best because PNMR = 50-80 % in diabetic ketoacidosis:
l- Proper preconceptional, antenatal & postnatal control of DM.
2- Treatment of any infection.
3- Any vomiting or metabolic disturbance)hospitalization.
B. Active treotment
l- Monitoring in ICQ:
'Blood glucose, ketone bodies, serum Iq PH, blood gases.
2- fluid theropy
. Loading: 2 liters of 0.9% saline (0.5% saline if Na >155 mmoUl) over I hr
r then 200 ml /lt

i- lnsulin theropy
'lvbolusegular
insulin is given 16 units if glucose is > 320 mg% &
l2units if between 215-320 mg%
.Infuslon:
It is given 8 units /h until blood sugar becomes 180 mg/dl.
When glucose + 180 mg% ) J insulin to 5u/h by continuous
insulin infusion & start glucose 5 % infusion.
When ketone disrppear, continue the cdntinuous insulin infusion
at 0.5 u/h for 24 h

switch to the sliding scale technique


4- Bicorb"-rr::";:::;;rs'
a. < 7 glve 90 meq'bicarbonate /liter saline infusion.
b. 7-7,\$ve 45 meq bicarbonate /l to be stopped when PH >= 7.2.
5- H theropfl; according to serum k level.
f?e.a.baT eal- aDF DkA
qci dof''
Dnbeh'e fi61-o
\
,U^ HCot
fr,lTh&ff

TCU
Classif ication.ti.a- rJ+ a.{.

r6estotionol hypertension (non proteinuric PIH)


. lt appears for l"time after 20 weeks
. It resolves spontaneously after delivery.
r Preeclompsio or EPH gestosis

r Eclompsio (PE * convulsions)

2. Pregnan cy aggravated hypertension:


.:.Pre-existing hypertension worsened by pregnancy.
.)Supen-imposed preeclompsio

'iSuper- imposed eclompsio


3. Chro ni c (coincidental/ assq ciated) hypertensisn :

o Definition:
. Hypertension or
. Elevated BP diagnosed during pregnancy & persists after puerperium
_#
o Etiolocy: essential hypertension - renal diseases - pheochromocytoma -
Cushing syndrome - Conn's disease - coarctation of aorta.

Definition
o It is hypertension, proteinuria t edema after 20th w in a previously healthy
PG
o It usually resolves spontaneously after labor (edema is not essential for
diagnosis)

9 It can occur eorlier


r In gestational trophoblastic tumors.
r Multiple gestations
o Other nonas:
1 PIH: Pregnancy Induced Hypertension (toxemia is a misn'rner)
: EPH-9esiosis: Edema, Proteinuric, Hypertension (by gestosls

organization)
-

lit rr Pr.?^an ilt*


Nof rnaL rvor.a^L
it (Qesf nt)
l'
=n6 ciaur;c-Pf il
T
I
I

I
I
I

oll t
.at
15
d
U
rt-Jll
o$
l,L
t0 HTil

qh,
ff eln. 2o *t 1q

A,/ol t*al* €PH /orm*L

Qo 0d&
ftult'Prq9' 'e)
7
vf1
':.. -'l-
Lr

) lncidence
. 5-t0% (commonest medical disease,with pregnancy)
. 30-50% of maternal mortality rate
. The recurence rate is 30-50%

9
1.

g Increase in VC PG: tluomboxane, endothelins, PG-F2u & fibronectin


o Decrease VD PG: prostacyclin, PG-E2 & nitric oxides (EDPJ).

g Delay in the formation ofblocking antibodies


o That leads to abnormal immune response to the hophoblasts & fetal
antigen
g It is more in Primigravida & SLE
3. l-rliS 6. Oi:.
g It destructs vasoactive peptides within the placenta.
4, Absent 2ry wove of invosion
o It leads to ischemic placenta ) release of substances that cause PE
5.

o Genetic (AR), uterine overdistension, Factor V Leiden mutation, RAS


activation (f sensitivity to angiotensin II + VC & aldosterone secretion).
Older theories: Dietary factors (fvitamins, proteins, ffat, CHO, salts),
cold weather (VC)t;.ll 6e, uterine ischernia & endocrinal factors.

1. Thiocyanate & nicotine destroys the placenta


2; More occulTence of PTL

iJ:lD.:'lr::L'.lt':.'t:11:.- .._.._.. i
g ffisk fottors
1. 15pe of patieqt:
o Old or very young (>35 or >20 yrs), low social class, obese
o PG (genuine .{\Ji:G.ll), occurs in MG if superimposed or another marriage
o *ve family history.
2. Hyperplacentosis:Twins, DM, vesicular mole, & polyhydramnios,
erc
S-, 3. Vasculopathy: Renal disease, chronic hypertension, antiphospholipid antibody
syndrome & Previous history
2 TAeor;es

@ fq /g/,- @b,7.7-'4o^-
e n hqcl;"',
.,- T2
PCT

/
AfFedeJ ia
roso.lopo\
@ irrmuntlolical /Aeor/ i-

- Noa f-.*..hia otr blccF.a,


Anlib"J;es, vel \vt1 l,
gcrt o u 176 r+rc Fu
- iatu/T;c;ent bl*Kn1 eofiloJ;cr
lyprrshrrnt*;,

-7ieori et )t ;rL.
S7z---rl-\7r-\7

? Pothology
.t GeneroliZsdvaso-spasm and edema
1 Intimal tear & fibrinoid necrosis of the vesset p6;; with sub-intimal
edema and deposition of hbrin & immune complexes g
-
. Obstpuction ,& thrombosis: Local hypoxia & infarction
. Rupture ond hemorrhage into vital organs
Multiple orgaxs are involved, it is o syndromg(not a disease) and hypertensior
^ is a milestone of this syndrome.

Lungs ;Pulmonary
edema, and Laryngeal edema
-h#' - '
+ rIF, biown u:t op-nytm-. - 'l
-Er------r--r-----{
es, detachment & papillcdema. r
.---J
I lPeri-portal necrosis and Sub-capzular (Glisson capsule) I
I Liver rupture
.. lhemorrhage + intra-abdominal hemorrhage after its :
- - - i '!
a
' ip;;iril.'l
| ;''';iil;FpririiJorm?-n;;h*s'G;ot.il
Hidney i Olmesangid cells, endotheliosis (diagnostic), Epi
nanowing of the glomerular vessels -+ |RBF & IGFR.

OAcute tubular necrosis & acute cortical necrosis ) acute renal;

Il-
I
I
I
I
Plocento i, )vaso-spasmofi
. i
!
placental blood vessels
_-___
) local hypoxi4 infarction, ruGR, !
!
JYP,-* gt-tgt:?t
htrgrygt
- - - - - -r-
I
I -
l-
q
--
f. - r-- --- -!- --- -J
I |. Hypernatemia and f uric acid "lst to be affected'.
I
I Me tobo I i smi' . Jfu' tfragmented RBC's,
t
HELLp g (Hemolytic anemia, Elevated liver enrymes, Low
t
I
Platelets) due to microangiopathy. I
,I
.I
Olftrurhb,
(furdthi,1

tubinlimal elen^
7'.)
ftrL;l )1cnrt:i

bro;a

labolic
Syrnptorns : (only in severe cases
l. Sr,tellings of lower limb_or uppqr limb (rinos become tight'l
- It is not essential for diagnosis.
2. Neurolggicol symptoms
- Nausea, vomiting (central or peripheral)
- Visual disturbance as bluning cf vision up to fvisual acuity
. Good prognosis: retinal deachment
. Poor prognosis: occipital lobe infarction
- Headache (frontal, persistent not responding to analgesics).

3. Symptoms of complicotions:
- Epigastrio pain (stretch of Iiver capsule or subcapsular hemorrhage),
- Oliguria (<4@mlldoy), or Anuria (< l00ml/doy),
- Symptoms of heart failure or pulmonary edema
- Other complications (as abruptio placenta,IUGR)
- Severe acute abdominal pain:
. Accftlental hemonhage
il

. Hepatic rupture tl
It
il
Signs : (More imnortont & present hefore the
syrnptorns )so ANC is essential :i

o. Edemo:
I Not essential for diagnosis & has little prognostic value
I It is due to capillary damage, I transudation by hypoproteinemia & proteinuria
a If absent = Dry pre-eclampsia (more severe)
a Type$
\ Occult edemo
- Is detected by rapid weight gain in 2 successiye visits > llzkg /wk
or > 3 kg /month'Earliest sign of edema"
- Nornrally: <lt2kglwk in 2nd &3'd trimesters
2. llonifest edeno:
- Lower limb (medial malleolus, chin oftibia & dorsum of foot)
- Abdominal wall 'Peau D' orange'r
- Generalized with face and hand swelling (late)
2'
rJr
prl t )rn^

4/
5 ['r.Jnc*T

I
f. .
QJe^0, l?ttuAa

\F*"f'xa
b. Profeinurio: (Lote & serious)
@ Detected by

' mid-stream urine (not by a catheter & exclude UTI)


Boiling of acidified urine (qualitative)
Albustix test (itrips with tetra-bromophenol)

. z4*,:'iJ:H'## l,'f,',.;;[ffi ]?'il,: i:ilff j*'


@ Normal )
l00mg/L, significant )
2300mg/L, severe > 500 mg/dl
@ It is non-selective (albumin, globulins & transferring)
@ Proteinuria without HTN is also risky for both mother & fetus.

c.
9, i ,Systolic BP to 140 mmHg or f 30 mmHg above pre pregnancy reading
9, I Diastolic BP to 90 mmHg or I 15 mmHg above pre pregnancy reading
g Precautions:
- In a semisittingposition or left lateral position.
- BP reading must be twicewithin 4hrs apart in n restingpatient
- Diastolic point is the change from the 4'h to the 5th Kortkof sound
- Blood pressure reading is not corrected to the nearest l0 mmHg.

d.
I mono symptomotic Aestosis (pesence only of hypertensionor proteimrio I

rl
i or edemo ) is rce & is colled iiirpr syndrcme rrtt qfis ur d'i.ilj IS! pts ;

9 SeS

t Routine investigotions: CBC, blood group, RH


L CBCI f Heqratocrit (hemoconcentration), HELLP
3. Detection of olbirmin in urine.;f Ca,oliguriaoranuria
a. KFT: t(uric acid lst to rise, urea, creatinine) & J creatinine clearance
L LFT: AST - ALT - PT and PTT - Serum albumin
6. Fibrinogen level.'and FDPs for DIC
7. Fundus exominotion to detect spasm, hemorrhage, exudates & edema.
8 'TeSB Of fetAl Well being.'as Feral Kick Chart, Nst 4 Bpp.
9 -DOpplef ilS .'For early diagnosis of placental insufficiency (fvascular resistance,

diastolic flow (J, stop or eren reversed)


t{a/urot |ir/o Irrfe sl;gafr onr
l,'/ea t C),ni CaL
/are pAa.t<

. Qou-oaer lei
o .fen,r; t;v; tv
r'
)
c ITat'n"l tsP
6^&-
S*
1. Roll ovef test trei,. -p jl rjli-:
"fpressure in supine position"
1 Mean BP is measured 0n one side then on the back then on the side again
i Norno blood pressure will I on the back due to supine hypotension
A ln potient lioble for preeclonpsiothere is fdiastole at least 20mmHg
because on the back, there is pressure on the renal vessels )renal ischemia
& renin angiotensin activation which in normal patient, no response
2.:
A ln normol potient no response

a ln potient lioble for preeclonpsio )l diastole at least 20 mmHg

3. Moximum BP of night (reversal diurnal variation)


i Normo lood pressure is more in morning

A But hereit is more at night due to recumbency )absorption of edema )


f blood volume ) f blood pressure (so coinplications are more at night)

o Edema:
Unilateral :I Rilateral
-r r " -'- r -' r r'
; irtf ;ih:yrilrE",i Grf;.:t d-urk-lr't'oi-dyi:
f
. Varicose veins II .Anasarca: hepatic, cardiac, renal, nutritional & allergic,
a oEndocrinal e.s. Cushins. mvxedema

o Hypertension L.i,a cjrl u,.l d+.a$:ll


l. Pregnancy induced hypertension
2. Pregnancy associated (coincidental /chronic) hypertension
3. Pregnancy aggravated hypertension

Proteinuria
l. Folse proteinurio(The commonest cause)
g Due to contamination from vaginal discharge
g Avoided by mid stream urine, catheter specimen (not preferable).
2. Gestotionol pro.tei nurio
3. Urinory troct diseosas:
9 UTI (pyelonephritis, cystitis),
o Renal tubular degeneration (hypoxia as CHF, seYere hyperemesis).
4. Orthoslotic appears at the end of day (pressure from lumbar spine on left
renal vein).
Moternol mortolity Perinotol mortolitlt
9-s% 2s%

:""""""
: CNS iedema, hemorrhage a.!aIt.ataa..
i \ fr€tP syndrome
!..{].{L:1...i*qyls.llt*leL*..'.911is*1.+.'.trg.ti:.......,i {Hemolvsis, flivet

: <100.000/mm3 with
3 rmproper soagurauoD,

rrJaaDaaarr..

Lung
' r,uu!,
. f .--- ' rl
:! egEItrE
edbma, hemonhage & reSP[aIOfy
OemoITtrage dg railure.:
respratory IaUUfe.:

- Remote (3R):
\ sidual hypertension (5 - l0%)
\ sidual proteinuria (3 -5%)
\- currence (30 5O%)
. Fetol complications: IUG& IUFD, Prematurity (spontaneous, induced)

i r=Gritetia ol seuerit I

1. Headache, blurring of vision, Epigastric pain


2. pulmonary edema
i 3. Systolic BP > 160 mmHg & diastolic BP > 110 mmHg
i 4. Proteinuria 2 500 mg/dt, Oliguria I
i s. HeUp syndrome, etevated tiver enrymes I
I
I
i 6. elevated serum Creatinine I
I
i 7. thrombocytopenia, DlG, hemolysis I
I
i e- tuen or IUFD I
I
!____- _______J

I Prognosis
e Mild cases )resolve lO0%
e 8409[ ?residual hypertension
s 3-5916 ) residual.Ielal affection
1, tosis
1. A,tild EPH gestosis:
A Systolic BP is up to 159 mmHg, Diastolic BP is up to 109 mmHg,
Proteinuria is < 0.5 g/dl (24 h) & edema is in the Lower limb

2. Severe EPH gestosis:


a systolic BP is > 160 mmHg, Diastolic BP is 2 ll0 mmHg, Protelnuriais
0.5 - I gldl (24 h) & edema is in the Lower limb and abdominal wall

3.
r Severe EPH gestosis + slmptoms
(epigastric pain, vomiting, mental &
auditory hallucinations) + Hyper reflexia + biochemical evidence of severe
organ affection

4. Fulminoting EPH geqtosis:


A A case of acute onset proceeding from normal to mild to severe EPH
gestosis to imminent eclampsia within a period of less than 12 hrs

r;lis
,.. r D.a. r a a.. t 4 a..... ra aaa a a.tr..a..a aaaa

I Fulminating & imminent rrrl3 tlLr O,::ll .rts qrrtl ...ijl :

u 8.,",;;;.:;;;;.;;;;;;;i ;- ;;;;;;;;'

? Types
l. Ante-plrrtum eclompsio 70 - 80%
2. fnho-portum eclompsio 15 - 20%
3. Post-partum eclompsio 5 - l0%
- "the least common and the most dangerous"
- During the I't 48 hours & up to l0 days after delivery
- The disease is far advanced that it occurs without the need of placenta

9 €tioloau of fits:
E) Unknown, but may be due to:

'
elecholyte imbalance (increase Na)
* Cerebral ischemic foci5, vasospasm or platelet thrombi.
? Stoges
l-
I Semi-coma* Severe headache + Hallucinations
r Twitchesin the small muscles of the face * rolling of the .yEbdt
2-Tonic stoge "30 - 60 sec."
a All muscles of the body go into contractions (episthotonus position)
o Cyanosisas the respiratory movement is stopped
9 Exhaustive HF: Hindrance of venous return
3-Clonic stoge "3 - 5 min."
s Intertnittent contraction and relaxation of the reciprocal groups of muscles
g lhis stoge is choroctarized by:
. Fall from bed, fracture of bone, micturition, defecation
. Asphyxia dropping of the tongue backward, aspiration, stertorous
breathing
4-The stage of como:
Variable from min. up to several hrs
-
Due to severe acidosis & cerebral depression
-
Fever is a grave sign may indicate intracranial hemorrhage.
-
I Fote of the fit
.i. The patient regain her consciousness
* Deep coma & death
.1 Stotus eclampticus: the patient passes from one coma to next without
regaining consciousness
.r Inter-current eclompsio:the patient regains consciousness and
continues her pregnancy ll$ &.ll 9{J pjYl gje+..,;rd
I Complicotions of eclompsio:
1.
+

2.
A. Asphyxia
& Tonic contraction of the respiratory muscles
& inhalation of vomitus, blood from bitten tongue
& falling of tongue backward
& Pneumonia
& cerebral depression: acidosis or hemorrhage
B. Hyper-pyrexia
ir
I

i
' Fits: recunent (especially > 6) &postpartum
' Como: long (especially > 6hrs) & deep

' Vito! doto: BP (systolic 2 200 mmHg, diastolic Zl20),


pulse > 120 b/min, temp > 39"c, RR >40 breath/min

' Oligurio, onurio


' Dry eclompsio

i.. .. 9lg:1.*r1e...i qllltj.!tl:i::Ti3:11,:.1_ _.._.i


I Differentiol diognosis of eclompsio 4t
l- Conyulsions:
I Cerebral shoke fit: With lateralization. Diagnosis depends upon CT scan.
$ Hysterical fit i Not a classic fit, usually due to a psychogenic cause.
$ Hwertensive encephalopathy fit
$ Epileptic fit of grand-mal epilepsy: It has the same stages and also the
EEG changes are the same as that of eclamptic fit. A past history of
epilepsy with absence of symptoms of Pre-eolampsia is diagnostic.

V Sfychnine poisoning: history of drug intake.


2- Comaz
t Cerebrol, metobolic, poisoning
!. 0r9on foilure: hepotic, renol foilure
I Couses of deoth
1. As pre-eclampsia t Complications of fits
2. Orgon foilure :

) heart failure & pulmonary edema


) renal failure
) hepatic failwe
) suprarenal failure
3. Hemorrhoge in vitol orgons:
) cerebral he &encephalopathy
) Abruptio placenta
) liver hcmonhage
D suprarenal hemorrhage
4. Complicotion of convulsions: Asphyxia, hyperpyrexia
?' +
Proohuloxis
The disease itself is unavoidable but
o Regulor ontenotol coras essntial for early detection
. o Patients with high risk should be seen more frequently
o Anfenotol odvise for high risk groups:
e ontiplotletsa.s low dose aspirin (81 mg) or dipyridamole.
: Volue
-ITX-A2 (formed in platelets ) VC.* platelet aggregation)
-Minimal effect on prostacyclin (formed in endothelium )VO +
platelet aggregation)
I Colciun500 mg lx4 )f prostacyclin ( no evidence&f stones)

: Antioxidontsas vitamine C & E


3 Solt restrictiol(no evidence )

? Conseruotion
OWhen:
o Immature fetus
o The fetus is not in distress
o The patient is not in labor
o The case is mild
OHow:
r Bed rest:
a,At least 2 hrs in afternoon *8 hrs at night
5 Better to rest on left lateral site (avoid aortocaval compression)
l Advantages
l.Mental rest )decrease blood pressure
2.Increase renal blood flow
- Help dieresis )J BP
- J Renin angiotensin activation )I BP
3.Increase utefoplacdntal blood fl ow
. Diet:
; Should be balanced

- Decrease salt, CHO, fat


- Increase proteins, Ca in diet
-T-r*F,neat oF n;ld FE..-

,lo aspiria
I

I otn lr'o Ii J onli

1- Gr.S<( Ya'troa

'o sr)t
- im r. tifu , nof in l;sfrctt - BDS, observafrd^
--llre Pl
ir asf in blot
c r1-i tJ b Spee;6? (Aoli Hrl,r)

3- G^inrEoa

-VD
- c_s
Sedatives:
* Diazepam (5-15 mg lday), phenobarbitone (60-180 mg/day )
* Decrease anxiety
I Steroids: to enhance fetal lung maturity
a 0bservation

Moternol Fetol
Doily ' Blood pressure ' Fetol kick chori
' Albuminurio ' FHS l6hrs
Weeklu ' Weight (for edemo), ' US for fetol grorth
' Reno! A liver ' FWB: CTi, BPP, Doppler
function ' Test of fetol moturity
' Fundus

Soecific:
- Antihypertensiyes which don't . plocentol blood flow.
--* Its use is a matter of controversy
+ They do not J progress to severe PE o, development of proteinuria;

on the contrary they nay J placental blood flow


< 100 mmHg: no need as the high blood pressure is compensatory to
high placental resistance
Used onlyif diastolic blood pressure is > 100 mmHg
r r r r r r r r r r ! r r r I r ! ! I r ! I I I t r I r'.j . ...;;;; " .f ..\j;;zjjj"...j
r] r r r r-frfr r r rJ I r r rffir
r r]
e i 250-2000mg/d I Hepatitis (liver !
r r funcion test done r
l: ro choice, safest ! vasomotor centre : ;l every timester) :
rrrllrrrlrti rll rt I l!t lll I lll ttrllrttrl
i Lobatol'ol ! - o & p blocker ! zoo*e tab. lyot used in heart !
h I therapiutic lblock :
tal
lr!
rJ rrr]rrrtrrrtrrrrr!
t : Not used- in I'r :
i fidotott I - r.ry effective lto-zorg+-Otn I uir.ri., or with r
I Usea with caution ] - t- affect the fetus : : :
l.r r. i..1... rr r f rri rllrrJtrr rril ,l r...t.rr rl rl
I Terminotion
* If mature, fetal distress or in labor or become severe
{'Eith.r vaginal delivery or CS if indicated
Prophyloxis as mild pE

.1. In order to avoid development intrapartum eclampsia.

t. Prophyloxis
9 Totally preventable by good control of PE

I St frid meosures l+ ++:


g At home @
' IV sedation
. Transfel 1s hospital
'o Avoid light & sound stimuli
O At RR:
. IV line + sedation
. Loading dose of anticonvulsant l+ lr1 lrg
' Transfer to ICU
o At ICU:
9 Semi dark, quiet lesrn (echmpsia room gr c,)lJl dn- +ii)
9 Patient lies on her side,
L 02 supply, mouth gag, tongue depressor, endotracheal tube, suction
machine, anticonvulsant drugs.
o Observation:
. Vital data
. Levelofconsciousnbss
. Fluid ih.rt.
g Durine fit (emergency treatment even done at home):
. Insert mouth gag, avoid biting tongue, avoid tfauma,
i Place on her side to prevent hspiration
', Gi6 ahticonvulsant as y.1ir6 (the best to abort a fit).
,I
After the fit:
- Ozsupply
- Aspiration of secretions
- Lower the hyperpyrexia.
4

,- F.ofhJlc'Xis
Jo.J freatm<at
af ?e
2- tst [); / tAea t'ufe r Aorvtc

?- finti HrY
fE
lyln/^Zin< 0tt choiccS

q - /+^fi Gu'hon
/-
(fi"aY 6t1
fl1 "foq Q"7\tut"S ,D'or'^
t- fi1P"'/;rt keo/n'n t
.-t
74rn pn" , 74",f&of

{- rt29,r'LlA
6
€P;dur*[

ro'r r
'o'^fi,!i Frrn,
7-
Anh'conrr'
Ar. d( tJeonoh
8-

?- e olz

Arr.t c
frnti-hypertonsiva
o Rules:
I Keep diostolic bl. Pr. ) 90-100 mmHg
o Very high BP ) Intracranial hemonhage
o Very low BP ) fplacental blood flow ) IUFD
I Avoid
o ACEIs:
s J placental blood flow & cause fetal renal failure
o Plosmo volume exponsion (As salt free albumin or Dextran)
Given with extreme caution to avoid volume overload
o Diuretics:
t Increase hemoconcentration & electrolyte imbalance
t Used only in heart failure or pulmonary edema

I (trandate)
: itotal
300mg
!.9
aygia.in nll ulogk
.l
se severehypotension I
I
sudden sever hypotension & IUFD !
ii ;'l J; i'- ii-ji ia:; J;-r-r--- - J
I 4tcjl 3 fJi:.c#,rlJl arg u;aa
FF--,-!Irt- r
-r-- -F-.-.-------r-.-'------,E--r- -1
death
seYere hypotension, fetal i
I
Fetal hyperglycemia
:
I
i
iI U /Vo lThrough (NO)" VDrCan affect the fetus by: i
!
I
!

4.

I Digitalis in heart failure, dialysis in renal failure


9 Atropine to I secretions, antibiotics to avoid infections
o Treatment of HELLP: high dose corticosteroids
5. frnti-convulsont Drugs
l-
Drug of choice to pievent fur$her fits'
A, Mechanism of action:
I Sub-corticaldepression
t Peripheral skeletal muscle relaxant (JAcetyl choline & Ca atNMJ )
I Transient hypotensive effects
I Mild vasodilator & diuretic ped urine;
-a- Cerebral dehydrating agent

r r loodino t Mointenonca tI What are the nrecautions


I l..- I

t llVl 14 g slo'r, IV r 5 g IM / 4 hrs r3. Heart rate >1gg7p1n


!. utin, >25mtlhr
l5. Thc best; ssrum Mg level

;f.r Ir , I -th..upeuticlevel:4-TmEqil
lZ.Yerypainful)fit l-;;:;;;;;;
t t t--r-------: llu:lnurroyglrtJggil
:--l-rr--t--r----

, Managemcnt
l----r-r r-r----J-r-:--r--r-
t > Absentknee reflexe "lstsign'at I0 mEq/l t y 10 ml of l0% of calcium
> Respiratory depression at l5mEq/l
|
I > Cardiac depression at25 mEqll
I > Available
Clrrorateslowly(=lg)
I 02, endotracheal
I > In high levels, crosses the placenta ) I intubation + ventillation
TI
neonatal respiratory depression. ; (life saving)

2-
g Dose l0 - 20 mg IV then drip
t Advant; more rapid - may be given in renal affection

. -rv DisadU; Iconsciousness - neonatal hypotonia & respiratory depression


3-In resistont coses:
1*\ fl Phenytoin
lfJ u PentothalNa (intraval ) 5 - 70Vo of 0.5 - I gm IV )
lntro-portum control
r Induction of labor:
o If labor is expected rapidly
o. AROM + Oxytocin
t 1st stage:
r Semi sitting
. Good analgesia, epidural anesthesia (best if plt > 100.000/mm3)
3 fluid l25ml /hour
. Intrapartum assessment of FWB,
I 2nd stage:
. Forceps to shorten the second stage
r 3rd stage:
Avoid Ergometrin 1VC + tonic uterine contractions
' -yf BP)

: Oxytocin can be given


A Cesarean Section:
r Indications:
d Ifdelivery is delayed > 6hrs or associated indications
r Timing
,d tt is done 2-4 hrs after the last fit after correction of the severe
metabolic acidosis

Post-portum
o Sedation
ri 1st el - 48 hours afterlabor ar the last fit
- Anticonvulsant + Antihypertensive.
9 6 weels later
- Renal function & blood pressure are reassessed

Core of neonote
s Managemerrt of IUGR & neohatal asphyxia
! ,,

! Regular frequent antenatal care


g Screening for EPH gestosis in next pregnancy
I Low dose aspirin * Heparin from the start of the pregnancy
I Definition
t Presence ofhypertensioh ltry or 2ry)before pregnancy

o €tiology:
* 25 % of hypertensive disorders with pregnancy

i Primory (essential)

* Secondory e.g. renal, pheochromocytoma, Cushing $, Conn's $, coarctation of


aorta, thyrotoxicosis.

I clinicol picture :
* Predisposingfactors:
. Old age, hi$ parity, +ve family history

. Cordiovosculor risk focfors: smoking I plasma lipids, obesity, DM


{. Symptomiess: ask for

! Durotion of hypertension before pregnancy, complications & treatment


I iledicol: sffoke, myocardial infarction

I Pregnoncy: superimposed PET, IUG& IUFD


{. Signs:

! Hypertension: (occurs before 20 weela, persisls afterpuerperium)

-l No edemo or proteinurio (unless complicated, by superimposed PE or

heart failure)

i-il;"I
l-..--;----r-
go-tos
'1
..1
!r-..E--.'r.r.--
mirc ! -----..-.J
I

m=:g:s
i . 1.. -.. -.10i:l'i.. -..-. i
I _
g_.. 1.. _.. _.: _.._. ]
eom&ations =.lt1..
o.EffEct of preonancy on hypertension :

. Increased severity
. Superimposed PE when
t Blood pressure (Systole >30 mmHg N $i llrf, Diastole >15 mmHg)
9 Proteinuria */- edema

P Appeuance of complications peculiu to PE


I Lower 24 hr urinary Ca (40 mdd) than in chronic HTN (220 mdd)
. Maternal: all complication of pre-eclampsia esp accidentalhge
. Fetal: IUGR, [UFD, preterm labor.
o lnvestiootion
t ECG changes, X-ray -+ cardiomegally
$ Renal function: creatinine clearance (120 - 160 mVmin), BUN (8.5 mg/dl, nor
pregnant l0- 20 mgldl)
t Fundus : atherosclerosis * hemonhage.
o Treotment

A Conservative TTT:
+
Bed rest, diet, sedotives, steroids, stop smoking & olcohols
+ Follow up (rate depends on severity of HTN)
. Fetal growth: monthly starting from 18 week
. FWB : weekly starting from 32 week
o Antihypertensive (if >105) single drug or combination of:
tr 1st choice: Methyldopa check liver enzymes /trimester
tr Znd choice: Hydralazin ( if >200 mg ld for > 6 m -, SLE)
tr 3rd choice: Labetalol (200 mg tab 1x3 up to 1 .6-2.4 g ld)
E Used with caution
a Nifedipine (teratogenic in animals, safe in humans in 3'd
trimester, but may lead to acute hypotension )
A Clonidine: safe, but acute withdrawal g hypertension
tr Contraindicated :

A ACE inhibitors
a Diuretics ... except in severe cases (heart failure)
I Tcrnrination wheR:
4 Fetal maturity (37 weeks)
+ Fetal distress (e.g. severe IUGR)
-- i- i- t- t-
(-:'f- t-
-'r-'f- l- i-- T
-rf-

* Illcdical trcatmcnt ....good lesponse


+ Failed mbdicril control "'terminate

* Hypcrtensivc crisis : same as in severe pregnancy induced hypertension.


-u-v-v4

i HTN i Pree
J-----._--.--L--
i noe i old >35 i ,tny ,

|a-rrrrffriru ! .ny parity ! pc

l-' a'-l
-;';;;;;;'-!'-; ;,;;-'i
bluning i
--
YlF;;;;'
-' I
mild hlPertension, i
epigastric pain 1 urinary symptoms :

| \rl9)1, r Aoortlorl, ltJrLr, : usually r\J I


i ntstorY
^ i lucn, Abruption i i as HTN ,
i
' ! Superimposed PE : : :
F-------1-------- -[-- -'f " --"-.|
IPH::'*ve: *u' -ve
lA !
J----
! !
L----
-u' !
L--- '--'-J
Signs ! nrN ----L-- ! ! Mild HTN excePt if
(before 2Owks) E-P-H : renovascular
heavy proteinuria
- " ;";k b;i;' - " l-'' -'uft+'",.r""'r|' - ",-l
la

l' .,rd*; -''


- ";| - -' -
| !''
hypertensive casts (red cell & hyaline)
retinopathy
-ve 4-nsiotens,iq +ve sensitivity -ve sensitivity.
sensitivity
Norm'al urinarY I urinary Ca Early unaffected
Ca
Uric acid: normal lncreased Early normal
except if
superimposed PE

HTN Chronic renol foiluro


€ffect on preg. IUGR, IUFD, abortion, abruption, superimposed PE

€ffect of preg. Aggravated Mild diseos€l ns effect

Treotment Mild: conservative till 37 wk Mild: conserve


+antihypertensive
Severe: terminate
Severe :terminate (cr. Cl. <65 ml/min)
* renal transplant
$ Definition:

-l The number of moternol deoths /100.000 live birtlrs


I Due to cousesrelatedto or aggravated by
: p[O$D?IIC/,
= labor or
: puerperium
: or their management
I Imespective of the duration & site of pregnancy
I But rct due to accidental or incidental causes (non maternal causes).
a lrtAi rutio (better thon rote)= M of moternol deoths/y X 100.000
l.lo of live births in the so,ne yeor

! / 100.000livebirtls el+s
140

'1 174 / 100.000 (l9q2o7au.I1


I Recently, the ministry of Health & population had reported that the result
of MMR is 84/100.000live births in 2000
t ftl 100.000 (2003)
! ln developed coun zolt$ @0livebirttn(. 8/100.OOinUSA&UK)
$ Couses !+Jill ohl+ l+ a-{-

I Direct
..couses
r' - " -lI lndirect couses
- .. - " -'
I-.-ppEiizf - ;ciliui i-si"'
! . HTN 22% .Anemia (Hb < 10.5 g/dl) l1%
l.epu9%
:'-P-u:!pjt-uliepgi18J.- i.-E-- -----i
* Other couses; anesthetic complications, thrombo-embolism, medical disorders
enal, liver disease or DM.
* Postportum collopsedue to:
ok.
O Septic or cariliosenic shock.
+ Diabetic coma.
o Othersl Postpartum eclampsia,
o Amniotic fluid embolism & Menrlelson $
$
t- Pority:
- Grand multipara(> 5 deliveries) has higher maternal mortality
- 2"d& 3'd deliieries are associated with the least MMR
- Then MMR in.r.ur., thereafter
2-A9:
- <20 & > 30 years are associated with higher MMR
- Between?} -24 is the best period & Ieast MMR
3-Sociol closs :
'The higher the class the lower the MMR
4-filode of delivery:
-
Vaginal deliveryis safer than CS or instrumental delivery
-
Use of ecbolicsJ MMR ( even their use by Day

E-Antenotol core: it is the most slgificont foctor


6- Worni;g s)'lnPtoms one
- Pain (abdominal - epigastric), bleeding, R.OM, Decreased fetal kicks
- Vomiting, lower limb edema, headache, blurring of vision

$ Reduction of fitMR:
.l Good antenatal care:
o Properhistory taking, examination, investigation.
. US should be available when needed
. Earlydetection & management of problems.
. Hish risk cases are either hospitalized or termination of pregnancy.
* Good intranatal care:
g Should be in the hospital
o pror..ffibor ltt' ,2nu ,3'u Stages,
.o Earlv detection &managementof PPH
g Use of ecbolics
9 Well qilalified doctors (obstetricians, anesthetics etc.)
.r Good postnatal care:
U Carc against ppH
s- Asepsis
U Use of antibiotics
!- Preventior of puerperal qggUg_&-DVT
L
CONT=NT=

o Normol lobor '


oP
o Passe
o Power
o Stoqes of lobo
. Active nt of normol lobor

tions & mo
o Occioito or oosition
o Foce oresentotion
o Brow presentotion
o Breech
o Shoulder oresentation
o Unstoble lie
o Cord presentotion
o GomOleX DresenToTlon
. Twins
. Abnormol uterine oction
o Controcted oelvis
t

IS
f;

R'
rrir{
r Aaa<
rir{ r{ierrnAaa<
r{ie,rn S

I',,
),s
DS
SL
sr.
s(
)s
t

ns
S
s)
-:--
-:-- ---...
---... ---4-:^a
---4-:^a D

n'
De(
D(
(,

s,
s(
s,
I

n'
D'(
\2
,1

o ADnormol
IS
2t jX' vagina' and vulva
=' '

1 t
',^::-:"'sacrum&
coccYx
I i i t
't
n !- - -
-
--
-
svTpll,j,-p,,bj,- g*r=1j=t*rU*1t,, 2;.1 .a.j,lus,1ol1.f:,:!

ove I ne reMlc 61rtm p' 3t ,ots: I2


I ' Not imoortant known by external pelvimetry )
t
,rom outer border of S.P. to tip of
from last (I
lumhar spine (L5) )

( ' IruLs4g'lfie-e; D

) ilil.ffi::,'.-i**,:
'(
In 5^--r-
;i-*r
't

nrioq t
[ \ t
s

c Joinlr -, tbones _, 2 h,f boAc

\ 'q ( ,n".r- +
t
I ,I 'I ioinr' t
.... Sacro oclSeoL
D ToinL ,(
,t
2

kt
Dr ' M'nr tr

l.Er Eblb lo{.m6h

,,, fc'nr -",a;


v

\-
i &rrrraMm

[ l;;---t
|l'',
If,r.
;" |
Etit
i\y

I .- t
t

n F:1r."',,r::::::,
-('::"'t) t
I ,'lon'€'att}./
J;mcns;oa'
t
I
Tcbic
Ft $
(
)
2 swzrsvzlrrzzrsvzt i
:
I
!
sacral promontory to : +F
. S,P.
S,P.
I
I
:
:
:
oint of sacral promontory to the most ! + Lower bo
ging point on the back of S.P. i
I
,u.rurn
;
t
Pushed

I promontory to lower border of S.P. ; head du

one measured bv PV. ........1


:

7/:;!rS
(!t^.r,rll,
N
N :: :: . --.,, Pen nointq on
fnrtlreqt points
een farthest nn the.
the ;

2 I Transv. ; ilioRectineal line (unimporlant). :


;':D.
\rn.r

7lr.
i :
I
ls
(SN.rl !r ir lUtlB'lttr to
njugate LU rI

)2 tr: r I I I r t lr: t t ! t rthe pelvic brim :


Il II Ifrrr! aaa r a ! l rrrrttrIrl r r ll r r I I I M Ir !
Nr!'
N::I
Nr

( i
D i ++^,^*: i(
Ii i,
left.
i:
: : nt to i E

it:s
D ioo,,o,. i
Sin -
2:
"\!D:
'd :
: i :: i':2is
.N ::
N I
:;
oFrom -------r'
sacral promontory' to th eminence: ., .I: Dl,7,
D : : ^-
on the same side,
+r-^ ^^.-^ ^;,.r^
. I q
( : eDiameter of engagetnent of biparietal!
i : )

E o In anclroicl pelvis the anatornical is behind the obstetric i.e. nearer to the promontory
(_
*
4 More use of right lower lim
n I :['J:.'::,::::[J.T::H
4 Pelvic colon encroaches on ob,ique(4.rJ,4s)
oblique (4-rL'.,,!d). I $
foa********-.r-^.

Fxte, nol .""IlSot S

lntercr&
I u I
S
g

t /," .--/)7
L2
crest
I- (

z'4 t
iS
,
S

?h//,./t
D
s IS

)
I The outlet :s. lil AXa
Ll+".e-

\ ItItisis auSEGIYIENT
9EJtlvIelU , bounded 2
' {or,-c: the plane of least pelvic dimension ( rstetric outlet +i5,)
c..sj
)/1
S
,
r fD-f^.... the
!|o[o'w: +1.- plane of
-l^-- ^f /'l^.,,o' border
(lower h^".1, of symphysis (
S pubis, ischio-pubic ran sity, sacro-tuberous ligament, ')

IT' a

Plene of oreefesf Pf) Plane'of leasf PI)


(

I'
)(
( I n^----r^-:^^
'r^---r^..,^^ l-.^,r-^^^.^.^'-^r^^^-^i..r.,-.
+^ +l-^ ^^-+^- ^f ^^^+^L,.1,,-. I -^:-^ +L^ r^...^-*^*,,^-^"1
*^*r^^
-^,-^ +^ l^.,,^- L^*I^- ^T
)

) t Leneth
Length ! 1?.5cm
7?.5 cm I 1o.5x13cm
10.5 X 13 cm I (
. Plane of obstetric outlet.

n L _ _ 1.11,,.,,,r
. Internal rotationoccursin
rotation occurs in .
The pelvic obstetric
l:i[:ifii::'lf,:[!..,.,i,l f axis

xr I"
-1 it

n l'*
Importance turns forward
. The head is engages if
l.Hf,H{to;n*u,.sirthel
vault it.
reaches
t the
n' I -tt )py'
S

n 1",' l. .yr. ll
l phnc o(
-A:Jt
Le"..rt Felric dimcnliont t
f; t
I pk,ne
Pt('^c oF onoro^,rf
oP ctAAromictL .ff/rt'
ourrcr I

nlt'
|

It,rt
I I t

nl' v-vllposterrorsaeittirpr'nel

2S
I

att
I t

,'rs vr vyJr !
rra.!r
is c hang es . fs d;r"
ht'o'd 5
)e^eat 4
tt o ccuil uAen fAc occiT
6r.c2t c-tPcmfiu ( iil
t,
o s al itr /c rd-
blocK is Cau ie J a
; h Ortqinati,ry Fro,*^ t

JJ
I'I')t
()

II
cavity and outlet till the level of ischial spine
(-nn,," l-A :*i"
I$ . IZ- n.rrn h' :::T:
:1"1:L"l: nrt onto nF (- nnr, fhen i::':::i:'n'1;''',:rw,rr,I
"
dnwnwnrd qnd forwqrd t
It
),s
sr.
\(
7/
(,)

It
nt rtltlt.lttntttr' U
:.,,.;, : ;5ub-Mentobregmotic(9.5) ! ,\
\ : : - From below
below nter: - From below chin to center of
of;! //,
, I Fully i of bregma (ant. fontanelle). ! bregma. : S
S
N : : - Engaging dianieter in O cipito- ;
EngagingdianieterinOcipito-! - Engagingdiameterinfacewithi
Engaging diameter in face with; Z ')
')
D
7
!h I l a
, !
t I t t I I tti r.a t aa
. fullvextendedhead
fullyextendedhead
aaAi rrr rr rr i aar tar r lra
aa !aa aaaatttt!a tr i
(
: \'S
N :;
S -
: ' isrrh-Manla-vo,iianl fll 5\ i ')
):rr
V : : ant.endi ign.rti (
S : : fbregma. : ior&l Z

S !........r..1 ......r..f 9......: ')


E lDeflexed:
aata : : S
:: 2
S I or I: ofnose:
of nose:
"""i
bone) ; - Engaging diarneter in brow; i (
aal..l
mirtwrv ,:
)( iI ,iornu, (rn,. of frontal uon.i^
(ant. end oifrontal uro*
S : ' i -tngaging diameter & distends! "largestdiameter" : 2
! . IIEAU Uf UI gEVrr I
rlar rttllaalallatalataltlalalllltaaaalaallalllaalllllalllll
sP'=
(
, I ront

I Ir ry-It
p |l:::"
I ""

Antg?ior
,r-rl
Blparieta
dl6m6tar
9.5 cm
I
Ill/lI smb
1."{6"y V*" I t
It /fu$"-
rrlt
,onlorielle

n
Il;-:-l Front!,
bone

Fonlal
3UtUtA
Sinclpul

S q - Q;tfttc,.hlfc Ca'put fro^ A

od oA
I )\ " .P. tsroc"') $
^-Fo.e
)'-

)^*;.
'

tli
n . lilo..u is rert and anterio, t
{here are 8 maln positions eoch points to
Left ant I Jeft iliopectineal en'Linence Rieht post 3: Rt sacroiliac,joint
Direct ant: symphysis pubis Direct post: Sacral promontory

Rieht 4nt 2: Rt iliopectineal eminence Left post 4: Lt sacroiliac joint


Right lateral: Mid iliopectineal line Left lateral: Mid iliopectineal line
n
l.:-= +^a3y f*il +iS ,='r !J& IIJ E
* Left occipito-tronsverse is the cornmonest position during engagemento@
n Occipito onterior is more cornrnon than occiPito Posterior positions due to
a

better occommodation between tlie concavity of the front of the fetus and the
convexity of the lumbar spine of the mother
Left occipito onterior is more cotnmon than right occiPito onterior, and
right occipito posterior is mors common than left occiPito Posterior,
because in LOA & ROP the head descends in the right obligue diameter which is
longer than the lefi oblique diameter used in ROA & LOP.

A. Mouldine(reshapins) B. Caput succedaneum


4 0verriding of skull bones o Edema of scalp or skin over
a Molding doesn't occur in skull presenting part due to prolonged
o pressure over the girdle of contact
f-t base & face (protect higher centers
& eyes from compression )venous obstruction )edema.
b 4sequence: parietal bone over each o It gives an impression that the head
t other then parietal over occipital is engaged (rule of fifths is better)
bones then frontal bone (late).
+ J Head diameters (0.5-lcm sL) 0 Show the presenting part
(Jalve 4 if excessive clLt) intracranial o alive fetus during labor
hemorrhage due to injury of vein ol + If very large = obstructed or
Galen prolonged labor.

+ 1't degree = obliteration of sutures o Cervical


trtPt o znd degree : overlap but reducible 4 Diffuse: obstruction by bone
I 3rd degree = overlap but +|IJ (obstruction by vulva)
irreducible +Artificial by ventouse (chignon)

:\7ZN\7ZN\?ZA\VZA
rl o l/;n
I

VCia oF
Q"r lle n-
IS
IS It
Il
. Never cross the sutures, . Not limited to one bone.
- t t t-- -r---.^ ^f ^l-.-ll -rr^-*^l ^t-:.. o_
'::-o:"
L^.^^

ln I -/ ts
I ll
! lnlectton: dralnage +antlDlotlcs t ulsappears oays Jusl
. Hemolysis & jaundiceo@.
l:ffiT;l,ulii:;::H:"'.' reassurance.
t
I ll
, Brain compression: aspiratiori
I'lJ::il:ffiJn'"avsruslll
. Can occur in is face

IS l'::lmffi::::T,l':::ii:l'"1';,1:.,,,.::'i'i.,,11.,,,Ti'll
not done t'infection will occrrr) on (tumefaction)
t
presen tati

7 : =Anteriorasynclitism ! =Posteriorasynclitism f S
:_
......r.. :-
..'.1..........
I :+ Less comm
abdomep Ot:t ) :+ More in
!-f

() :i
ia

":"'::o";i:'.'"'-':':::,L"j,'.1"','
,n edge (promontory) to pass i i surf oce of the pubis
:',1:r^Y:,,'.1:'",1:,"0*" 'n'! t
i e
(S :
: -Correcled
'---Jby uterine
-'-- contlaction
i - Worsen by -- contraction ,i
-J UT
)(2
)7 i,l Sag. suture is closer to pronrontory ,o Sagittal suture is closerto the SP :
:i+ : .: s
N,- Anterior porietol bone is the lower !+ Posterior porietol bone is the ! A
gqnd gsft|auls a[
prluoq 0r!I| q orurs lel|6os otll
ulnllool.v
r\
lilit \
*\ ?^ nmwtr \
IN, HSI
,.'j,#l
,f l{ I
t
,I tYtr/
ilu,t
Flfr'
uotrrfi$dEt
l" I{il#[nrtrEft ,lutmoto$El
IS
D(

" i;; -'


!'fi'fn
(D rPerifoneum:dAdherent
1--'
tpdler0-4.*i,rd
ldLoose
r/4ii0."Dlfi,il."1,i' i"'ru5- !
!ldentitul,llSinI,SCS l')
(
drhick(3layers) !dil1ztuy..r; !;; ! (
a rdd Well developed , d Poorly developed r p1, ia ')
developed i
lM.mb.un., ld d Loose ;;:':xl1Y<rrnno
).
N rActivity
N:-gr:'L--id- r$
Adherent
!
:'iilYS-E:'98.-i
!t e

nv
()

t
tv - It starts at the uterotuboljirnction (po
- The right cornu precedes the left by 7-
- Impulses pass downwards 2cmJsec
- It depolar.izes the whole uterus in 15 sec
- The muscle fibers contract simultaneou
(fundol dominonce, can be felt abdom

! \ Simple observatio
| \uunuul palpation .xlloh el" q+r
i \f*t.rnal (tocodynamometer) & intern

A.
.-. Voluntary
, v.Yt..e. f
t+ll aljr.+
t. ,- ,3jr=lt:
9J-..
S
+ -J-'
Abdominal, diaphragmatic, interco
glottis.
B. Involuntory (reflex) k+ .-J^-t JJ|JI :
+ When the fetal head distends the pe
+ Pressure on L. Ani ) reflex contra
+ It marks the start of 2nd stage
\
q
I
s.
-.t^,!\ \t
rc F\
-

\
t
,!v \
+il
s' 'h
t\14
\. -.
S' fro'
I'n I - in normal labor
Iis il#J?11'.,,,, of labor) I
rabor (not induction orrabor)
allowed in normal labor
t
,(

terine contractions cslj t'l

rine contractions (false labor pain


infections ) inflammation & relea
hanges,

gesterone ) | PGs

as a quiescent (relaxing) effect so m

to J oxytocin receptors & PG so mu

(by pressure of presenting part on


ptors or * activity ofplacental ox

release of uterotonins ,,s.!l cr

on

thalamic-pituitary-adrenal axis )
ommon in anencephalYot*jll irJ-l
Afr6rtr'o^ La.l=o -
_
I S

I' lv o r lTL aL
S llY () t ffL A L Labo.
L*tr2arl 7

,SJ7, sse^J<r
II / \
/qr/o \
J lYv
$

t
t
porrcyet / \ fo,wer
I Aa ??s.*^, / \ Drtare-t- $

[ S
n'
['.

late, may be in 2nd stage of labo

lief of upper pressure symptoms

rward of the uterine fundus (tfr


in standing
re slmptoms: frequent micturiti

,$
sr,
D t - rrue lobor poi
S7/ S

DI irme d
confirmed
Conf by crG
CTG I (
K),
ri le
63 3i5t
c!
r::*
$ ll-ffi 96
JO
o!
{E
9o
l: s I
iiq?
o
o
rsit
3-=o
, n =D -
o Eg
o aI
l*# 3i ir
{3
g9
n It
o
re 33 ar
1i
'2
TO
=6
@-D
? t;
s ril
I
d s- -5!r
o oo -c.
c !o eE'"
=.:t
o:
o
:.
J
o
ii1
3o{
{Et
agf
s.3ffi
I E5: I

lffiii iliiI |
IlE
dAc
3Ee
2ii.
x

io:
IK -r.*o=.I**^ooooo -r.o-r^-
Ir'\' I
.-r-.-r^.-roo.-r^*rroo.-.-.-rooorro.-.,o.*
$dsif, Obstet
$

change that occurs in the cervix


agenase & hyalouronidase ) split cervical
ls )J osmotic pressure ) absorption of water
qturql or artiliciaffl
Q insertlon of FG)

I ^+ .',i+r^ +Li--i-^ ^f the cervix (shortening of


) the cervix due to
S r It ic lznnrrrn hrr
Percenloge ("/.) U 20 40 60 80 100

Lensth (cm) 25 2 1.5 I 05 0

: uterine contractior.is, pressure of forewater & presenting part


)
In PG: ripening then effacement then dilatation

In MP: ripening then effacenrent + dilatation (occut'together dueto soft cerrrix)

s
( rvix&LUS

Stages of labor

, o)' Lotent stogg


S 4 W.rk contraction with little effacement & dilatation.

9n pG 8 hours up to 20 hours
qn 0(P 4 hours up to l4 hours

ains,2 - 3 cm cervical dilatation


dilatation t ROM
rase: increase in the rate ofdilatation
: maximal rate of dilatation.
ing in the rate of dilatation
1,2 cm/hr. I\{G: 1.5 cm/hr
-
Sl l--D-2
A'

II r, u-l,,'n,
-12:,/ Y) I

I*t -z*do I ,,

.:. ;: l::::-., ,
-=Y ctlacod
f [Vf *o*nto.inisnoressentiar rbrrabc,r
I
'#;:i'::':'*'
I . Corvix is nol 2. Cervix is lully olfaced
"nddiraredtolcm * Give pG or phosphoripids for (

.
)
(Fr7-anlabororabonion,)
*de,avab.r
| Nfll W )l t
Il"ftf] 1,"ffi|
::

s(
?
3 corvixisdiaredroscnr'
*
*-r-reg+;,
,n:r".",,:r.
,:."i:
t
)
(A\zA\rza\Z-rrrzt\Za\Z\\za\Zr

llead d"@
fl17er bol;s Curvt-.

Her, ilP qh 2-9h


lafinf PA,lr- ,hp
3

r {ri eJ rr\a^ Ce( v; Co3CQ,.r,r


ataaatraaaaalala!aaaaaaaartaaat

( 5. n ( tapping on the fundus is not transmitted to the cord)


D
Hech-

Sepord.nn
r, I r*

DunC
cttblidn :-

Jtobular

/.rk blo.J
I
.-f
L;ruP

glM
e last 2 hours before delivery
respiratory disffess, the antid
esia

for:
PG
ation of the head on cervix -
tic fluid embolism
liquor oi'Jiti -J*iil -.xrl -ri'al
ractions are still inadequate, a

lucose (41,.i# lls ufu ) for reh


be given
avoid Mendelson syndrome)

r obstruction by full bladder


the bladder & allows analysis

soiling of the perineum by s


ne naturally & avoid enema (
r, fNN infection or perinatal

e -*
walk
tal head on the cervix by gra

teral position
dextrorotated uterus
compression by gravid uter
ion syndrome
v(
ilo ,r

I
SD
I *wl lr.nt lrt
lrarLf,,
t

*.I- .. dr-l t
)\-'--l't'7t I n'' """--- ts i
n
Active Phase / ,/
I / .{ ,./ r^o

A X
/
/
Laten PI tase

) 4 5 I 10 1l 1 1 1 19 20 2 2 2

Drugs given
and lV lluids

180
170
160
150
140
130
120
110
100
90
80
70
60
oc
cfrq

atation
ring down.
ay occur before full cervical dil

y room, NPO
tomy or lateral

nal: Entoxone 50% O, + 50% N


gnocaine I %
dder & rectum (J pain, J soiling,

t not to bear down without contr


houldn't push during delivery of
min & after each contraction

ion till crowning


n the occiput & a pad on the per
: BPD has just passed the vulval
below the S. pubis (descent with

to allow the least diameter to di


tends before crowing + Occip
boccipito frontal (10 cm)) will d
he head after crowing inbetween
't push during delivery of the he
ort by sterile dressing
the perineum is about to tear (m

: :.: : :'.,,.'.T. :.'11.':.::l l::::: : :'


f ,---'

lbnornth'/r1 d;aqnoie Ca
I

u.t ot Plorrttt;t Hl.ttq


-t jrrrl t
o;f otogeJ PQ>zo tr lals2 d.hSacilt ol E*cetlenl- Fes| +
laknt pharc 6p 7rt h tabor- 6cnce.L
(t^Jct IcJalioa
o Aratl CirorJcrr oP- General- loc d olylo cirt
-oF c* e;L 72 l-,r
c Pt>
-oF llctl*eccal 2L\r ,

o 7nl raclios {'tord.


<l c..l\ CPD - C1<neral- bcl c.5.
-oF cl d;L
Acal Cucenf <lcntt iPct OP
-oF (2 a/a P tlP

e flrtcipi borlC Labor 5e e a bn oronl uf. ocfrba

head sl-al;"n b-j


ir,t on
f e tati l-o i sc h; aL SPin<

(fru'n -5 to +5)

7R, heel di stea,l r


lil ctornn;n7
-flre Per;rcurvr 14einlai^ ftcx;o"
DdiE!dd. b{tDrh odsrd i e ki hcd cb dt &rdfutio

-1 lrcn alloat fx fcnt, in frfter le^l deltoT, ,lr/;w7 4


t^ bclucen co nfuac hint

" Pulling h.ld


lo cncoungt
dkntlon.

7
ND)7q
D"t(
s2

D -t (
S ) slinned
ir *iltipt. ) double clamped a
? i
I i cia,rorceps-lt
cia

n i lJ:rt#orn female genital cutting or poorly healed 3'd or 4th degree tears.
i t

o 1 ,
I Tfil';1,, ti, slgn
tne ulerus nll orpra
sign 0I
o Encourage of the patient to strain
pra

o Then give ecbolics: Methergine(0.5 m


o Catch the placenta & roll it between h
( ffill,ff;l[il,
enerol anesthesio
ood light & assistants t
I Exomine the UUS from above downward
I
II I
1"il81;:r:rromabovedownward
Exomine the LUS transversely

l:ilffit[:Y::;,';H'J'",..,,
Exomine the cervix by 4 ring forceps

l:::":',^':vagina&vurva I
Exomine the vagina & vulva
t
t
t
I l"rnTiil'
)
(
I 2-
:-
PPH
I
f n otions:

Aftel instrumental
After
3- vaginal
deliveries
illrumenlal deliveries
PPH I
's

,1.,, I
cs + PPH
Vaginal birth after CS
I :- I )
I I 4- If abnormal labor courset gl
rii;:tffiiJff:,',:',T:t
5- If general anesthesia is us r)tS,rl
6- Routine by some
t
xlllrtr
ll I I
3t3
($
D'Z
,
;!o
li,

((

&r^/t r4,r/,r*',

Cort rh //ed Co

F.-- l^

afeu\
RI
I\S
t\
s

_ s
l'
IS pole I!:
Btue oote
Blue Hond & teet& !
blue:
blue !
feetbtue .
t
tA
2 r r r ! r r ! ! r r r r tr t ! t.t ! ! ! rt rr r rir! t l t I r lr r !l !1 \
S Absant r <100 , >100 i l,'d
S !r r r r r r r r ! r r .!.. r lr r ! r r !...Jrr lll! rr......!
Z No response Gritnoce r cryr r rr r!: X
S !r =
r r r rr r r r l r *.. rrrr rrr rr r rJ tr rr...lr V

7t !"r'!rr;;":;
d
)

n' + &lrorg vernix caseosa mark the baby &

^-^"-^"-^a--^^
dressing

a
t
Flat suburnbilica'l reg
fv6,-g
vPul 1"rro1/ '
lr,,rfu/ /tt,1t -
+
,,y1olo.'7 [
L*.=*^a^.^*.-^"-^ Oaz-a---a---a--*a__-\nr)
, . F€IT small (Bitemporal diameter is felt) (
S r A,.-^..r+^.,^- D

\ tLat PlYz 2
( The ANr€RtoR is ANT€RIoR so felt easily (also due
t
(
own
ffifffid
aput is formed
by palpation of the
(mask the fo
ear
rontaners) the >,

is ANT€RIOR
Iix is PO5T€RIOR

- UlS ) Confirmotion of diognosis, molformotions & 1


ossociotions. 2
I
\uorF c-'(,,11
r"r
I |
I i*J- " i*J I
d 4uo*'tt"el,J 'lvv
I I
cn T4,
^d \td <o,,a-1
',r/l f
(I I
tlil I
I? q'tj L! |
(
1
I
I-"**-.:^-^:-.-^-^.-.--".-^."-"-*
I'.$
IS
)(
()

FORCEPS or C.S.
nthropoid pelvis
rVaS
(
t n^-.4f1 fq'{' 4) go
rw '"fiffi
o1
"n e-,
c ttal irt o7r- Yrrntte7 2
tr t1n vo.t>ra1
^*j
- During labor:
o If there is an indication of CS perform CS
o General causes as

o cPD,
o Fetal distress
o Soft tissue obstruction.
I
o Arrest disorders
o Protractiondisorders

. Failure to deliver or failure of rotation of:


o Face to pubis
o DTA
o POP.
t.ltt stage: avoid
Iaataatataarraaaaaaraa!!tir.rraaaaaraarrarrrarararraaraal!aaatarrrraraarraraaraaaraarraaraaraaaaarrltrataraarar

;
, .. .. .! a
fnertio
r! .... .! ..r. r. .. r!.
:
.ar
PRO/I :
. .. ... .. ... r r.. r a . r. ... .. !
Sepsis
r.. r.. r.r... r
:a
!. PV ! o Prophylactic antibiotics !
Evacuate bladder & rectum !o Avoid excess
a t -. I

!.
. ! Correct dehydration io Patient lie in bed ! . Avoid PROM
vvrrvv
atl

!. Avoid excess sedation !. Avoid strong enema ! r Avoid excess PV ! a


I r a a aa t ! a ! a a a a a a a r a r r a r a a r a. aa r a a a al a a ra. r r aa rr.aa a r aa r a

Observations are done by the partogram:

a ,\rafgeslo is required due to pressure of the fetal head on

o The rectum
o Presacral plexus
o done cautiously due to:

o Improper application of the head on the cervix


^q00(is
o So liability to cord prolapse is increased
o Qxltocin ougmentatlon may be needed due to:
o Improper application of the head on the cervix
o So the Fergusson reflex is lost
. 0xc[ude obstetrlc ccuses for CS (CPD, fetal distress)

*^-^a*a-^a-"^^
Careful assessment of the station and position
) iZ i (

D i teatest pelvic dim sions. i ?


( iI OA
head the rotation become
i )
;d
r-r
n t
ensqsine /( Nil\:I:J',,
Jiq-ctc,rir I \ lx,t lnenro_

;.J"i \x<l'i:xt enja3ial

'[6'ilt \ .5
(tt cc.,y d'a".eTor

t--'-J .,ot' S.,,b 6Crrfo


bregm.[i<
o P"CDeF[<r; ( 1-9 ,,^)
---va-.-

t (aarno n
t { l' Ae cno'f
.Sp.r-. of ae<.k, nqaclt,r

. lolicr -
)t lon Comrmo n cclhrt\
' fintnct'rl;
aeck
Tt
otloolt
cF cr,e)
azooal
neck

( f"lt (iu<, on1 qS


Crr
#*:;i""-,d

(
t
g.t
Can

({;o {.rtenr

?ra
/ As during pregnanc)I
/ Earty p/V (4P 6rlJl)
/ Late P/V: Feel the nose, maxilla, mot
/ If Tumefaction develops, so it is di
lAbdonrinal exarnination (Fundal G
I Careful P/V:
- Face: Mouth * 2 zygoma mak
- Breech: the anus + 2 Ischial t
Supro-orbitol ridqes Nose
Double curve, nasal root, orbits & frontal It is recognized by the "saddle". & its
sutures. All rray be obscured by caput firm elasticity.
Mouth Anus
Soft lips, tongue & firnr gums Grip on finger, nodules of sacral spines

I
-MA 80%:
! Descent
! Ensasement by BPD (9.5 cm) &
. Submento-bregamtic (9.5) if
o Submcnto-vertical (l 1.5 cm)
d 1,, or. of the oblique diameter
!.tExtension: chin meets the pelvic fl
! Internal rotation: 1/8 anterior ) di
S Flexion
I Submentum hinges behind SP
. Submento-vertical (l 1.5) dist
!
-M P 20%:
N ZtS of the coses rotote to becom e MA
)
$ The remoining 1/3
o Rotates 1/8 to become deep transverse arrest

a AS

- It needs extension of an already fully extended neck


- The pelvis can't pass thoraco (sterno) bregmatic D.(18 cm)
oP -Jr
DHP (/
)

o JeliYerT

))r
)7a 7^\vz-s ?
;!
? malformations t
! If there is an indication of CS perform CS
! lst staqe::. 7/

. Avoid S
U
o Premature rupture of membrane
o Inertia
o Sepsis.
. Active management: partogram, analgesia, AROM, oxytoc
. Exclude obstetric causes for CS (CPD, fetal distress)
. Exclude CFMF

@-
o*, sponta eous rotation for
I hrs in PG
%hr MP
Th n examine the patient
. If the Head is not engaged perform C.S.
. f the Head is engaged
o DMA:
. Deliver normal
as
. Forceps is used ifneeded.

o DTA / PMP
. C.S is the best.
r Manual rotation and forceps
. Kielland forceps
.r /rroniafn , for
Cranioto^.,y fnr dead
r{ool foft-o
fetus
(
o DMP: CS is a must

- Exceptlonal tnar.agefient in very small head or in anencephaly


t

Woit (2 hours) for spontaneous correction to face


occiput
) or 2
. ff iled, perform C.S. or Craniotomy for dead fetus
$^ D
I Complications:.gs face
t
st
I I(/,,,e.
3
f l[.'(\(
n\"\\ _,
ll 3

S
g
t

i week
lr r r r r :r r r zSth
15% :
Ir
Ilrrlrrrrr
a=r r r r...'.i
rl t ! r rr. r r lr
lrrrrr rr rr r r7".....
r r r r r .r r.
r r itr
:
7-

IS
l4th week
=
rrrrtrrrlrrtrrrlrrrrrrrrrrrrl
6% N
,)

). (

r*,*oJ;ffi;;il
n'
( - ior (right &left) ')

n- t

n'
)
n'
s

D<
? lrvpes:
z"'
trd
- Antenor & to the nght
I:;l::::::::::::::1i
Dnciorinr 2, +n +ho ,inh+ tffi I R5A
RSP t

D
I1 ,l

NS
It'
(l)t

IlIl 0",
Il ,.,
{-,,\
t
t
t
r.sz
Nlr"t'? t
, Zoe; 29r.; 7lu llot^r E

ce o f Lre"

COr--r.r r.r A
It
I'
I'

n'
v nrL\,r J-r wvvr\D rrrL rvruJ vrrr4rSvJ lrl lvl4lrull tr, !Ul,4rrruurlu ut rrllluur 4
s
') with stabilizationof fetal lie & presentation (
SD

n'
)f |t| Recurrent breech presentation
7 ( lL-
occurs with a permanent cause as congenital
uterine a normality.
normalitv.
normalit
congenital ll K
lll )
' Dyspepsia

Palpotion Complete Fronk


o Early: corresponds o Lower due to early
Fundal level
o Later: Higher engagement .

Fundal grip o Head well felt & ballots o Less felt & ballots

o Difficult to differentiate
Umbilical grip o Back & limbs
back from extended limb

o Large o Smaller
Pelvic grip
o Delayed engagement o Early engagement

Auscultation o Above the umbilicus Below the umbilicus

//s
s,,)
( . gEI
' In frank breech
- Usually early engaged
- Meconiurn stained liqu
! Late P/V:
' Socrum known by:
- The shape
- Nodular ridge of the spin
- The posterior portion of t
- The spines are continuou

' Ischiol tuberosities, feet


Franl kee* fufi Ueadr
D(
s>.
,N
D(
sr,
D(
s')
E I separate
I-- The thumb is separate I(
D(
(
- Finsers run from ankle to toe - Fingers run from r.vrist to palm

the position of great tt

2S
S- u/s:
')

,
Bu tJ o cCt .Sh ootdeft l-le-a-d

Du Jcen J

s
o

I$
+J '&'P'n/;
Vt
C€

C
0
x.

e
$ to 4OYoof
s 15 40% of which up to 85
I 1J

l. Any associated indication of CS


2. Prolapsed pulsating cord, primig
3. Cephalopelvic disproportion
4. Distress of the fetus
5. Extended head
6. Footling & knee presentation
7. Growth restricted fetus
a

- >9 indicates safe vaginal deliver


- 5-9 risky vaginal delivery
-<4:CS
- Its value in improving fetal survi
0 I )
Paritv I'G Multipara
Gestational ase 39 weeks or more 38 weeks 37 w or less
fetal rveisht Over 8 lbs 7-8 lbs Less than 7 lbs
Previous breech None One Two or more
delivery
Cx Dilatation 2cm 3cm 4cm or more
Station -3 or hieher _) -1 or lorver

Another scoring svstem


Vasinal C.S
GA 34-40 rveek <34 & >40 rveek
EFW ?_1 < <2 ks. & >3.5 ks
Tvne Comnlctc .frank Othcrs
IIead FIexed Extended
CPD No Any degree
Paritv I'IP,if successful breech delivery PG or any complicated
CFMF +ve -ve
I'
Qru;o fTachoa
br'agin1 dotrn q6.t

Loue t
dlaocttvc/

"^::xdo"ra & ffi#


N Till the su egion appears. I)
B, ee ch ex trocl-i csn

Bu /n t r/a rt lr.r^! ,
appears below the symplrysis then deliver the posterior buttock
1,
rn of the fetol trunk
- Troction trunk D
- Deliver the shoulders by:
S Bringing dorvn an arm
! Lovset maneuyerdlt+ "Jr lq a+. cJl.l-
c Advantages:
I

- No general anesthesia i

- No passage of hand in the uterus


!

i
i
c Technioue of Lovset maneuver
- It depends on obliquity of the pelvis (what was within the pelvis anteriorly
will be free when rotated posteriorly)
- Grasping the fetus by the pelvis
- Pull the trunk till anterior angle of the scapula appears below the symphysis
- Rotate the trunk 180o keeping back anterior ) posterior shoulder becomes
anterior

i : -reP:l! 1ill -,ry sp ['= Ik s1-!-:l -u-g:-o-l: 9:livg:9 - - --- i


i

- Delivery of the ofter-coming heod:


s Burns Marshal. MSV & Forceps (best).

1- Arrest of buttocks 2- Arrest of shoulder


Either at the inlet or the outlet due to: o Extention of an arm
1) Passases: - Bringing down an arm
o Bony pelvis (contracted or tumbrs) - Lovset maneuver
)CS
o Soft tissue ) o Nuchal (dorsal) displacement
o If rigid perineum ) episiotomy of an arm
o If tumor ) Cs - Due to malrotation.
2) Passenser: - Management: proper rotatior
o Large fetus Cs, ) in direction of tips of fingers
o Tumors (sacro-coccygeal teratoma) o Bilateral nuchal displacement
)
destructive operations,
- Rotate in any direction to
o Extended legs )
bringing down a free one limb,
leg or groin traction.
- Then rotate in the direction
3) Power: of finger tips of the
o Inertia or pendulus abdomen ) remaining limb
or vD
correct ul
vulrgul or breg^L
Cs vr urvv owrrqnrinn
- living fetus )symphysiotomy or pubotomy
- if dead ) craniotomy
s Soft tissue
'/ Incompletely dilated cervix:
'tfliving fetus perform cervical incision (Duhressen's incisions).
)
' If dead fetus allow full cervical dilatation
/
Rigid perineum perform episiotomy
'/
Costriction ring: amyl nitrite inhalation or deep GA.
- (Pqssenqer:

.! Hydrocephalus
/ Perforation (either roof of the mouth or at the occiput)
y' Ballantvne operation if associated with spina bifida)
-
! Extension of head:Burns marshall, Mauriceau smellie viet or forceps
!. Large head: living fetus ) symphysiotomy & if dead ) craniotomy
!t
/ Try to rotate the occiput anteriorly
'/ Jaw flexion shoutder traction to deliver the head as face to pubis
'/ Prague technique "shoulder traction + flexion of the fetal neck by
bringing the trunk towards mother's abdomen"

n'
s
( Cause Prophylaxis
D

n'
n I
II
I
. oecompressron
oecompressron c- vltamln K to momer ds NN
l;: IfffiiLil:fr:T:::"ili
d- Kristeller should be gentle It

n' oi,iLr,irn ,,
fracture base
r uI lrruE,lrE IELIID lrErur E
t

subocciput appears below SP


.t \I Y 9l L\, Ltrt tl Lllg .tgLuD uqlul
subocciput appears below SP
g tllg
tlon f""f^l t
, lteal
,
Jo*
.
^/eck
.UL
-Abdo orcn 4 genifal
,VL
NS
D

D I . ' -r'- L---'-- ---'-r--------'-' r--r- -- -'- J


N
I X
( I -_9(*surestodeoleqsepretwtalmortalltlz-
- $(easures to deqeqse prenatal mortalltlz -
I ()
) |e
E Many obstetricians CS.
oow regard a PG breech as an indication for CS. |
( Bren in parous breech labors, section is much more readily used
E Even used
I t;.;;.i I )
) should not be used
I E= manual breech extraction should used I e
( I m Anficinqte.^rd
lE Anficinqte cnld prolapse, fhe enmmnneet narrsa nf cnrrl
bfeeCh ig the nrnlqnce I
cnrd nrnlqnce
pfOlapSe, breech
.l)
/t
;)o
-N
ofcord prolap
,;^:^^:;::-;::;^:

DS
idence prolap
( t
s

D?
(D 2.
'r
3. esthesia: To avoid exc 3ss force and detect comolications.
comnlications. I t
I disimpact br
iffiff;"r,..
D (
N
D
N
N
6
6.
1
1 ^ in
lll the
i,i*iffi.'.,
"Lould be sentle.
gentle.
ds head to fflexion.
lirr-
direction Ul
tIlC UII'gUttUIl of lirr-
lll .^ ^-.^:J
r^ ^-..^--:^- ^rt-^^l-
^-.^:r ^-.r^-^:^- ^f L^^1-
I(
t
2

7
/,t

2 e checked after the p p (


N abnormal FHS r
ein or ahnormal
,-ain r------ r---- r 2
l0.Tocolytics are given to relax the uterus during the manipulation.
I l.Anti-D. Apply an abdominal binder for a few days.
l2.Re-examine after few days. If breech recurs, repeat the procedure.
Causes of failure

rote is 5 % in
- The foilure 1-PROM.

MP&?5%inPG: 1- Multiple pregnancy.


2-Preterm labor.
- Hydrocephalus.
Polyhydramnios.
3-Accidental
1- Extended legs (commonest) - ROM
hemorrhage.
Z-Large fetus, - Before the 36th week
3- Deep engagement of the 4-Knots of the cord.
breech.
4- Undiagnosed twins. 5-Cord presentation Elderly PG or MG with
5- Short cord. and prolapse. no living children.
6- | Liquor amount. - Severe degree o
6-Fetal shock &, contracted pelvis.
death (0.3%). - Antepartum hemorrhage.
7- trritable uterus. 7-- fryPEr
Hypertension
rErlDrurl
8- Uterine anomalies as septate 7-Rupture of uterus. l0- Scar in the uterus
uterus.
I l- The presence of any
9- Rigid abdominal wall
indication for CS.
10- Obesity '

o Hemorrhage: 18 %
( o Emergency C.S :7 %
D
o Transient fetal changes are 10 % & usually resolve in 5 minutes.
n t
RdS
E
Wnat aro me ty[es oluelslonP
1.- CDxtenra[ lcrsion: usually cephalic, that is the head becomes
the presenting part.
2- Anlcrnat podetis- _rcts-t-o-rt: the breech becomes the presenting
part. Internal version = from inside the uterus
3 - (Ripqtq
e, r t----
p q{aJic-"Tsrsio rt-
o Definition: Bringing down the breech through incompletely
dilated cervix, it resembles internal podalic version, but it
can be done early, in the first stage of labor when the
cervix admits 2 fingers.
o Indications:
.It is not done in modern obstetric
. Placenta praevia lateralis or marginalis ,
'.
. Scapula dL.hlYluJSJl(or acromion lr,-J y ,.r+tt;

I incidence:
20.5% (more common in MP & in pretermo Otsi.Yl,r v++ ljl)
I Position:
9 Scapulo-anterior (right & left)
o Scapulo-posterior (right & left)

9 ScoPulo-onterior positions are more common because the concavity of


the fetal abdomen fits the convexity of the maternal lumbar lordosis.

I Etiolosv:
__
o7P's

l. Soft tissue 1. Fetus:preterm, twins, CFMF . Main: malformed uterus e.g


obstruction 2. Placenta: Pl.previa septate (commonest cause in
2. pelvic tumors 3. Cord: short, around neck PG).
4. A.F: PHA or oligo HA
3. Bony: Auxillarv: pendulous abdomen
contracted pelvis (commonest cause in GMP)

d from side to side + J fundal height

oad from side to side

the period of amenorrhea

is to one side & lowers than breech in the

{ Pelvic Grip: Empty

! Auscultation: FHS is better heard at one side of the umbilicus

,"r^"trxtzrtr
"t,
nrI
IS I

I'"
xt
IS
S
,
SD
K
E ' .rqrrr
.'fir-r /1
(
xt
nt
I - lv equal
ly lensth
eoual in length t
the thumb i, ,.purut.
nl.lyEtlu.1ltrrrtrrtBtlr|-,r,.*:ilisseparate
s2
)(
Right & left ore known by the position of greot toe or thumb
i

sverse ) internal podalic version *


is, srnall fetus).

rrzszrS
I9 ef
-
inition:
The patient is allowed in labor wi

e The clinicol picture


- It is as obstructed labor di'''ll dJ'aillt+ qtq ,e

- Usually the membranes are rup


- + prolapsed edematous arm
- The baby is usually dead

o Complicotions:
- RuPture uterus ch'dl d|4'illl+
- Other complications of obstruc

9 iAonogement
- CS is the best blp ulrit" 6,t

I.
h: the membranes are intact

membranes are ruptured

gside the presenting part


amination
pected fetal distress.
able decelerations.

e p?esenting porf in the pelvis.

lyhydramnios or latrogenic after AROM

* Malpresentations (breech is the commonest cause) A Contracted pelvis.


* Small fetus. { Pelvic turnors as fibroid.
* Twins or polyhydramnios ,l Pl previa.
t.

OSIS:

spected in feto! distress.


ing) or non pulsating (dead)
presentation) or ruptured (prolapse)

ervol ) low mortality (5.5% if delivery

(
I
)
I
I
Protap*d umbiticsl cord

Fetus

Uterus

Umblllcr(
c0fd

(
0 c*tr CorJ pabyl Corl pn hyrc )(
b, ptt
1 fy
Ca,r'/
n'
D I
( I
E
-
-
Spasm of the vessels
spasm (f temperature)
temperature) a '(
Compression between the pelvic brim and the presenting.part.
presenting part.
I )
! A
there is no immediate danger for the fetus & ,or.
more
uterus I )
( e may try manual reposition inside the uterus

()
thl lEfbn

tucfGd
,orffi

. VariaUq ).rrlrrl,i^,
varialh *.- w,s".".e"
( Jecchrol,on, D-trt
co,'t'' pl"fir'"'/
?)ttttr,- qnequc.

iI l*nu"I.."::l:',o"iiuffu*7
l\t\ffi
ll r -,
lryt
Y s
t
S

\t
iLl\ n
R l-n S

I r*.tJ o'^ '


D ;l#l
S
4

S I
| ;Ni -p!.r,nt,';<'r'r
(nmu,n) p
r*^
(oaplut
hnpual ([b*ytu)
t
I S

z(
Do4
(
5i2
2. Prolonged labor leads to maternal dehldration, [ta distress & death
dehvdration, infections, fetal death
f,
D' |]:
3. Obstructed
13.
labor.
.'rt*;,rtl[l.ads
labor. II S,
4
( 4. Premature ROM
ll iilllll:',1:X,rence rive.v
d-livery. II tt
(
D* 5. Increased
15.

.,
oronerarive
the incidence of

bi*hinjuries
6. Maternal trauma as rupture uterus.
ll Y::::::',:':lT::':ti:':J::t','-,
rr
operative d-livery.

1t
I
7. lncrease incioence offetal birth injuries.
I
2( lI: I;:n:;:',HIilffi.;'
8. Anesthetic comolications.
S
men of a pregnant t in a single pregnancy.

et l/803 (frigfrest nc is 7$t is erlj rull rlj uls).


actice due to use of ovulation induction

s & constont ollover the world)


icol in genetic pottern but differ in
\ Fingerprants, iris prints & voice prints
-.r-i

! rncidence t. zs% t, l5% | t"t | <1% !

lPlocentot 2 | 1 I 1 I I I

ichorioni 2 i 1 i 1 i
...-.. 1 i

L.
I
.4nq,gr- ..L .. -.?-. . -.
Fetuses ! 5
i-I ?.. -i.. L - .. -. j-.. -.1-. . - i
-.. I seporote iconioined:
: Type : Di : M nic : Monochorionic: Conjoined :

!- -rt ! ai ! ioti. ! monoomniotic ! t*ins !

- Fertilization of 2 ova by 2 sperms ) dichorionic diamniotic


- The placenta may be fused in 50% of cases)
- Have the same or different sexes ) share characters as brothers & sisters
- Risk factors of binovular trvins:
! Personal history: f age, parity, heavy tall female, blood group O & A
! Past history: Ovulation inductions, recent stop. of COC (rebound effect)
S Family history: *ve & Racial (more in black > Caucasian > mongolian)
! Early US in pregnancy (20% disappear ) vanishing twin)

! Fertilization of 2 different ova of the same c14[g by 2 different sperm;


from separate sexual acts

S Fertilization of.2 different ova of effetd gldsby 2 different sperms

tuper-twinning: Twinning of 2 ova & elimination of one of them ) triplet


Dlzygotactwans

Dlplaent.L i4orct)!a<ar*.1-
dirmoaott< dlr.tmida<

rc SZ\Z\\YZN
duction:
KCI under US guide to J num
: infections, death of all embry

I') r:'pr"ll'I"='po"r"riir
uepnalrc uepnallc J) (

I'l't Cephalic Breeclr 20

I' Transverse transverse 5


: Diqgnorir 5-7 w, by 2 fetal heads in 2 perpendicular planes

t tingleton ) Vanishing twin.


20% continue as
g Diflerentiqtion "Tau (t) or Lambda (1") sign"
+ Lambda sign y (twin peak):
114 monozygotic). The dividing
a double layer (chorion &amnio
triangle like tongue of decidua (e
r[ Tau sign: a thin membr
2) 2nd trimerter: Malformation, Gro
3) 3'd trimester:
. IUFD
9 Fetal wellbeing, Doppler &
. Presentation & position
! Complications: twin to twi
4) lntrq-pqrtum: presentation assis

{B} Exclude CFMF


l. Fetal nuchal(ranslucency measure
2. Genetic amniocentesis: (in mothers

(C'I With other procedures


A. Chorionic villous sampling,
B. Numerical reduction
C. Version
t:

s Increased maternal mortPlitv: due to


! prcgnancy nsKs:
:ncreased pregnancY
Increased risks:
'/ Early: Abortion, hyperemesis, anemia
'/ Late:
F PE, DM, pyelonephritis, preterm I
F Pressure symptoms: dyspnea, dy
(compression of aorta & IVC),
F Ante-partum hemorrhage:
- Abruption (due to PIH and fo
- Placenta previa (due to large
I
ti
a
I
I
I
$
I

I
I
t
I
,/
\.,

P
tt sig^ \o f,/ sig,t T " -Lb J
C^?^f l,'c1lt31t

frl-l
- f ?t'rHR
Drt ,f e,Lorh'on 4IUFD

TRhE
Ir
ui.b qrF.
Arl
eahg h^r^F
. Preterm Iabor (50%)z Spont
polyhydramnios - APHI & i
. Malpresentation & non engagem
. Prolonged labor (inertia dud to ov
r large bag of lorewater, PROM &
/ Znd stage:
o Obstruction due to rnalpresentation
. Locked twins.
. Sudden decompression of the uter
separation (intra-parturn hge

/ 3rd stage: PPH


/ Puerperium: 2ry PPH,2ry anemia, s

! Increased abortion rate

) ' O fidurlngd'et1vslyt (
S l't fetus prolapsed co d D
! znd fetus: excess sedation, abrupt
manipulation, ltypoxia
! It is mainly due to LBW from preterm labor, IUGR, PE
! Other causes: placental or cord accident, operative manipulation, CFMF.
S Increased CFMF :2Yo malor - 4o/o minor
S

. .*"..g,, l/1000 twins


. Only occurs when the twin A is bre
.-m- Treatment:
o Disimpaction of general *.,r,-n.rl1
^nd t

I reformation: ait for spontaneous dilatation (if fetal '(


)
S occurred ) cs) D
D ' Large size of the 2'd : CS

xr
S - ertia:
ertra: evacuate tne
the' '
\ 7/

s)
') alpresentation or (

( o Very rare problem only in monozygotic twins )


o Treatment: CS the surgical correction
I7r
(ffi./?Ert
2r*(
ti$Ftt

IS
(r,2
Dt
?,
rF
,
Yt
.,?
_----
\==---/
, , . -
X
-i

Loct(d fwias

e lonS ,sf 1<ri o J bt h^reta 2


of ,yC;Vrl
is q c.ronthr

;() A? Jel,rtrcJ Il,n;n 11


Th, ollu n b7 c s
he( for't I
D
tL ri 1 ),
A/b
V lrtarh
({it
S/l
s2
,(

D(
s,
IS
x_wr
s rn u,tr, ( crrr,,r,rs #,Tl*'r:tiltil;;;"' "'"'
I W t

EU.Z
,
S A ^Lror,roko(l J Jrfl )

3o , Awil PDtt

x;.
q, :' AvoiJ 9s
( S- i^)

n
| *\,, a)V *u b'ry 'S

t
! 5rdElg{g: avoid retained placenta, PPH & Ex of the cord & placenta. t
s (Pucro
_.==a_ erium: - avoid 45 )

s 9lndicqtion of C.S.:
-_-----------)_
'/ lst is not cephalic, Monoamniotic
'/ >2 fetuses, Twins .2 kg,
/ Retained,2nd twin * formed cervix + fetal distress
,/ Locked Twins, Conjoint twins, Discordant twins (1" . 2'o )
{ Other indication for CS
z Indicotions of emercency C5:
o Delayed progress in labor
4 Fetal distress
d second twin. 2
s (111000 twins). TTT ) either unlock under GA or CS
t
s (Xanasemenl of retalned Znd t'vrrin

* GywCsqqinst qlClH:
+ Episiotomy vit K, avoid rapid delivery
I Forceps which is better than ventouse

+ super-rwining (mono-zygotic triplet)


+Trizygotic triplet
+ Twinning of 2 ova & elimination of I of them
I'
.;;l;; ;':-' . '-. ;-:.-:"-: D

j;
I
Distress
1^L^- nnnrr
labor, PROM, io
!
Death

eatment

(i
s
actions ofthe different parts o

mon in Primigravida, malpres

: UUS contracts irregularly


acted uterus:
ree ofthe above
E Whole uterus contracts on he fetus ) arrest al distress (
( o nertonic LUS: LUS contra ts f UUS relaxes ')

o History of the cause, prolonged labor & PROM.


n Peinfirl eonfrecfions low hack26hs. Pain iS befOre & during COntract

longed labor (see later), maternal distress.

? -"ii:i:Tl:i'l::'
ilffi:
between contractions is increased (hypertonic).
(,

^ _ *-,--^-^-+^+ ^-" )

7 PPH + retained placenta )


St,
$ o fetal distress & death - cord prolapse - injuries - sepsis. 'l
(r:'jrJr)

)ods
},^ If fully dilated cervix e
ijlllll Llull ,1,

o Very rapid delivery


)
DO

o Power: + hish pain threshotd


I
S fetus.
sma, fetus.
Small t
s.

ion decompression trauma)


) ; i.rur distress (
(

D
! Analgesics, don't give any oxytocin.
2r
o : Bandl's ring
o : Impending &iJ eJ" rupture uteru

I Pathogenesis:
o If obstruction occurs;
. The upper segment retracts &
. The Iower segment becomes m
. The ring of Bandl is in betwee
o The condition may lead to rupture u

! Etiology -o+lu: 3P's


o Possoges:
! Bonv obstruction: CPD, tumor
! Soft tissue obstruction: (dysto
/ Vulval: edema, varicosities
./ Vaginal: septum (longitudi
./ Cervical: ex dystocia, sten
./ Uterine: displacements, my
./ Ovarian tumors
o Possenger: Malpositions, malpres
multiple pregnancy
o Power: pendulous abdomen

I Clinical picture
o Symptoms:
! Usually Multipara with
! Strong frequent uterine contra
! Fetal movements usually
o Sions:
! General:
/ Exhaustion & dehydration;
. Coated tongue.
. Dry mouth & cracked I
uPf*{ ,ltlueat

bu'tr

ascenl uP'^ra' &' u) o


prD
lt,zy.r?/1 t\e Q I
"t<
-1
:
[ roid 6.
r()r(I & Lonsrncrron
Constriction nng
ring 2
?
S/, s

.t ---- atony),
e or atony), ,loo Dlstress
Distress
D' I I '(,

( I' to prolonged labor,


labor, PROM,
l'' oeatn
Death
I )

n'
\(
SD
a

nt .1:.
'rntranatu, care
) t 'rntranatal
rmptv bladder
( | - Empty
..
1Y11.:
inh ib ition)
&
-a ^4I^L

t
rectum (J
^-

_ l.
reflex
.uood
uood management or
L,u,rt."t:ntations rie'
malpresentations as.
ot
as breech, face,
o'"* & transverse lie.
brow
l (
| )
I

X ---o,,0."i,U:l?J)un*,.,r;
I It

x'
Good analgesia (J anxiety)
I
Correct dehydration & give antibiotics.
- Proper use of ecbolics.
nl.i,""T:::t'Ji::i:nfgiveantibiotics I ll
(\u)t
iI \\tht(\ s

(
ilo ///
./ :::-.,3L"Y
gJtt.tlutr-2 )')
(N,) fn,.t-
C*rc oF n hr FYr..rh','^
^C obshuchon S
'
I
-""^1- u. Ftr b PV S

)e
; annurar deta
J::I'"
I S
SD
D?
( I Definition )
I
I
I
)
a

I
I
)
I

ne or more of the diameters is reduced by I

e or more of the diameters is reduced to a


g with the normal mechanism of labor.

etter ANC)

e alum) ) oblique pelvis.


o ala)
sacrum is made of 6 vertebrae)
acrum is 4 vertebrae). Not a cause of CPD.
the pubic bones & the gap is bridged by
ica).
m which encroaches on the pelvic cavity

column:

f last lumbar vertebra on the promontory)

pressure of body weight, the ischial


a yields outwards ) | Tr + IAP diameter

alking ) trifoliate pelvis


lvis) promontory is pushed forwards under
pressed inwards during walking
es in hip joint (TB arthritis ) arthrodynia)

v.o cation of hiP joint


N^
4 Usually unilateral ) oblique pelvis (the obliquity is on normal side)
Types of femole pelvis
(Coldwell & Moloy rodioloqico! clossificotion\
rrrrrr rrrll llltIlItItIlIIlltIlrIrlttrIrIIlr

: : Gynecoid ! Android
:
tal

llMttrrrr
TITITIIITTTIII!IIIIIII
: 50% .?o% (f
rlrlrrrrrlrrrrrllllrrrrrrrrlt IITIII'I

:I :I !t tt
Fernale I Male like
r ra r r r t I I ll I r r r I r r I r t l r t
llr

lshope :
rftf\
lla-
!lr-
: A
rll
rll
!rrrrraraaaaarJarr rrrrrrrrr..J.... rrlrrrrrrr]....r.r....rrr{..rr
t ! r . AP> !_
i Inlet ! Circular ! Tria ular ilr rrrrr,runru.rr. ! Tran
r.. r... r r. r r r r-r . rrl rr rr i rr rrrrr- r r rr r
r-tl
isuu-Pubici wio. rqoo ! Narror, <70,
: onole :
rr r r fr r r r r r r r lrrr rr rrr rrr rr
:
r rr rrr rr r rr !r!!r
lPelvic iwia.a iNarrow&
i.:g.ri.ty. .. . ..i.tn f ]:l:......i.o,tto.........
: : Short I FIat upper
lSacrum ! con.or. ! and lower pa
l. . . . . . . .' . ., . f ,Y.i.d.t . . . . . . .'1 j yi l'Lq .1 9i
! Pel.vic I pararrer i conr.rg.n,
! wolls : :
"'i'li;?' " " " "1
il#"i'
Ir ' r Dromiltent ;;;;.;"'
rr
.
!
.-j_ -^..e :proJectlnwar
i.: l: l.u.t, . . . . . i .f] g'.',,i r.E .tlr, . .r . . . .. . . . . .. ..
ifSO
!fSO
lll ltt tlllltllatrrllI
!wiae rlllllltlltlllrlllll
!Reduced

I Diagnosis:
o Symptoms:
+ Past history: rickets, fractures,
a Obstetric historv: difficult,
neonatal injury or death

o Exominotion.
4
/ APPcarance:
> ffeminine appearanc
P $fosculine appearar
) ()lstocta dlstropfrl
hirsute,
.ru!v, Jv f-ertility,
rvrrrrrrJ, andr
srr
(
7/\#
) Anh;l ytt,i1

D
r-a
( '.)
* pendulous abdomen / Malpresentations / non-eng. in the E

D o speciol tests: $
sr=)/
t

\7zaswz\\7ZaNZ\\Z\\Z\\7\\Zt\\Za\Za\z\]y1v\\z(
Vr

Sytpfiyls Pr.{is

i
i)
K o $(ost important in breech presentation

) o (|oti small BPD & longer APD


(

N rlaatrrrrra.raarrrataraarrarrrra.tr 7

2 i
S :
Deqree orcP i ^^H,*^ S
7/

I !*=minordegree:1*r z' t
'$

) roa..ut. " | S-ecm


i*=-.conlugate I
al
a ;c.tyzv/3$,?'v(/a/c*,t37,?2F/29?Dj2r)u."rrra
'! 6-8cm --- :.
2nd = head overrides : -
.
: ;
,AP/4/E/4/,,
theSP :-De
,r4
U
I <6cm I a a
2ndr I tt t t tr ll :CS
r r rl r r l r I r r t t I t I f rr r r r r t ta ! at lt a rrrtt
? hanJ p"shn,
-tl. (6t"1 hc-"{

hqnJ 6clia;
'i'[re S.t.

e han J 6rL
t.P

frt{ hc'J
ll. Tdtt Ao^l Qcl
iaf. - -il, f't' 4
eaXaSta,.c,,l

AanJ 1uh;Y .+ F,tryTt t'P.


-lk frt^t hcnJ ialo
-lh. pcllt

,f rl;ullc^ Ui f*
t\44P
,
Hr
ffi)
hu.l ;t plush henl i
witl 6ufet L,rl<r over-ri
alThe,r'P. c.

**(
I engqgemsnt or the h S

(o t

) s Asphyxia (due to prolonged or obstructed labor - cord prolapse) ?


( t Iniuries & intracranial hemorrhage
D
'
,
S
'l -lsto"'l*-,
-L
)' - ro antrerttJ K
t
K outlct (
n
(D l/ll \t\/ ,_,
(

II Y Y,':t'::!!
\ ldurt+t
)
S

s
S NARROW ) forceps + generous episiotomy

? q Q6tique pe[vis;
x'
|. : ?,r,',ion:
Gappi
Def icient perineum

) f ffreotment: Posterior colpo-perineorrhaphy if symptomatic (


r\
I I
71
Perineal teors
lCItiorogl:
Perineol teors '
! t

. Y CXlllC | .
Lozenge
CIP:skiniffi:: :

'....,...:
P"js
'l -.1
AelA

a
Ct -e
i

^dP
,J,,.
eue

I
l/&n ,-, ..fll i4_oS

'o

Submcnli
nriicol
lr.r
)rt)_
/

k
{ l\rL, lur .touls rllErl uurl r EDruuE ur
r Enema or Suppositories are contrai
z Avoid intercourse for 2 m & pregna

n
4. Treatment :
- Prophylaxis: proper managoment of the 2nd stage
- Active :
o Resuscitation
o Immediate repair: vaginal suturing from above down under GA
o Do not miss the paraurethral tears
. If bladder is injured ) indwelling catheter for 2 weeks
o If failed ) vaginal pack + catheter + antibiotics for 24 hrs
. If failed ) bilateral internal iliac artery ligation

e Fh{sto-fo-Aiet[te*an leads to slit shaped cervix(< 2cm). +\-l,+ &

s In the power: precipitate labor


! In the passenqer: macrosomia, Malpresentation & mal-position
! In the passaqes: edema & scars (Cx dystocia).
! Iatrogenic: as forceps, Ventouse or manual dilatation of the Cervix.
)o

a '/ '1 ' ^' side)


sloe)
(obstructed labor or Ventouse)
lip tightly applied between fetal head & pelvic
le (after cerclage)

(cervicitis & parametritis), ureteric injury.


, ectropion ) leucorrhea

by 4 ring forceps -l Sirns' / Auvard's retractor


o TTT of complications:
1 Infections ) Antibiotics
! Patulous Os: )
Cerclage &
if Pregnant if not ) Trachelorrhaphy
g Cerryical dystocia) Cesarean section
I
!. I rreatm
I rtuMe-rcci t

n'
D
r\
ulation of the uterus (
/t

n'
D(
()
\

ute"''
6or^ptett rugTurc

-,{, / t)scf
D Car
sSCar (alr, (
s -,LtkucteJ
ob.5l?:^':'..^':!*"
t

7/
D- t )uooen paln (may D0 relerr shoulders + nausea &, ,1,

( \ I:Il'::l::::::Ti co,apse )
S USCS or 2 LSCS always CS (electlve at JU w), prevlous repalr oue
( all-through
rupture should be hospitalized
)
s^v\

rP ufll
t'r" ^'*n
?
IS
.o1 0
/-ot
letrtfcitalioa -

t-,,: xte t;t

..-'/ ut'i; rz.Ar n


:':;.

rY
^raiA
+ Y" ,,t'
tattat tla rt iataaaa a a aa aa a a a lt a aa aaaa alll

durlnolabot
ma d-urln
m-s Olshsr ?
Q :
:
:
a
a
nes ...rupture SBcoccyx, ant sacroiliac!
:
a

muscle
mrrscle !:
E :;: o.. il;;;;;;il;;.*;;ilTl;
S
\
t-l

:: .
aa
Due to rupture of the superior deep epigastric
MP
Mnre cnmmon in
ryIOte commo[
More common lll MP
lvrr
vessels :
.:
,,l:
)Ia.
: . May occur after cesarean section. :I
a
a
a
a
a

:l:l Vagina
Vagina I Cervix
Cervix I uterus
Uterus l:
a
Dyspareunia Chronic infection Hysterectomy I
a
ta
ll
r'
t'
rl
!;rnconttnence
lnconunence rtvrr LL | ' - - a
ll
a ;rrwrrrr a
Fistula Cervical dystocia Ureteric comp. I
a
I

treunra'l
a

-
Whet ere
:ry6et the^[ate-sgglueJqe
the^[ate-pggpeJq-e gf Js'lator"anl
9[ leioator.anl treqla? :

E
I
l'
o Prolapse (utbrine, vaginal) :
ce (urinarv. anal)
.. r'r r r r r r-rir rrr i r r rrr r
ss
,
s
:t

I : shock (hypovolemic or neurogenic from Crede's)


-) t(
)a o Abdominal: fundal level 2 fingers above the umbilicus
? { , E/y,lr" uv prolapsing
mayJ be Prvroyorrr6 rrvrrr Lrrv vagina
from the v46rrra )
""td
,--:*(
f/0, b' J
,hn,,l
I
a I he/(^Q

Hour-gloss comtriclion thar;1 e ff. "l-


-

ir

,l
Nc blcclial H

**(
Brrall crele'r lltlArt t
/,nhc"''
ilethoJ

H anuaI
f ern O tJ a-L
ef -t ne-
P lqctJ^

a))u'
flwol lcyrah'oa ,fb - '/P
maaiv? f,eb nohnol 'f
trras kr
t

I'
I ! Definition i actions t
pa ^J€ /
L

') ;:
D
(
tlre uterus ;l;;
;;.;;;;; i,,*,--
; large.
is lax &
(
),
9-
I Monooement S
2
o Qfep[rfIsXiEi Avoid predisposing tactors: ante-natal, intra-natal & post-natal.
,iii--
'(,

!
)
Resuscitation: large bore IV cannula --- FB. FFP & IVF
! for any surgical intervention
!
e Sp-ee-{ier
l"' Iine cPl+Jt
! Examine the placenta * Massage * Evacuation of bladder + Ecbolics
- Oxytocin 20 units IV drip
- Methylergonovin (methergine) 0.2 rng IM (not in hypertensive patients)
- Misoprostone 800 - 1000 micrograms rectally
- PGFzcr 0.5 mg intramyometrial or IM (not in asthmatic patients)
2{ [lne ) CI(IA
! If trauma treat as TPPH * Evacuate blood clot
! Bimanual compression
3'd [ine ) [aparotom2
! If the patient conrpleted her family: Supra-vaginal Hysterectomy
! If not:
) Bilateral uterine & ovarian arteryr ligation.
} Internal iliac arterv Iieation
- Difficult, ffi&y injure the ureter, iliac vein
- J Pulse pressure (becomes venous flow) & 70% J of bleeding
- Uterine blood supply will then depend on collaterals.
P Direct uterine massage, hot foments, Enzaprost (intramyometrial PG)
) If failed ) Supra-vaginal hysterectomy
(Qcenttl
! E-Lynch operation (Brace suture)
s Hydrostatic intrauterine Ealloon tamponade
! tilateral nterine a. embolization (by polyvinyl alcohol particles)

Old methods
! Uterine douches by Bozeman clouches using warm saline 37 -38 "c
! Utero- Vaqinal Pack * Abdominal binders
! Blunt curettage
\iza\\zaNzeNvz\\z\\7\\z-\
D,.
st) js f rt tct lal g1q 6 6ch; a
^u

er5onelrh

oxyfocia

_ :cn l-e
, ^/ Ft7Rc ?lat

*8ftya't:y
'
II '1a|f^t-t^))ce
-*e bb'lder
t t1t,t'4e
(2 '::"'T )l,"tn re 'n)
ern'n,.rr- s
E

3 @3 valiaaugs::,,*t",;"i';'i,"',i,

*no,r"q^o\
n . t ,iajur,1 oF

|) v
@ 3 fifidon^4 K\ '.,i,^^in#?--r]
^i*..* o.l,r^t,., ,s.w, (t
\rl'crc"'otf I )
( ?Ac ) E"'. , ,..,- r',,* I D

D (
'zirin \---
roh'o7'in
oo"n'7)u
aot;u7,in
^b Wa{,
)jltl cll ot
tfr ( brace
t./or-l
t{enl 1

,-/
- y0 bolloo

2 u/ rY s
:t

o Thronrboembolism: pulmouary, renal, mes


s Blood doesn't clot

r:zzr.vzurrzszlrzrrzr
D l' cDqc^;lt^l'-
e,
Sl/
N S

/ Vascular wall weakness


k rmophilia Aloex linked recessive)

3t
i I
,,^,/\-L./\ -^,-* fetus,
S

t can cross the placenta affect the


S
I i !
6 Prolonged:
ff,klix-illiT'J3,*T[i:'J:ffi,)<100'000imr)
ml (N: l0 pglml) & D dimer (N < 0.5 pglml) t
.t Bleeding time (N: 2-4 min), tclotting time (N: 6-12 min)
. t pf $: 12 sec) &tPartial thromboplastin time (N: 35-45 sec)

nr
D s
s

)
il.'#ffi(.;ffi;,".;.ffi;'
o Failure of the uterus to return to pre-pregnancy size within 6 w postpartum
( i cuur.,
t
(
D

! Y,r4ttg

I Treotment
o Ergometrin
e Curettaue if retained placental fragment
I @ fL\! t

\\ irrrrrrrrrrrrr..a.aaaaaa.aa.attrrtlrrrrrrllrrlltrritttltraaaaaaaar.ar...rJrrrllllltttrtrrrrrrrrrtrrrri 74

'1, !""""""';'::I:t:'^tr';.;i'D;+""""""'f "'r"""""i::':'::;til;';+'..'.....r....!

, 'lt )
"""""'!
.
D!itttlTissuethromboplastin . ! i?t
L ;X 1 !E
D' ! tv lcalcium : xt i in antecedent ! (
( !y ! Pro-accelerin (labial factor) : XU i Hrg.run factor !)
:tor i(
( iuiPro-convertin i : :)

| Cases o-Sr-tfy ots-, t


I
,
I

A. ldiopathic (True) obstetric shock


it t2
S ! causes:
g. obstetric
2 :
2r.ffi^l...l..L.-^..L...:......:^-^t..r....^!Il
i. lry or 2ry postpartuur hemorrhage, inversion of uterus :
"
) i.
S
N : compticated ovarian cyst. |
bolism & comptlcated
Doltsm
(
,.
i
-- I --L r--r' Ar
)$ ii
S :
',
lii,
tl,i,,li;ffiHl
Anestheticcomp"--r!

Shoc
Endocrinal yroidism,
i:
i ')
(
')

2zrvszrrzrrzt
Acute 1i3000-l130000 Chronic

Causes
l. Spontaneous (precipitate labor , short 1. senile
cord) 2. fundal Tumors
2. Induced (Crede's, traction of the cord with 3. chronic on top of
atonic uterus). It is more common missed acute

Symptoms:
- Severe lower abdominal pain
- Something protruding from the vulva
. PPH
Pain
Sisns:
Bleeding
CIP . Generally )Shock (hge or pain)
Discharge
r Abdominal: Maybe infertility
- l't & 2nd degree) cupping
- 3td ) uterus isn't felt
. ry. znd &,3'd degree: large mass covered
by endometrium + placenta + Bleeding

.1" degree: cupping ofthe fundus


Types .2nd degree: the fundus protrudes into the vagina
.3'd degree: the fundus protrude through the vulva
o Resuscitation, Manual reposition under
GEA then remove the placenta
Reposition
o Tocolytic before reposition
TTT o Ecbolics after reposition.
o Antibiotics + pack
@*
o hysterectomy or
o Division of the Cx may be needed
o Avelling repositer

I'
o Hydrostatic method may be used
e, 3E\---
.'v tv l-, =- ?-Jt,
i-"6N: :;'
I S

ns
s(
,,
sE
N71
E'ttr (
s\
ns
D

s,
ic /-/o
7ld;of"
3 / /'
,1,'o t
X /
ilaftrnot?'Drl
lturttttv--\p,l t
I,
s ^.-A ,(
S D
-t-t
n -ft. t. t

n \r t
ulruiluililgrutila \rnf\ ltl JU70 ul uasgs/.

romnios:

(I0 times more common than acute)

an the qnlount

::o Rapidlv progressive (days) :! o Gradually proqressive


!o Severe Pressure symptoms: :: (rveeks)
Symptoms
dyspnea, palpitations, abd ;: o Mild Pressure svmptoms. i
dyspepsia & LL swelhng
pain, dyspepsra
parn, swelling ::
G: LL edema, varicosities, increase weight, * / - Pre-eclampsia
Abd: The abdonren is markedly enlarged rvith shiny skin
/ Tense abdomen, Fluid thrill is present
Signs / Mal-presentations are comnton

lq a-,04-r;

tal life, multiple pregnancy

the position & presentation of the fetus.

! (I h0ur rl'BG), RH typing, fetoprotein for ONTD


lests tor DI\'l
! Amniocentesis: bilirubin concentration, fetal karyotyping (takes 2
rveeks), congenitaI infections by PCR & fetal metabolic disorders.
o Routine investigations:
lmniolic lluiil inilex tec[niuue

1. While the patient is supine, divide the uterus into 4 quadrants


using the maternal sagittal midline vertically & a transverse line
approximately half way between the symphysis & the fundus.
2. The transducer must be kept parallel to the maternal sagittal plane
3. The largest vertical pocket is measured in each 4 quaclrants.
4. The sum of 4 pockets = AFI.
5. If it is < 8 cm repeat the evaluation 3 times & take the average.
ith s[ow drolnage of llqvor to alotd:
o Splanchnic shock
o Abruptio placenta
o Sudden alteration in fetal position
' cord ot"'"::"*
;)\ - S ls uo
isdonebyt
4one G^.-
bti t
A
d drainage of liquor by finger E
D- nrruvvaLtrr rupture by Drew Smythe catheter. '1,

D every 1 wks (larger amounts ) preterm labor). (


S relef (t ient effect) & decrease preterm labor. ')

5. fndomethocin: (PG synthetase inhibitors) (


2

r\ eL.rr urrtsurr4 (rsluPur4ry,r, urrtsuuyur4lrllrrlrJ .J, 7/


premature closure of ductus arteriosus'
k { Dcse: t
kt
| l:i:ffi;[1,'i,'#,*:"' l'
. ADortlon, rreterm labor o Lg_regrrhgqez
----u-
"':*, large I I

n'
- ff:'T::',,,*.

|
o Acute pyelonephritis.
. (Iel
(Ier os'a I
(25"1)
placenta
revia with
"":. I t
l'Xrr"te,sion
!^tturtension ^{:::Ti , ":.
l L-----1:^
-I^^^-1^
---:r1^ -.^.^ll

I'D(
()
. shoc-k mal-presentation
n I:
5[o_c*k Atonic PPH
I: fl::::.:il.,_ I t
I l: &";,"d,J I
o v-, * y, -r*yre lmar-nresentation PuerperaI sepsis
o
o
Arm prolapse
Inertia
l. ffi:il;:ilH' I t
Sub-involution

Ie t

(
,rzza:.vza:.vza:.vza\'wzalrzr
I Definition t
o the amniotic fluid that is
! Detected clinically
s <500m1
! The Amniotic fluid index (AFI) < 5 c I

! Less than 5th percentile at all gestational ages{.t)l

I Etiology
o PROM (the most common cause)bl-,j3 urjl
o lotter's syndrome : Renal agenesis + Facial abnormalities (Low set ears {bat
like), deficient ear cartilage, flat nose with a crease below the lower lips,
recession of the chin & joint contractures)lq teg.

o llacental insufficiency, Pp A IUGR


o lostmaturity
o ldiopathic, after amniocentesis & long use of antiPG
I Clinicol Picture:
e SlmP-loms:
. PROM, PE
- Reduction in fetal movement

e S!SaE!
s Abdominallv:
./ Fundal level: less than the date of amenorrhea
(
D
I ('//nho; )) t
I\Cilu'"-7/ ^At
I Y?R"u ff t

Il/H,,t
./'l ) t

I h, ,"o/
It)lr
I Vn
lt.,"nufd,.,

lo!,^a,nlu';f7
t

I frnh l.qhnl;nt |
'
$
o
I terminate by ARoM * oxytocin S

n. I

Jl']'_'_'i :T':_"1_
I
I
I

I j. o..t-^narrr lrrraanlacir r 7ol^ due tO


) i '' lung with inhibition of lung growth i (
( 1r,,, n svndrome 10-15o/o: cranial, facial, skeletal abnormalities j ,
(t
\\ u : ursruprrolr or rsral memDranes Delore onset oI uterlne /1

kt
s trSrrarrurEJ r.rtr!"urc J / writrr\s 71

so),
)(
( c ic lnnql inflammatory reaction with release of: ')
ic enzymes dissolve the membranes
eterm labor

- Gardnerella vaginalis, gonorrhea, urea


S
S luri
IUt(-
lutc- onJ'to{ 'ott'
,,A te
( t conht )(
EA-',1
I ',n'|.
u $

IrA
('r"!'9 $

I ;7reter1 $

s | \-cI t | 'f '4

tft
s,
) i''"e,s
D,*
s t
S

all'i
') t\tr u,r s

t/s)
t"r/
(\,,-..)

uledx rYnmbraaet ,j{ -' " -


!
ot "l t
I Tahtour $

2 svzlrvzswzlsvzl
I
o
! Labor and delivery occur in:
- 80% ofcases in the.success
- If delayed : Prolonged PR
(

! Spontaneous onset of labor is expec


- Three davs in 507o of cases,
- Within one week in 85%
- Within two weeks in 95o/o,

I Clinicol picture
o Symptoms 4rriJA-u Ctj.l;"3
t History of gush of watery vaginal d
! Preterm labor
! Infection
o Examination:
s General: infections (fever)

S Abdominal c* 6 2'-r.,.J uJ..r..o


-
J Fundal level
- Abdominal tenderness (if there is
- Fetus is well felt (if liquor is draine
- The fetus is easily heard unless distr
S PV:
- Not done to avoid infections except in tpYl qti ,/ eX
. Infections
. rermination (for cx changes,"T:i::s pelvic adequacy).
s (
z Trickling from the cervix (increase
posterior fornix & Meconium in the
v Fluid is collected over the lower bla
r If no fluid in the vagina sl-21 el '-::
o ASK
o Ask the patient to
tne patrent strain (]r
t0 s[falll or uc^
o Slight pressure on the uterus

s Special 6s^a3
\ Nitrazine paper
o Alkaline pH 7-7 .5 so the gr
o False results if blood, in
\ +ve fern test (Arborizations i.e hig
\ Nite hlue sulfate: detect desquama
S I
') Lqve'
La[or
s ''u-c^t
38 Cy 4 J^z? -r)
,- , -^t I?oa s
G"ton3zd
I)(
D (n/r\ t-v(
,
(,/)
S

t
Sferile Gttc4) G

g= -s;oJ
l- lllurvuru,4vlLllL, -Ys <rll LED
;)o
, 2- richomonas vaginalis,
rlis, candida candid" S
s 3- a, urgency,
fistula, SUI. Urine is acidic with A

1 f
et
t

s(
sE
,

D o
( ,..
c-L---
e fetus < JJ --. 1--(<
35 weeks
vYssr\J
,2
\\ ^
z2 inate)
rminate)
rurl,attr'' S
')
D ! No fetal;,;.rr
n(
S : TL- ^-+i^-+;^ -^f :- r-L^- s')
Sra\zavr\\zza\za\z\\z\
N
t) 1. , Ternrinotion
r\rlllrllr\.l.rrlr . . 1 U
A o Andicqliqxsl
--, \')
N

)s'D .eiafNeonotolsomplicotio (

n'
I

n
2.
r|
-^^t ^^^xti^^*r^n
, septic hock, DIC, ARDS, renal
Intrauterine: chorioamnionitis, p
Preterm Iabor
failure
t

3. Prolonged bed rest: DVT (avoided by


4
4. abruption 6% PROM (gene
Placental ahruntion (sene J,
(
zrffi
i Treatment accordins to gestational aqe

'[oo imnralure ! RDS ! Chorioamnionitis I Nothing :


.....................1......rrIfrrrrrIrr ...1..........,.....1
No : "l'T"l' of choicc : No :

TOP i No except if ..... ! 'fClP :


..o r. r.t.... rr r r.i r r r rr r r r r r.. r r r.f

Gauses oI accelelated lung maturity

1. PROM
7
Pre-eclamsia
3. IUGR
4. Intrauterine infection
5. Corticosteroids therapy for mother
Gauses of delayed lung matulity
1. DM
2. Congenital infections
3. Asphyxia due to
a. Cord prolapse & compression
b. Obstructed labor
c. Infection
IUGIf & Prolonsed oligohydraminos

7zr {
Srl\za\z.\v^va\Z\\z\\Z\\Z-\\z\\z\\7za\\za\\rza\\7za\\7z)
Chaii6rrnrnianil;l lr^ ,.3-o dl-!-, )Sr.
x.
evere fetal & NN affection (due to release of
S damage
reaplasma urealyticum, Staph, Gardnerella,
ause bacteremi a@), Clostridia.
horioamnionitis without ROM@)

ternal fever lq e46-c

n l2
---^-----n, I ) -3
- i davq
days hefore
before fever\
fever)
s ROM)

- Eorly sign of infection in BPP is loss of breathing movement


- in CTG loss of variability, basal tachycardia
as gastric aspirate wood culture.
)
s
o Moternol:
! On preqnancy ) spread
) ! On labor ) prolonged labor
( ! On puerperium ) endometritis Asherman's $, sepsis, PPH
o Fetol:
gitis, funitis, hemolytic anemia

cin, metronidazole@ till the results of C&S

Z ) most preferred
S !s z f^+^I A;^+-^^^ &
rn fetal distress
- e-
[ fetal exposure to infections (but I maternal
spread of infection)
o Core of neonote
q TTT of comPlicotions ' i.,r[
(
nrt
n\'
n ' t-'t'ot'ooPlvri '''\ t

n \l /l\Drc t

I t-rtPho
{ deatl
I sho ck
,
sr
E NN condition due to destruction of RBCS by maternal antibodies.
s
/mo,n/ of rtA.Uohr,_l
l;,xt it lnlr.rcel Leh

phc,nra, t g;jA"NeJ,,,,-tot-
I

icfen-Jral'r
L^^u@uo^'*'Y*" 3

\t )
7,
7 \vz^\vz- (
k'I) t

n " "u":"iiftl:';ff1',11,'';:$#H,'3"il,fT#.x::I?weeks t
r\4

S 3) sis: ),
{ Timing: 10 weeks before expected event (not before 18-20 weeks) S
\ ,/ measuring the chsnge in opticql
Method: measuflng denrity of Al'
opticol dentity against //
AF agal.nst f
'e wave length 450 nm (Lilev's chart) & also L/S ratio.:
.,

D$
r\
$
7t

I'(E
SL
)(
-
n | I lu:+*)roP I
Ds
S r'
U:+*)ToP

then
t
11

r'
( ssment of fetal wellbeing )

,;ffitris(*a-{oGA9Q'
\ t7
nat2Swks
n at 28 wks S
given as soon as possible in he l't
I't 7j
72 hours of labor I
300 pg, sufficient for most deliveries
28 weeks
#:iiji:l[T :: itT"i:iJ'"-"i t
, tst Yis;f

?x
lu- 5rn1 I
ir g(-'*l 4 I (trc)
+
7* oP trasih'Z. Cro .tS t., t u .u ,)L)

ttd
,u rLRsk
,Jf,
llefC .l a ri tk
a

, HP -oAV,f : I
- .L), ( TnJ;rrt+ C,o',b', )
I
Araale 6,

Fvit
'Pubs
- felaL Wctl bcin
E,^t b. uret of fqilure of qnti D prophylqxir:
'0 I roo small dose, Too late, bad quality or the patient is already immunized

SZN \ZSZ\\ZN r\Z\Vl\ZlI SZN \z\\:zrsz^s


(rilS
k o -'- \Zonis( t
I{l Rp,^t€nry W $

) - il latau aborc Tle achon l;ne - ^'-l;Fc&'n-'( l'\i clat S


4- . ,L.t Dau
Tf' ftelt
sl;verY
/ " '1;1 bil t't'l n''ct
"l
-lv

SJI 1'r^ Z
)SrE 1o
s
s
ne contractions & cervical chang

&
ng 37th week (37 X7 :259days)
2.5kg, very LBW <1.5 kg & ext

of pregnancy is }{AZARDOV5

(40%), Low matem


pre-eclampsia, renal disease

gical as SLE or anti-phospholipi


al as diabetes
s, hypo-plastic uterus or septum
FMF,IUFD, Twins,

f PNMR other than CFMF

"soft skull, fragile

atelectesis, RDS & pneumo


mia due to J iron stores in fetal li
fibroplasias if the baby is given
d physiological jaundice due to

al rate (75% at lkg, 97% at 1.5


ental deficits
to Jsuckling + poor stores *
(k;l,u, Bef Ke

ktl
, Da,*1ul W ?Sct

38ut 90,lr
{r+"t l/iaL;l;,
arln^cototT oF T, ,oLl ltcstt
ISAS
R
I
l.:
I *I,t
oy
tat'

a I
with weak cry, rveak or a
rt wirh
I S

f
I
D
(
I
* '- -
;i L
'.'.
.l T
.r
Inegular respiration with attacks of apnea & cyarosis
liiT::lir"f-i#llilffff;mfr;n:::l*a,
T' -r r ' -'-kled
skin due to lack of subcutaneous fat
:;ffi',ouelora'Kolsuocutaneouslal
caseosa
- - r rt'' rne
"r"
I
I
f(
ilo I| ;; I rs < Jz w, lunugo nau ls
is<32w,lunugohairis thebody
ooqy
thebody II )/l
V | * Fingernailsaresoft&shortanddo offurger
*Fingernailsaresoft&shortanddo offurger I S
.\
so |I t Unstable
q.
+Unstabletrmperatureduetoincomplet
unsraDle r(mperarure
trmperature oue to lncompler
due r0 incomplet ' 'rr I ' r" ' l/E
V. I centre, lack of subcutaneous fat & | mu
mu

NE S
I'I
n' ( Po.Sf- n\oluru

r-l
A CI,Qru(-eJ
CAQL?er ) t

'S

t 40 weeks.
/,:,?,,,-lil.Il'X,lo
I il
t..lttrin. sphingomytin L/S
i'-*.X1tj','*'ngomvtinL/s
ratio 4: I or more

| ill ZSO
it ffilJii":lteconum I il t
nrl at 42- 43 wk.
tu
.:lor. ir.gr:en dt Meconium

n I ,.
D ll r.
!,,
.,r,f, villi. llt
ll
uecrease in diameter and lengh of chorionic villi. 'd

xil"'
( ll 2tl. Fibrinoid necrosis, I atherosis of vessels, Calcium deposition & white infarcts.
+
D n /I Wrinkled caseosa.
wrinkled skin and no vernix caseosa.
( ll ,/ Long hair, long finger nails, greenish staining skin due to Meconium.
Lonrhair, Meconium.
ll '(
ll )
infarcts.
ll )
ilt
/
D ll ' ;;;r* suture
Closure orsuture
of line in the skull.
skull. ll (
( ll ,r' iorpt.,.
Complete closure of posterior fontanel & lsize of anterior fontanel.
fontanel. ll )
D ll '/ *r,'rroo'ished
Well established centre
ossification centre (
Sr'a\7\\7/a\z\\zN7 \za
f.,.-sh--"--.+.rqt

TNTRAUTERTNE GROWTH RE$TRICTTON (rUGR)


:: 15o/o7 b-?s.?s. 4+"AYl ,t a,r-i

t trre abiritv to grow (birth weight < r0th percent,e or


I \ L Ot-/ Utrrulr r,iltr ultralr,, ''
I;Tfi5:I;:T:;1,"' ,(
$
D-
K
s $mattfor Bsstetiqn s,I
qge is a baby w th a birth weight < l0th percentile as )

n' 75 % (most common)o@ t5 -20% r0%

: pre-eclampsia, renal disease, cardiac diseases


s.

as SLE or anti-phospholipids
al as diabetes

ations
(bacterial as TB, $, viral as CMV, parvovir
as toxoplasma, malaria)
pregnancies, exposure to teratogens.
ei

, abruptio placenta, placenta previa, place


9 The prognosis is similar to those with normal weight
t
D 2'
,
these are abnormal features during fetal and postnatal life
----'--e r---'------'-- k
s ric E

N:^: t""'ii;il;;;;t"""i"iliiil;'ilii;il;il;;;'iiliil;'l"th;;;1;ffii; ": D


') i
(<
growth i same slow rate ! parts with brain sparing effect :
r.-....-...:.-...._rrr.....r..r.]rrrr..rr.rrrrrr..r...r..rrrrrrrrrr.r..rr.rr...r
]..........,...rr...rr..r.Jr
l) :l.! ... Extrinsic (Maternal/Placental) :I
j"""""" ""-"il;il;;i""
I*:*:i: lY.lle*r'll ""-
"" ''.'-.:
!?:.?::?!)...i
"" "': 2
: di.;;;;t
...........i...1........-......i.......!.....,.:.,.'............:,
,,r...r..,,...!
N
R) ! Good with catch up phenomenon !

) Dg erot-: (
S J Weight gain especially i ')
- Preconceptional weight is <50 kg (
( - Failure to gain weight in pregnancy
D
S
";;;;;l;;;ffi#;ffi,
nersrr
lJlundalheight
)
(
')

n t or"ri,rrioar t
t Fundal level < period of amenorrhea,
- Normally between 20 &.36 w, fundal height : gestational age
,-,
i+ Proohvlaxis
! Prepregnancv care: good diet enriched with folate.
t Avoid the causes of IUGR
! Low dose Aspirin 81 mg HRG) | Placental thrombosis & | the circulation.
e.
s Indication:
. if immature fetus, the patient is not in labor, no fetal distress
. No CFMF (l't to be excluded)
s Method (hospitalization):
. Bed rert (left lateral position), good diet, sedation & steroids
. Treqt the cqure if porrible os PE
. Obrervqtion by
5€RIAL Ul5 every 2weeks
F<TAL W€tL B€ING:
. Daily kick Chart
. Twice weekly CTG
. Once weekly Doppler
. If non reassuring results do BPP
o centes is

. il*',Yf H#ll'J,:ilIll
o Medical
- Low dose aspirin & anti-platelets ([ placental thrombosis).
- Heparin (5000 units SC/l2hrs)
- Antioxidants (vitamin C & E)ot pX.ll qr1
6 Surgical:
- Amnioinfusion of amino acids, glucose & Thyroxin.

S hydramnios & CFMF

?!
D Management of the IUGR according to the
'1, i.i ..........:.......i.;;;ft..............:..........d2
sestational age
2 weeks .............( z 32 -_.;.;;;ft..........i....;,.i;...:
36 weeks i >36 w
.'..........1................1:
ization !1. Fetal surveillance I Delivery :
!2. Amniocentesis L/S i. ,
kly BPP,: ratio t t
Doppte, i3. Delivery when : :
ratio ! indicated
is L/S : !
teroids : : !
whenrr"""rr"ry. !
rr Jr rr rr
rrrr r r
i
..........t....r.....r......
!

R at preterm: the management depending on comparing the


ontinuing pregnancy & preterm delivery. L/S ratio of 2or more
ion of deliwerv

? "':-l--l--:":lllll*

7 swz svz svzrsvzrs


! Method, (
. Continuour fetol monitoring by 2

t ldentificotion qnd monogement


. Hypothermis: due to relatively la
. Alphyxiq neonotorum: due to
o Extension to IU asphyxia
o Meconium aspiration.
I Hypoglycemio due to
o Poor glycogen reserve in
o Exaggerated by chills fro
. Hypocqlcemiq: manifested by clo
o Hemorrhogic tendency: cause pu
o ttunted growth & mentql retqr
s Rerurcitotion in lCU
: Early feeding/ 2 hours, glucose (o
9 Ca gluconate, MgSO4, vitamin K
9 Heating
. Care for RDS
o Proper & regular follow up especi

I
t Soft tissue wasting
! Loose skin, thin with little subcutaneous fat
t Scaphoid abdomen
! Decrease the amount of liquor with yellow or green tinge
! The placenta is small with increased numbers of infarcts.
I'

t$
I
snouloer oyslocla
.irllltylr
#::ffi lllllilL.,,", I *birth
injury
,_,,^ lt
I$
ny due to overdistension) n L - -ir-- 7 -z-- i.- 1'-t^
t i........i;:.::r.j:.-.^1i.:.;.ji::::]!:.:1.-.::rr...t............,..rr..!.......rr..x.........rrr..........r.,- t

tl
n

II -
II Iltfl "T; u,/--
I

f',

II Ll/ot;l ,l \

n
D(
()
l_-..r I I r
: if
estirnated fetal
or previous shoulder dystocia or Erb's

Dyltociq O.l5 - 1,7 r"irl -tr rLeJl)B-)S-


houlder (11.5 cm) after delivery of the head
p tightly against perineum or mother's thighs
ration phase (9-l0cm) t prolonged 2nd s[age.

4.000 grn and 22 times if > 4.500 gm

Maternal complications Fetal complications


! Prolonged labor. ! Asphyxia & Injuries (Erb's,
or humerus).

o experience better to perform CS,


,.fi ,' cott Fr Aetp s
I
t
(
Jencfou.'t
e Vsioto ^', ?,

+;$.
;tr
?,.--trrt:;"" t
yi;t il- ,

3+t
t\'
, ,-i.

(t
I'
n
n

2t
Peri-natal montalitv rate
;ilt
n one year
t

st
2t
k of st
Number or
t

Number of neonatal deaths

D$
SL
s'n(
v . AAA

st,,
v

)s
S)/
I3o%t
.lnl-ections '(,

. DIC: if Fibrinogen level < 100 mg yo (After month from fetal death) )

!
! Angulations of spine (Ball's sign) and re thorax
S
! Large placental shadow
. U / S: (main tool of diagnosis)
s Absent fetal movements & fetal heart sounds (F.H.S) with scalp edema
. Fibrinosen Ievel - PT and PTT - FDP's
I Treotment:
o prer_enllqu Avoiding causes, assessment of FWB & monitoring the fetal growth
o Actirg-t?selt"rql
s Ante-natal care (ANC)
! Spontaneous deliverv
- within 2 - 23 wks in 80% of patients
- Drawbacks: Psychological impact upon the mother & DIC
! Termination of pregnancy according to the fibrinogen level:
:"' """""

+Induction of vooinol delivery: +Give


o IV Oxytocin "effective near term" o FFP,
oPGE2 Suppository o Cryoprecipitate,
+C.5. or Hysterotomy: o Fibrinogen,
. If 2 previous C.S. o Heparin: Stop it after correction.
. Other obstetric causes +Evocuote dead products of
o Maternal indications i
t.r....r.r...
conception. :

s, After deliverv:
-- ,/
,{.utops1, photographs for full baby and X - ray
./ parlodet
,/ Qounsellng, emotional support & chromosomal analysis

a
-)

v,/,
Intrauterine asphyxta
Intracranial hemorrhage
Congenital pneumonia due to prolonged ROM
Birth trauma due to ruptured liver or spleen or fractured cervical spine

. Definition:
Death of the neonate in the l't 4 wks of life following delivery
. Etiology
Prematurity (50%)
Birth asphyxia
Birth injuries
Congenital abnormalities
Infections
Hemolytic disease of newborn
RDS

I Definition:
Failure of 02 & C02 exchange or to maintain acid base balance

! Chemo-recerrtor: due to high COz tension and low oxygen tension.


I Thermo-receptor: In skin, are stimulated by cold inhibited by heat.
! Baro-receptors: In aortic arch, sti
! Stretch receptors: In muscles and j

I Couses:
A. Maternal Causes
s Anemic ) in anemia.
! Hvpoxia ) in pulmonary disea
-l Stagnant ) heart failure
! Histotoxic ) alcohol, cyanide
B. Fetal Causes: as IUFD

I C/P z J f.tul movement, passage of m


S/rsza\z\\Zrv\vrN7ra

I Couses:
dru[tslnreP.cenlers:depressed by drugs or damaged(hg or edema)
deultsjn_q]e_sdral_ary_p6sgfl g$insideatresiaoroutside(necktumor)
d qutls ln h+lfrs-: atelectas i s, RDS
----u--
d:qv11r.a_respuetqry-rus&sj weak muscles as in preterm
Q e rs_l5t_eIc e _qf iatr$$e_rlng asp hyx!_a : ( m at erna l, fet a l, i d i o p athi c)

I TyOeS 15 e-a""2a21;

I Clinicol Picture: occording to (Apsar's scorine svste4)i

!. :
l- !.
- -_ -. -: I_i- :Abscnt
:: .:
-l.llg,r-tg : . r,- . : I : :
- - .. i: :: I.
_-.i-:t!t -_
- - -'].2.
-r-!gq
.
- l.
-

! 2. respiratory ratc ! Atrscnt ! Sl,,t' t'cuular ! Rcgl,lar, ('rvirtg


n,1,

, 3. nruscle tonc I r\trscnt rt'laccid) I Sn,,,.' lleriorr o['lirnbs I z\ctir e


, 4. Ilcllcxcs r--F t \o t'cspons rr-l
, tvcak lcsponse i--, Sncczt'or cough
t-r-

I 5. skin color I l'ale or lllue I I'}ink body. bluc Iinrbs ! Cornplctcly pirrk
Doneatl&5min
t Score 4-6:
D" surfactant & abnormal capillary permeability ) hyaline membrane (structurless
?,
eosinophilic membrane formed of fibrin)
( )
D rhc hrearh
rrru rirst
rrrur uru
(
S 2
D I The alveolirequirO, ot to expand & this amount of - I '(
( be
u. pressure must 'by ory musclet
I I )
D i ,"rre lung depends o, ,urru., 'ho:lipi rol), I
s%), phosp
S
N l-choline (75%),
(7 phosp ,
,S

I
)- .'r''; A
( trnclion ol suilactant I )(
) Decr!
r-
i 2- Helps or wall
of alveolar
i
S | ve inspiration L)
S
), i
| . Types:
'o
'*''
',$,.n.....,,.
,yp" I ....Hyaline membrane disease Orr,tD)*ts.*
I'i
'
( \ oTypell $
,t

s
I
I
)
atment of DM, steroids & good assessment (
)
,

disturbance of CNS control on the GIT)


e

cter ) increase meconium in the amniotic


sucked at delivery ) MAS.
estinal epithelial cells, secretions of the GIT,
kin cells.

meconium * amniotic fluid --, slightly green-


inimal significance)
dition) large amounts of meconium +

st breath.
er birth.

k
)
)
N t -i,.
l lf in
thin rnecon;
r^e ur.r lTr',r
co ni unrr Tt i c,r
t< .nrconin,. t
1

1 | J !
c, rrn
ni u,ra, cc
| G

I
l, | ..,o,
Cclof 'tt''^'.
| ,.u,
skaincl ct '
[5ht\lrcet
S
Thicr, grGGrr stqioel
' 'r f,f \srcerr lmt ' '::^
.f ctlor^r .r s !5ht
'"
AT *t
slqioel
I t
I | |Lrger o'"n hh I S

I I ar^3irlcnrr | ,,*,
Jrnell arnonnr
6on sislcnry or
arnonnl eY
Of | '*g"
or.
I rn '
t
.l.n,l^"*' I I olilch1lreffn. !r,gl

IS|--""'''"''l !p }ri)
| rorilchllreen.
nn aCooi Unn
S

-'7"''-'^
J.2..1rlsr,E
dishess
rnaller
nnafler +
) ffcJ,trtnt
NoffcJrtrtnt
Mo
I dishes s
I t
I I |

nr
I

)rA
D(
(5)

SD
D-71
( Kuphot-hemqtomq 0.5 -2.524 t:- a-<-l t

S
') (
( -,aalarrca
^- f^-^^^.
ventouse or ,{-l;,,--.,
forceps delivery, or even after
-f+^- ^ normal
a -^.*^l delivery.
,t^t:,,^-.,
D

S I Z.SutrpelosLeal: locllltzctl to sutures !* f issLre edenra; diffust (overlies ! D


) ri! ! ,,,or. rha' o,c bo,r.i i ?
(
\
3.nlastic,
-).r. l.t5UU, !!ll n.t pit
rto ilur on prcssLrrc
lrrr !,rr pru\sLrr" ;*!;';;,;, pits On
SOlt. pirs ., pres
pfes !i L)
7, , 4.Ovcrl)"ing skin is not'tttal l* O*crl*irrs skirr is : S
$ I s. pt.i'bc
_. Mai,'be
5. .
,,,.... "_ associarcct
associatccl skrrll .:,-.
rvirh skutt ft'actttrc.
fr':rcture. assoclatcd *
a* Not rrrn.i.,.a rv rure. !| )
s- ,; !*
!,. \\
2 " ':':.]':':
6.Disappeilrance u'ithin fcrv
6.Disapt)earance rt'ithin fctv n'ecks
n'ecks rs
ro
1r ^;;r;r;lin,.a
r\i-^.^.^,,^..^ rr,,irh
l)iu,nnr.r,.r
l)isapPcars ..,r,r^
u irr
ithin
i
l'ur,dnrs_ :
I'rru, rt.rrrr
7 i N

* {he -'
mo1her should be informed abou
it or apply anything over it

I Monooement
o Expectant treatment: spontaneous absor
o VitaminKl (1mg) is given I.M
o Aspiration l+ l+ l+ a.i'-r, if absorption
chronicity (but it is not done as it leads t
D'(
,1r
S ^E+i^r^^..., D

=
,{gpfr1xto )rupture of blood capillari

o Blood diseases hemophilia.

I Site of Hemorrhoge:
o $uMura[ hemorfiuge: in full term b

exclusively due to trauma


o snbor"ocfrnoid hemorrhage: iry is due t
trauma or hypoxia in preterm infants
o lntra-cerebra[ or c[QtH

I Clinicol feotures:
o Still-born infant or
o Born infant in a state of asphyxia
The infant is drowsy (J suckling, irreg
=
= With sharp cries
Rigidity, convulsions or paralysis may
=
= Anterior fontanel is tense & bulging.

I Investigotions:
reveals bloody CSF
puncture
= Lumbar
= Sonar and CT scan locate the site of h

I Monooement:
o Minimal handline. warmth, oxygen and n

o Drugs:
- Vit Kl (1 mg) IM (| bleeding)
- Luminal (J convulsions)
- Hypertonic solution (10% NACL
- Antibiotics (avolo
AntrDrotlcs (avoid pulmonary ln
in

o Lumbar puncture (to reduce the high intra


$ubdural hemonhage

).
S ,Q, honach",ail .r
,. ,.v...v, , ,.*o-
,,.DUU474CI1I1O14'.-".-"'.-U- S
,,
D I $ubarcchnoid | 7
( .!. causes: traunra . )
ss
)
S
sr.
2,
*
t
Clp, S.izures occurring on the 2"d day suspects subarachnoid
Prosnosis: Infants with minimal svmntoms have 907" chance of heins normal
bleerjing ?
D

D L_ .. r. onutt ).----. -. I (

'
)

; r4JD L4r\gll O-rV rr4JJ l4L9r Drlrrw AaU


!t shell
- of bone over the hematoma.,. ! w
!rrrtwhich mav oersist for several months. ! e
Jaaaaatltttlttaaallllf aaltaraaaalllllaatr
:
trlrrltltltattaallaalatrirlaatallllllllaa
Diagnosis of the
(
zrffi
2 Q)zrtebra[ Cofumn Cracture

IS
n ottisdueto
S.funB llonas
).
( ost comnton fracture in
2
)

({t
\x
S./l
,)
( r- forceps )

)a--r, . rJt\magement: resuscitation then earl


$(gnogement:
life saving + blood aspiration.
SD
IS ffacla[ Fofrt

- Patients should be examined at l0 days of age, to differentiate true nerve


S ,
,( , -----/
degeneration from neurapraxia (edenia of the nerve, resolves in 4-6 weeks) S
D
N7t
) I $rachlat patsl
Sracfrlqr Pqtst l (
S o Due to forcible loterol flexion of the heod. ),

')s
N71
It

SD
I ea, cyanosis, apnea (bilateral diaphragrnatic paralysis are rare) (
D
S !. oIr: ')

*o:-r--o--o-\z^-ao--o-*:--:::*:(
'
1- General bodY changes
I I !|raas, S
?
( ;o i"*
lst ar"rs.enlnsrrrrrrs
3doE: colostrumt
S D
7l

?
N
; After thot: established
: mitk
rrLu'trr\ $
,1,

(
S Jr+Jpnl E
i' released rrom the
I .\ ,iJ::"'*tin ' f
o Kidney: Return to normal by 6 w, calyceal dilatation persists for months. V
o Ureters: Ditotion may persist for several month s. )
o Bloddeti Ove? distention and incomplete evacuation )t Uft
o Urethro:
9 Stress Urinary Incontinence after difficult labor
' Retention moy occur due to:
- Lax abdominal wall
- Atony of the bladder
- Painful episiotomy or lacerations
o Volume I in 1" few days to excrete fluids retained during pregnancy
o Loctosurio & some peptone (due to autolysis of uterine muscle)
I Skin
o t Sweoting
o The obdominol woll gradually regain its tone (helped by exercise)
o Strioe rubro becomes white (albicans).
I (Rodrr yteiqht
----u---
o 9 After labor (evacuation of uterus)
o I During puerperium (loss of fluids due to polyuria & sweating)

o Gopping for some period


o Fourchette & fossa navicularis disappear.
o Wounds heal well unless infected

o Reappeoronce of rugae by 3'd wk '<

o 6roduotty decrease in size (but doesn't reac.h nrenartrrm size) E

I Qerrrix:
o Atter lobor: diloted 2-3 cm &
o 2- 3 doys regain its tone
o lst wk ) closed
o Slit like external os. (
o Phvsiolooical ectoDv may be present (no treatment)

^J
2
9 Size:
! After delivery: At umbilicus, 20 cm long (lkg)
! lwk: midway bet Umbilicus and S.P. (% kg)
! 2wks: pelvic organ (Y^kg)
! 4 wks: pre-pregnancy dimensiions (75 g) gc +.1;4 ++1. #l o.,+ 'e
line degene
9 The excess muscle & BVs: hyaline degene
)
c Glonds & bosol toyer: cover the raw surface in about 6wks (
)

)ooQ

s2
,(

D(
n l@ S

s Lox ) genital prolapse & incontinenc


o D-?glh its tone gradually (especially

n rtu; t
) s Dischorge of the genital tract during puerperium (
2

) - subinvolution
Persistent red lochia= ', (
S - offensive lochia = infection ,
7'

- Sudden suppression = severe infections. $


t.
s

(most important & most dan

nt (3'd day)

ESS

, UTI, wound in

associated genital tumor as fi


Typhoid.

( ' estisate a case of P.nvre b.)e. t

N r r r-l Uen0fal; UIJU, UflIlC a[aIySlS, OI(


ll I a

,:. o ryt: US, cervicovaginal culture


( i Investieation i
ta
!rrtrrrltttrtrrrrJtrttltrlrrtrltlllllllltrrtlrrrlllllllta
(.2\vaY^\Za\Zrv\\z\\zN/7a\z\\z\\7\S7rc

I 0aftnitionz
o Bacterial infection of the genital tract.

I CItioloqr:
- ---.-.--.- u c'
q Causative organism ql dl^^,,|:

(Usuolly mixed = polymicrobiol)


t G +ve: B-hemolytic Streptococci, Staph.
s G -ve: E.coli, Proteus & klebsiella.
t Anerobes: Anerobic Streptococci (most common), Bacteroids,
Clostridium.

o Source c,u- !h :

! Exogenous: droplet infection or instruments


g Endogenous: from commensals in the vagina
! Hematogenous: from a septic focus in the patient (by her hand or
blood)

. Commonest & usually mild


Anerobic
. Commensals
Streptococci o Becoming pathogenic in the absence of remnants
o Group A p-hemolytic Streptococci
Aerrcbic
o Most virulent
Streptococci . Responsible for most of the sever cases

Stoph. oureus o.Suppuration f pus formation

ction, bad general infection e.g. DM.


mental delivery, lacerations.

veins & blood


(
I
n i"
i"il:h, !
-ril,:
i:is t'rvvryrev w ..- , -,r,,tlGhtS
it
a
a

ir )

s io terus !
( i tuppressed lochia & leucocytic i

D 1 i uita infection:4tn dry, moderate rise ! (


( ii .-."," strept) on^ i! or ,".p.,
srrenr\ + goo- temp'' excessive locnla &
excesslve lochia suD-:
dg srru- ! E
-^r*^.^-^^^ ,..-,erobic
involuted uterus
X
"\
2 ! resistonce, due to remnonts or i :
N:
\\r..
I,:': : conceotton. :
:
i/l )
S io Ut.rur is subinvoluted + leucocytic !

L\ i . r...t..r...rr.r..rt rt..t .rrrr.r.....r.rrrrrrrrrr....lt.] 7/

D : -.. cute salpingo ! T ! (


llness in both fornices.
( i , aticspread) :B iontenderness ir)

D i t"r"r"tric obscE55: push the ute i! abscest sides or the uterus' Softening = (
:
( i: ,"
,, o,r.l Dlue. tt
utlltil side. rt PUruL) ingui i
atong urEur
points 4ruuts 1 E
s : a
Z ! ligament, rectum, vagina, extend to i i )
S i brtto.ks, thigh & perinephric space ! : ,.

N :* P.lui. Peritonitis or Pelvi


( i* otnt'alizedPeritonitis'
I\ O rsucp f
\ i<
x lxcr'liots/ tt,h'lt\n t

n
(
fe,A't '\ .l
fl,r S

\ \ \ |
s
D
\\ l^ lnlru,'<. (
D

t(
t bo'rri
k'3'PTresseJ rvL"Dr
5'Pft<'lr* lochi'

D_(
(\/)

S fic'S,*e o n ri/t )
\7\vza\z \za\za
.(
t @_s,nera[ $lnrptonrs
o General: FAHM
o Local: Pain & Discharge
f G_e_te_ys[_lrgns
o General FAHMRT
o Abdominal renderness, thoracic respiration & rigidity not marked.
o P/V Tenderness, Discharge

)S ro ridium), DIC
oJaundice septicemia with liver affection, hepatotoxic
Iilt j;:
useo [
ln ,::ffi,#r,
trea t
2 c-i Renal failure due to:
l\ - Hypovolemia, septic shock '4
- Hemolysis, liver cell failure, DIC
- Nephrotoxic drugs used in treatment D
(
S r cln qtions
qtions D

,ESR&CRP
| oCultures:b.lp-qd_&__v_?gin_e!_.d jq.qh.qrg_e

o Otlrers: Midstream urine analysis, Widal test & Chest X-ray.

) ! ization,antibiotics, avoid unnecessary PV (only 4-5) (


S s J visitors,
r, isolate
rrvr4Lv from
I I vrr! infected 9vrr" t
person
rrrf veuvs pvr ')
,(
E
2oN
N rr-/-r-.. ')

I
D om the uterus. (
S oval of sutures (lacera ')
; 5*/e/ Tosih'oq
Dief , ?0, I + Ttcni
o( fr'to
Coll 6mynttet
At/ "l-7 crict
F 1,,*;l
Au ti b;ohcr
AM h77,'hct

Septicemic shock
/P: tachycardia, tachypnea, hypotension, oliguria & anuria
tages:
l- First hyperthermia (warm hypotensive stage)
2- Then hypothermia (cold hypotensive)
Then irreversible st

You might also like